КОС ОУД.06 Основы безопасности и защита Родины ТД

Приложение 8.2
Государственное автономное профессиональное образовательное учреждение
Саратовской области
«Саратовский техникум отраслевых технологий»

КОМПЛЕКТ ОЦЕНОЧНЫХ СРЕДСТВ
по общеобразовательной дисциплине
«ОУД.06 Основы безопасности и защита Родины»
(38.02.08 Торговое дело)

Саратов 2024

Комплект контрольно-оценочных средств разработан на основе Федерального
образовательного стандарта среднего профессионального образования по
профессии/специальности СПО 38.02.08 Торговое дело программы учебной
дисциплины ОУД.06 Основы безопасности и защита Родины.
Составитель/разработчик (составители): Конюхов Алексей Владимирович,
преподаватель ГАПОУ СО «Саратовский техникум отраслевых технологий»

Содержание
1.
Паспорт комплекта фонда оценочных средств
2.
Критерии оценивания успеваемости
2.1. Критерии оценивания теоретических знаний
2.2 Критерии оценивания практических знаний
3. Комплекты оценочных средств
3.1. Комплекты оценочных средств для текущего контроля
3.2. Комплекты оценочных средств для итоговой аттестации

4
13
13
18
19
19
61

4

1. ПАСПОРТ КОМПЛЕКТА ФОНДА ОЦЕНОЧНЫХ СРЕДСТВ
Программой общеобразовательной дисциплины «ОУД.06 Основы безопасности
и защита Родины» предусмотрено проведение текущего контроля и промежуточной
аттестации.
Фонды оценочных средств нацелены на оценку достижений, общих и
профессиональных компетенций, дисциплинарных результатов.
Результаты
освоения
программы
общеобразовательной
учебной
дисциплины, подлежащие проверке

5

ОК и ПК

Дисциплинарные результаты

ОК 01. Выбирать способы
- сформировать представления о возможных источниках опасности в
решения задач
различных ситуациях (в быту, транспорте, общественных местах, в природной
профессиональной
среде, в социуме, в цифровой среде); владение основными способами
деятельности
предупреждения опасных и экстремальных ситуаций;
применительно к
- знать порядок действий в экстремальных и чрезвычайных ситуациях
различным контекстам
ОК 02 Использовать
- проявить нетерпимость к проявлениям насилия в социальном
современные средства
взаимодействии;
поиска, анализа
- знать о способах безопасного поведения в цифровой среде;
и интерпретации
- уметь применять их на практике;
информации,
- уметь распознавать опасности в цифровой
и информационные
среде (в том числе криминального характера, опасности вовлечения в
технологии для выполнения деструктивную деятельность) и противодействовать им
задач профессиональной
деятельности
ОК
03
Планировать - сформировать представления о ценности безопасного поведения для личности,
и
реализовывать общества, государства; знание правил безопасного поведения и способов их
применения в собственном поведении;
собственное
- владеть основами медицинских знаний: владеть приемами оказания первой помощи
профессиональное
и личностное развитие, при неотложных состояниях; знать меры профилактики инфекционных и
неинфекционных заболеваний, сохранения психического здоровья; сформировать
предпринимательскую
представления о здоровом образе жизни и его роли в сохранении психического и
деятельность
физического здоровья, негативного отношения к вредным привычкам; знать о
в профессиональной сфере, необходимых действиях при чрезвычайных ситуациях биолого-социального
использовать знания по характера;
финансовой
грамотности - сформировать представления о роли России в современном мире;
в различных жизненных угрозах военного характера; роли Вооруженных Сил Российской Федерации в
ситуациях
обеспечении мира; знать основы обороны государства и воинской службы;

Тип оценочных
мероприятия
- Фронтальный
опрос;
- Защита алгоритма
оказания первой
помощи;
- Презентации;
- Тестирование;
- Выполнение
заданий на
Дифференцированн
ый зачет

прав и обязанностей гражданина в области гражданской обороны; знать действия при
сигналах гражданской обороны
ОК 04
Эффективно
взаимодействовать
и работать в коллективе и
команде

- знать основы безопасного, конструктивного общения,
- уметь различать опасные явления в социальном взаимодействии, в том числе
криминального характера;
- уметь предупреждать опасные явления и противодействовать им

ОК
06
Проявлять
гражданскопатриотическую позицию,
демонстрировать
осознанное
поведение
на основе традиционных
российских
духовнонравственных ценностей, в
том
числе
с учетом гармонизации
межнациональных
и
межрелигиозных
отношений,
применять
стандарты
антикоррупционного
поведения

- сформированность представлений об опасности и негативном влиянии на жизнь
личности, общества, государства экстремизма, терроризма; знать роль государства в
противодействии терроризму; уметь различать приемы вовлечения в экстремистскую и
террористическую деятельность и противодействовать им; знать порядок действий при
объявлении разного уровня террористической опасности; знать порядок действий при
угрозе совершения террористического акта; совершении террористического акта;
проведении контртеррористической операции;
- сформированность представлений о роли России в современном мире; угрозах
военного характера; роли Вооруженных Сил Российской Федерации в обеспечении
мира; знание основ обороны государства и воинской службы; прав и обязанностей
гражданина в области гражданской обороны; знать действия при сигналах
гражданской обороны;
- знание основ государственной политики в области защиты населения и территорий
от чрезвычайных ситуаций различного характера; знание задач и основных принципов
организации Единой системы предупреждения и ликвидации последствий
чрезвычайных ситуаций, прав и обязанностей гражданина в этой области;
- знание основ государственной системы, российского законодательства, направленных
на защиту населения от внешних и внутренних угроз; сформированность представлений
о роли государства, общества и личности в обеспечении безопасности.

ОК 07
Содействовать
сохранению окружающей
среды, ресурсосбережению,

- сформировать представления о возможных источниках опасности в
различных ситуациях (в быту, транспорте, общественных местах, в природной среде,
в социуме, в цифровой среде); владеть основными способами предупреждения
7

применять
знания
об изменении климата,
принципы
бережливого
производства, эффективно
действовать
в чрезвычайных ситуациях

опасных и экстремальных ситуаций; знать порядок действий в экстремальных и
чрезвычайных ситуациях;
- сформировать представления о важности соблюдения правил дорожного движения
всеми участниками движения, правил безопасности на транспорте. Знать правила
безопасного поведения на транспорте, уметь применять их на практике, знать о
порядке действий в опасных, экстремальных и чрезвычайных ситуациях на
транспорте;
- знать о способах безопасного поведения в природной среде; уметь применять их на
практике; знать порядок действий при чрезвычайных ситуациях природного
характера; сформировать представления об экологической безопасности, ценности
бережного отношения к природе, разумного природопользования;
- знать основы пожарной безопасности; уметь применять их на практике для
предупреждения пожаров; знать порядок действий при угрозе пожара и пожаре в быту,
общественных местах, на транспорте, в природной среде; знать права и обязанности
граждан в области пожарной безопасности

ОК 08
Использовать
средства
физической
культуры для сохранения
и
укрепления
здоровья
в
процессе
профессиональной
деятельности
и
поддержания
необходимого
уровня
физической
подготовленности

- владеть основами медицинских знаний: владеть приемами оказания первой помощи
при неотложных состояниях; знать меры профилактики инфекционных и
неинфекционных заболеваний, сохранения психического здоровья; сформировать
представления о здоровом образе жизни и его роли в сохранении психического и
физического здоровья, негативного отношения к вредным привычкам; знать о
необходимых действиях при чрезвычайных ситуациях биолого-социального характера

ПК 1.6. Организовывать
выполнение торгово-

-сформированность представлений о возможных источниках опасности в различных
ситуациях (в быту, транспорте, общественных местах, в природной среде, в социуме, в
8

технологических
процессов, в том числе с
применением цифровых
технологий

цифровой среде); владение основными способами предупреждения опасных и
экстремальных ситуаций; знать порядок действий в экстремальных и чрезвычайных
ситуациях;
- знание основ безопасного, конструктивного общения, умение различать опасные
явления в социальном взаимодействии, в том числе криминального характера; умение
предупреждать опасные явления и противодействовать им;
- знание основ государственной системы, российского законодательства,
направленных на защиту населения от внешних и внутренних угроз;
сформированность представлений о роли государства, общества и личности в
обеспечении безопасности.
-Заботящийся о защите окружающей среды, собственной и чужой безопасности, в том
числе цифровой,
-готовый к профессиональной конкуренции и конструктивной реакции на критику.
-самостоятельный и ответственный в принятии решений во всех сферах своей
деятельности, готовый к исполнению разнообразных социальных ролей,
востребованных бизнесом, обществом и государством
-умение реализовывать лидерские качества в производственном процессе,
-стрессоустойчивость, коммуникабельность

ПК 2.2. Идентифицировать
ассортиментную
принадлежность
потребительских товаров

-сформированность представлений о возможных источниках опасности в различных
ситуациях (в быту, транспорте, общественных местах, в природной среде, в социуме, в
цифровой среде); владение основными способами предупреждения опасных и
экстремальных ситуаций; знать порядок действий в экстремальных и чрезвычайных
ситуациях;
-проявляющий уважение к эстетическим ценностям, обладающий основами
эстетической культуры,
- самостоятельный и ответственный в принятии решений во всех сферах своей
деятельности, готовый к исполнению разнообразных социальных ролей,
востребованных бизнесом, обществом и государством.
9

ПК 3.7. Организовывать
продажи
инфокоммуникационных
систем и (или) их
составляющих в новых
каналах сбыта, в том числе
с использованием
цифровых и
информационных
технологий
ПК 4.2 Осуществлять
подготовку, размещение
товаров в торговом зале и
выкладку на торговотехнологическом
оборудовании

- сформированность представлений о ценности безопасного поведения для личности,
общества, государства; знание правил безопасного поведения и способов их
применения в собственном поведении,
- сформированность представлений о возможных источниках опасности в различных
ситуациях (в быту, транспорте, общественных местах, в природной среде, в социуме, в
цифровой среде); владение основными способами предупреждения опасных и
экстремальных ситуаций; знать порядок действий в экстремальных и чрезвычайных
ситуациях;

ПК 4.6 Осуществлять
эксплуатацию торговотехнологического
оборудования

-сформированность представлений о ценности безопасного поведения для личности,
общества, государства; знание правил безопасного поведения и способов их
применения в собственном поведении
-сформированность представлений о возможных источниках опасности в различных
ситуациях (в быту, транспорте, общественных местах, в природной среде, в социуме, в
цифровой среде); владение основными способами предупреждения опасных и
экстремальных ситуаций; знать порядок действий в экстремальных и чрезвычайных
ситуациях;

сформированность представлений о возможных источниках опасности в различных
ситуациях (в быту, транспорте, общественных местах, в природной среде, в социуме, в
цифровой среде); владение основными способами предупреждения опасных и
экстремальных ситуаций; знать порядок действий в экстремальных и чрезвычайных
ситуациях;
-Проявляющий уважение к эстетическим ценностям, обладающий основами
эстетической культуры,
- Самостоятельный и ответственный в принятии решений во всех сферах своей
деятельности, готовый к исполнению разнообразных социальных ролей,
востребованных бизнесом, обществом и государством

10

-знание основ государственной политики в области защиты населения и территорий от
чрезвычайных ситуаций различного характера; знание задач и основных принципов
организации Единой системы предупреждения и ликвидации последствий
чрезвычайных ситуаций, прав и обязанностей гражданина в этой области;
- готовый к профессиональной конкуренции и конструктивной реакции на критику.
- самостоятельный и ответственный в принятии решений во всех сферах своей
деятельности, готовый к исполнению разнообразных социальных ролей,
востребованных бизнесом, обществом и государством,
-умение реализовывать лидерские качества в производственном процессе
-стрессоустойчивость, коммуникабельность
-опыт научно-исследовательской деятельности в рамках студенческого научного
сообщества

11

2. Критерии оценивания успеваемости обучающихся.
2.1. Критерии оценивания теоретических знаний
С целью проверки теоретических знаний по ОУД.06 Основы безопасности
жизнедеятельности могут использоваться методы устного и письменного контроля в
следующих формах:
 выполнение творческих заданий (подготовка сообщений, доклада, реферата);
 создание мультимедийных презентаций;
 ответы на контрольные вопросы;
 тестирование;
Требования к устным ответам
Результатом проверки уровня усвоения учебного материала является отметка.
При оценке знаний обучающихся предполагается обращать внимание на правильность,
осознанность, логичность и доказательность в изложении материала, точность
использования географической терминологии, самостоятельность ответа. Оценка
знаний предполагает учёт
индивидуальных особенностей обучающихся,
дифференцированный подход к организации работы.
Критерии оценки устного ответа:
Оценка
Условия, при которых выставляется оценка
Оценка 5 («отлично»)
показывает глубокое и полное знание и понимание
всего объёма программного материала; полное
понимание сущности рассматриваемых понятий,
явлений и закономерностей, теорий, взаимосвязей;
умеет составить полный и правильный ответ на
основе изученного материала; выделять главные
положения, самостоятельно подтверждать ответ
конкретными примерами, фактами; самостоятельно и
аргументированно делать анализ, обобщения,
выводы. Устанавливать межпредметные (на основе
ранее приобретенных знаний) и внутрипредметные
связи, творчески применять полученные знания в
незнакомой ситуации. Последовательно, чётко,
связно, обоснованно и безошибочно излагать
учебный материал; давать ответ в логической
последовательности с использованием принятой
терминологии;
делать
собственные
выводы;
формулировать точное определение и истолкование
основных понятий, законов, теорий; при ответе не
повторять дословно текст учебника; излагать
материал литературным языком; правильно и
обстоятельно отвечать на дополнительные вопросы
учителя.
Самостоятельно
и
рационально
использовать наглядные пособия, справочные
материалы, учебник, дополнительную литературу,
первоисточники; применять систему условных
обозначений при ведении записей, сопровождающих
ответ; использование для доказательства выводов из
наблюдений и опытов;
самостоятельно, уверенно и безошибочно применяет
полученные знания в решении проблем на

творческом уровне; допускает не более одного
недочёта, который легко исправляет по требованию
учителя; имеет необходимые навыки работы с
приборами, чертежами, схемами и графиками,
сопутствующими ответу; записи, сопровождающие
ответ, соответствуют требованиям
хорошее знание карты и использование ее, верное
решение географических задач.
Оценка 4 («хорошо»)

показывает знания всего изученного программного
материала. Даёт полный и правильный ответ на
основе изученных теорий; незначительные ошибки и
недочёты
при
воспроизведении
изученного
материала, определения понятий дал неполные,
небольшие неточности при использовании научных
терминов или в выводах и обобщениях из
наблюдений и опытов;
материал излагает в определенной логической
последовательности, при этом допускает одну
негрубую ошибку или не более двух недочетов и
может их исправить самостоятельно при требовании
или при небольшой помощи преподавателя; в
основном усвоил учебный материал; подтверждает
ответ конкретными примерами; правильно отвечает
на дополнительные вопросы учителя;
умеет самостоятельно выделять главные положения в
изученном материале; на основании фактов и
примеров обобщать, делать выводы, устанавливать
внутрипредметные связи. Применять полученные
знания на практике в видоизменённой ситуации,
соблюдать основные правила культуры устной речи и
сопровождающей
письменной,
использовать
научные термины;
в основном правильно даны определения понятий и
использованы научные термины;
ответ самостоятельный;
наличие неточностей в изложении географического
материала;
определения
понятий
неполные,
допущены
незначительные нарушения последовательности
изложения,
небольшие
неточности
при
использовании научных терминов или в выводах и
обобщениях;
связное и последовательное изложение; при помощи
наводящих
вопросов
учителя
восполняются
сделанные пропуски;
наличие конкретных представлений и элементарных
реальных понятий изучаемых географических
явлений;
понимание основных географических взаимосвязей;
знание карты и умение ей пользоваться;
13

Оценка 3
(«удовлетворительно»)

Оценка 2
(«неудовлетворительно»)

при решении географических задач сделаны
второстепенные ошибки.
усвоил основное содержание учебного материала,
имеет пробелы в усвоении материала, не
препятствующие
дальнейшему
усвоению
программного материала;
материал
излагает
несистематизированно,
фрагментарно, не всегда последовательно;
показывает
недостаточную
сформированность
отдельных знаний и умений;
выводы и обобщения аргументирует слабо, допускает
в них ошибки.
допустил ошибки и неточности в использовании
научной терминологии, определения понятий дал
недостаточно четкие;
не использовал в качестве доказательства выводы и
обобщения из наблюдений, фактов, опытов или
допустил ошибки при их изложении;
испытывает затруднения в применении знаний,
необходимых для решения задач различных типов,
при объяснении конкретных явлений на основе
теорий и законов, или в подтверждении конкретных
примеров практического применения теорий;
отвечает неполно на вопросы учителя (упуская и
основное), или воспроизводит содержание текста
учебника, но недостаточно понимает отдельные
положения, имеющие важное значение в этом тексте;
обнаруживает недостаточное понимание отдельных
положений при воспроизведении текста учебника
(записей, первоисточников) или отвечает неполно на
вопросы учителя, допуская одну-две грубые ошибки.
слабое знание географической номенклатуры,
отсутствие практических навыков работы в области
географии (неумение пользоваться компасом,
масштабом и т.д.);
скудны географические представления, преобладают
формалистические знания;
знание карты недостаточное, показ на ней сбивчивый.
не усвоил и не раскрыл основное содержание
материала;
не делает выводов и обобщений.
не знает и не понимает значительную или основную
часть программного материала в пределах
поставленных вопросов;
имеет слабо сформированные и неполные знания и не
умеет применять их к решению конкретных вопросов
и задач по образцу;
при ответе (на один вопрос) допускает более двух
грубых ошибок, которые не может исправить даже
при помощи учителя.
имеются грубые ошибки в использовании карты.
14

Требования к оформлению доклада
Доклад предоставляется в распечатанном виде, объёмом 3-5 страниц. Текст
доклада должен быть представлен в текстовом редакторе Word, шрифт – Times New
Roman 14, межстрочный интервал – 1.5 (полуторный). Поля: верхнее - 2, нижнее - 2,
левое- 3, правое - 1,5.
Доклад должен включать в себя: введение, основную часть, заключение, список
литературы (не менее 5 источников).
Критерии оценки доклада:
Оценка
Условия, при которых выставляется оценка
Оценка 5 («отлично»)
материал изложен в определенной логической
последовательности. Тема доклада раскрыта
полностью.
Оценка 4 («хорошо»)
тема раскрыта, но при этом допущены не
существенные ошибки, исправленные по
требованию преподавателя.
Оценка 3
тема раскрыта не полностью, допущена
(«удовлетворительно»)
существенная ошибка.
Оценка 2
содержании
доклада
не
раскрывает
(«неудовлетворительно»)
рассматриваемую
тему,
обнаружено
не
понимание основного содержания учебного
материала
Доклад может быть представлен как доклад-презентация. Необходимо
представить 5-7 слайдов. Время доклада -5 минут. Критерии оценки доклада такие же.
Дополнительно оценивается презентация.
Оформление слайдов
Стиль
Фон
Использование цвета
Анимационные эффекты

Представление
информации
Содержание информации

Расположение информации
на странице

Параметры
Соблюдать единого стиля оформления.
Фон не должен быть слишком темным или ярким,
чтобы не отвлекать внимания от содержания слайдов.
Слайд не должен содержать более трех цветов.
Фон и текст должны быть оформлены контрастными
цветами.
При оформлении слайда использовать возможности
анимации.
Анимационные эффекты не должны отвлекать
внимание от содержания слайдов.
Параметры
Слайд должен содержать минимум информации.
Информация должна быть изложена доступным
языком.
Содержание текста должно точно отражать
этапывыполненной работы.
Текст должен быть расположен на слайде так, чтобы
его удобно было читать.
В содержании текста должны быть ответы на
проблемные вопросы.
Текст должен соответствовать теме презентации.
Предпочтительно горизонтальное расположение
информации.
15

Размер шрифта

Выделения информации
Объем информации

Виды слайдов

Наиболее важная информация должна располагаться в
центре.
Надпись должна располагаться под картинкой.
Для заголовка – не менее 24.
Для информации не менее – 18.
Лучше использовать один тип шрифта.
Важную информацию лучше выделять жирным
шрифтом, курсивом,подчеркиванием
На слайде не должно быть много текста,
оформленного прописными буквами.
На слайде не должно быть много выделенного текста
(заголовки, важная информация).
Слайд не должен содержать большого количества
информации.
Лучше ключевые пункты располагать по одному на
слайде.
Для обеспечения разнообразия следует использовать
разные виды слайдов:
 с таблицами
 с текстом
 с диаграммами

Критерии оценивания презентаций:
Оценка
Условия, при которых выставляется оценка
Оценка 5 («отлично»)
выполненная презентация отвечает всем
требованиям критериев
Оценка 4 («хорошо»)
в презентации имеются незначительные
нарушения или отсутствуют какие-либо
параметры
Оценка 3
при
оценивании
половина
критериев
(«удовлетворительно»)
отсутствует
Требования к оформлению реферата
Реферат предоставляется в распечатанном виде, объёмом 10-15 страниц. Текст
реферата должен быть представлен в текстовом редакторе Word, шрифт TimesNewRoman 14, межстрочный интервал – 1.5 (полуторный), в таблицах возможен
межстрочный интервал – 1(одинарный), поля: верхнее - 2, нижнее - 2, левое- -3, правое
- 1,5.
Реферат должен включать в себя: содержание, введение, основную часть,
заключение, список литературы (не менее 5 источников).
Время на защиту реферата: 5 минут.
Критерии оценивания реферата:
Оценка
Условия, при которых выставляется оценка
Оценка 5 («отлично»)
материал изложен в определенной логической
последовательности. Тема реферата раскрыта
полностью.
Оценка 4 («хорошо»)
тема реферата раскрыта, при этом допущены не
существенные ошибки, исправленные по
требованию преподавателя
16

Оценка 3
(«удовлетворительно»)
Оценка 2
(«неудовлетворительно»)

тема раскрыта не полностью, допущена
существенная ошибка
при защите реферата обнаружено не понимание
основного содержания учебного материала
Выполнение тестирования

Критерии оценивания:
Оценка
Оценка 5 («отлично»)
Оценка 4 («хорошо»)
Оценка 3
(«удовлетворительно»)
Оценка 2
(«неудовлетворительно»)

Условия, при которых выставляется оценка
если студент при тестировании дал 85-100%
правильных ответов
если студент при тестировании дал 69-84%
правильных ответов
если студент при тестировании дал 51-68%
правильных ответов
если студент при тестировании дал менее 50%
правильных ответов

2.2. Критерии оценивания практических знаний
Оценка
Оценка 5 («отлично»)

Критерии оценивания
студент демонстрирует знание теоретического и
практического материала по теме практической работы,
определяет взаимосвязи между показателями задачи,
даёт правильный алгоритм решения, определяет
междисциплинарные связи по условию задания.

Оценка 4 («хорошо»)

студент демонстрирует знание теоретического и
практического материала по теме практической работы,
допуская незначительные неточности при решении
задач, имея неполное понимание междисциплинарных
связей при правильном выборе алгоритма решения
задания.

Оценка 3
(«удовлетворительно»)

студент затрудняется с правильной оценкой
предложенной задачи, даёт неполный ответ,
требующий наводящих вопросов преподавателя.

Оценка 2
(«неудовлетворительно»)

студент даёт неверную оценку ситуации, неправильно
выбирает алгоритм действий.

17

3. Комплекты оценочных средств.
3.1. Комплект оценочных средств для текущего контроля
Для осуществления оперативного контроля по темам изучаемой учебной дисциплины
используются следующие типы контрольных заданий:

ВХОДНОЙ КОНТРОЛЬ
I вариант

1. РСЧС состоит из:
Найдите ошибку.
а) территориальных;
б) функциональных;
в) ведомственных подсистем.
2. Территориальные подсистемы РСЧС создаются:
а) для предупреждения и ликвидации чрезвычайных ситуаций в городах и
районах;
б) для предупреждения и ликвидации чрезвычайных ситуаций в поселках и
населенных пунктах;
в) для предупреждения и ликвидации чрезвычайных ситуаций в субъектах РФ
в пределах их территорий и состоят из звеньев, соответствующих
административно-территориальному делению этих территорий.
3. Назовите федеральный орган в России решающий задачи безопасности
жизнедеятельности населения:
а) проведение аварийно-спасательных и других неотложных работ при
возникновении ЧС;
б) локализация зон ЧС и прекращение действия характерных для них опасных
факторов;
в) организация строительства жилья для пострадавшего населения.
4. Рабочими органами комиссий по чрезвычайным ситуациям
соответствующих органов государственной власти и местного
самоуправления является:
а) специально создаваемые штабы;
б) органы управления (комитеты, управления, отделы) по делам ГОЧС;
в) эвакуационные комиссии.
5. Назовите закон, в России определяющий правовые и организационные
нормы в области защиты от чрезвычайных ситуаций:
а) закон РФ «О безопасности»;
б) Федеральный закон «Об обороне»;
18

в) Федеральный закон «О защите населения и территорий от чрезвычайных
ситуаций природного и техногенного характера»;
г) Федеральный закон «О гражданской обороне».
6. Комиссия по чрезвычайным ситуациям органа местного самоуправления,
координирующим органом РСЧС на:
а) Региональном уровне;
б) федеральном уровне;
в) объектном уровне;
г) местном уровне.
7. К зоне чрезвычайной ситуации относятся:
а) территория, на которой прогнозируется ЧС;
б) территория, на которой расположены потенциально опасные объекты;
в) территория, на которой сложилась ЧС.
8. Международное гуманитарное право-это..
а) совокупность норм, направленных на защиту прав человека в мирное время;
б) совокупность норм, основанных на принципах гуманности т направленных
на защиту жертв вооруженных конфликтов и ограничение средств и методов
ведения войны;
в) совокупность норм, направленных на защиту жертв во время стихийных
бедствий.
9. Основными документами международного гуманитарного права являются:
а) Декларация прав человека;
б) Устав Организации Объединенных Наций;
в) Четыре Женевских конвенции и два Дополнительных протокола к ним.
10. В качестве закона, обеспечивающего зашиты медицинского персонала в
зоне вооруженного конфликта, может использоваться:
а) красный крест на белом поле;
б) белый флаг;
в) красный полумесяц на белом поле;
г) белый квадрат с красной полосой по диагонали.
11. Для обозначения людей, имеющих право принимать участие в военных
действиях, в международном праве используется термин:
а) репатрианты;
б) комбатанты;
в) интернированные.

19

12. Какими правилами обязана руководствоваться каждая воюющая сторона
при оказании помощи раненым согласно нормам международного
гуманитарного права? Укажите правильные ответы:
а) раненых нельзя оставлять на произвол судьбы, даже если они принадлежат к
стороне противника;
б) можно оказывать помощь только раненым собственной стороны;
в) между ранеными и больными не должно проводится никакого различия, по
каким бы то ни было соображениям, кроме медицинских;
г) каждая сторона обязана разыскивать и регистрировать все имеющиеся в
наличии данные, способствующие установлению личности раненых, больных
и умерших, попавших в их руки, как со своей, так и с неприятельской стороны.
13. Тяжелое, угрожающее жизни состояние, характеризующее резким
снижением артериального и кровяного давления, угнетением деятельности
центральной нервной системы - это..
а) обморок;
б) травматический шок;
в) коллапс.
14.Тяжелейшее состояние организма пострадавшего, наступившее в результате
травмы, - это..
а) травматический шок;
б) обморок;
в) коллапс.
Ответы
1
в

2
в

3
а

4
б

5
в

6
г

7
в

8
б

9
в

10 11 12 13 14
ав в авг в а
II вариант

1.Потенциально-опасным объектом называют:
А) Особо охраняемый объект
Б). Предприятие, на котором возможны хищения опасных веществ.
В). Предприятие, на котором возможны аварийные ситуации.
2.Услышав сигнал «Внимание всем!» необходимо:
А. Спуститься в подвал.
Б. Позвонить всем знакомым и предупредить об опасности.
В. Включить радио или ТВ, прослушать рекомендации.

20

3. Дополните фразу:» Радио- и телевизионные приёмники необходимо
включить на местной программе передач и прослушать сообщение органов
«ГОЧС» после сигнала:________(выберите правильный ответ):
А) внимание всем
б) SOS
в) говорит МЧС
Г) говорит Архангельск
4. Установите соответствие между группой средств индивидуальной защиты
и их разновидностью (ответ представьте цифрой с буквой, например, …2В…)
1.средства индивидуальной защиты органов дыхания
2.средства индивидуальной защиты кожи
А) общевойсковой защитный комплект
Б) противогаз
В) производственная одежда
Г) ватно-марлевая повязка
Д) противопыльная тканевая маска
Е) повседневная одежда, при необходимости пропитанная специальными
растворами
Ж) респиратор
З) Легкий защитный костюм Л-1
И) аптечка индивидуальная АИ-2
5. Из перечисленных веществ выбрать наиболее распространенные АХОВ:
А) перекись водорода
Б) хлор
В) соляная кислота
Г) аммиак
Д) карбонат натрия
Е) сероуглерод
6. Объект, при аварии на котором или при его разрушении могут произойти
массовые поражения людей, животных и растений АХОВ называется…
А) радиационно опасным объектом
Б) гидродинамическим опасным объектом
В) технически опасным объектом
Г) ядерным объектом
Д) химически опасным объектом
7. Поражающими факторами АХОВ являются:
А) ударная волна
б) пожары и взрывы
в) токсическое воздействие на организм человека и животного
г) стихийные бедствия
д) заражение местности, воздуха, водоёмов
21

8. Авария на химически опасном объекте, сопровождающаяся проливом или
выбросом аварийно химически опасного вещества, способная привести к
гибели или химическому заражению людей, сельскохозяйственных
животных и растений, заражению окружающей среды называется…
А) гидродинамическая авария
Б) радиационная авария
В) катастрофой
Г) химическая авария
9. Территория, в пределах которой в результате воздействия АХОВ
произошли массовые поражения людей, животных и растений называется…
А) зоной поражения АХОВ
Б) очагом поражения АХОВ
В) радиусом поражения АХОВ
10. Свойства ртути:
А) жидкий металл
Б) газообразное вещество, тяжелее воздуха
В) бесцветный газ без запаха
11. Признаки поражения парами ртути:
А) раздражение слизистых оболочек глаз, рвота, головокружение
Б) повышенная утомляемость, слабость, сонливость, головная боль.
В) резь в глазах, слезотечение, раздражительность, кашель.
12. Средства защиты при разливе ртути:
А) ватно-марлевая повязка, смоченная 2% раствором питьевой соды
Б) ватно-марлевая повязка, смоченная 5% раствором лимонной кислоты
В) ГП-5, ГП-7,ПДФ-2 (ДШ)
Г) полоскание рта 0,25% раствором марганцовки, чистка зубов
13. Как называется смещение костей относительно друг друга в области
сустава? А) Сдавливание
Б) Вывих
В) Перелом
Г) Открытый перелом
14. Противорадиационное укрытие предназначено для защиты людей от ...
А)... радиоактивного заражения
Б)... сильнодействующих отравляющих веществ
В)... ударной волны
1 2 3 4
в в а 1.бгдж
2.авезик

5
бг
е

6
д

7
абвд

8
г

9
б

10
а

11
б

12
г

13
б

14
а
22

1-2 В ЧЕМ ОСОБЕННОСТИ КАРТИНЫ ОПАСНОСТЕЙ СОВРЕМЕННОЙ
МОЛОДЕЖИ
Тест: Опасности современной молодежи
1. Что такое скулшутинг?
а-Вооруженное нападение в образовательном учреждении
б-Субкультура
в-Уличная акробатика
2. Что такое зацепинг?
а-Паркур
б-Проезд вне салона электрички или трамвая
в-Изучение подземных коммуникаций
3. Причины, по которым молодежь интересуется вредными привычками. Что
лишнее?
а-Любопытство
б-Это программа обязательного прохождения
в-Проявление себя
4. Какие из перечисленных признаков НЕ относятся к руфингу?
а-Лазанье по подвалам
б-Лазанье по крышам
в-Лазанье по небоскребам
5. Любые явления, угрожающие жизни и здоровью человека — это ...
а-преступление
б-безопасность
в-опасность
6. Как алкоголь влияет на работу головного мозга?
а-Затормаживает
б-Возбуждает
в-Улучшает память
7. Акрострит — это ...
а-лазанье по подвалам
б-прыжки с парашютом
в-акробатика
8. Что можно отнести к опасным увлечениям?
а-Танцы
б-Селфи
в-ЗОЖ
9. Попытка выделиться и примкнуть к меньшинству единомышленников — это ...
а-субкультура
б-проект
в-движение
10. Что не является причиной опасностей для молодежи?
а-Негативные отношения в семье
б-Моральная неустойчивость
в-Обдуманные решения
Правильные ответы:
1
2
3
а
б
б

4
а

5
в

6
а

7
в

8
б

9
а

10
в
23

3-4 ИЗМЕРЕНИЕ ОПАСНОСТИ УСТНЫЙ ОПРОС
1. Что понимается под опасностью?
2. Какие виды опасностей по вероятности воздействия на человека и окружающую
среду вы знаете?
3. По каким признакам измеряют опасности ?
4. Что понимается под риском?
5. Каковы причины возникновения рисков?
6. Что такое индивидуальный риск? Как он определяется?
7. Какие меры по управлению риском применяют для предотвращения возможных
опасностей и минимизации возможных последствий?
5-6 ПОНЯТИЕ О ЗАЩИТЕ ОТ ОПАСНОСТИ
Фронтальный, индивидуальный опрос
1. Какие цели достигаются в процессе анализа опасностей?
3. Какие вы знаете средства защиты от механического травмирования?
5. Какие принципы и методы защиты можно сформулировать на базе обобщенного
защитного устройства и как оценить ее эффективность?
6. Назовите методы защиты от вибраций и шума.
7. Назовите методы защиты от электромагнитных полей.
8. Каковы методы защиты от лазерного и ионизирующих излучений?
9. Как классифицируются помещения по пожаровзрывобезопасности?
10. Раскройте понятие огнестойкости.
11. Каковы средства локализации и тушения пожаров?
7-8 ВЫЯВЛЕНИЕ И ОПИСАНИЕ ОПАСНОСТЕЙ РАБОЧЕГО МЕСТА
РАБОТНИКА ТОРГОВЛИ
Фронтальный опрос
1.Дать формулировку основных понятий безопасности жизнедеятельности: «безопасность
жизнедеятельности»,
«опасность»,
«безопасность»,
«деятельность»,
«условия деятельности»,
«идентификация опасности»,
«риск опасности»,
«ущерб здоровью»,
«гомосфера»,
«ноксосфера»,
«приемлемый риск»,
«управление безопасностью жизнедеятельности».
2.Расскажите о концепции приемлемого (допустимого) риска как системы управления
риском, ее характеристиках
3.Каковы принципы и методы обеспечения безопасности жизнедеятельности?
9-10 ПРАКТИЧЕСКАЯ РАБОТА №1 АНАЛИЗ СВЯЗИ ВРЕДНЫХ ФАКТОРОВ НА
КОНКРЕТНОМ РАБОЧЕМ МЕСТЕ И ЗАБОЛЕВАНИЙ РАБОТНИКА ТОРГОВЛИ
Алгоритм анализа связи вредных факторов на конкретном рабочем месте и заболеваний
сотрудников:
24

Различные виды вредных факторов на рабочем месте. Рассматриваются физические,
химические, биологические и психологические воздействия.
Влияние вредных факторов на здоровье работников. Анализируется связь между
воздействием вредных факторов и возникновением заболеваний.
Методы оценки воздействия вредных факторов. Описываются способы измерения уровня
воздействия и его влияния на здоровье работников.
Заболевания, вызванные вредными факторами. Рассматривается патология, связанная с
определёнными видами вредных воздействий.
Законодательство о труде и вредных факторах. Анализируются нормы и правила,
устанавливающие требования к обеспечению безопасности и охраны здоровья
работников.
Профилактика заболеваний от вредных факторов. Рассматриваются методы и способы
профилактики заболеваний, вызванных вредными факторами на рабочем месте.
Эпидемиологические исследования в области вредных факторов. Анализируются
результаты эпидемиологических исследований, посвящённых влиянию вредных факторов
на здоровье работников.
Роль медицинских осмотров в выявлении воздействия вредных факторов.
Рассматривается важность регулярных осмотров для контроля состояния здоровья
работников.
Психологические аспекты воздействия вредных факторов на рабочем месте.
Анализируется влияние психического состояния работников на их здоровье при
воздействии различных негативных факторов.
Эффективность мер по предотвращению заболеваний от вредных факторов. Оценивается
результативность профилактических действий и их влияние на здоровье сотрудников
11 ОПРЕДЕЛЕНИЕ ОПАСНОСТИ НА ПРОИЗВОДСТВЕ
ТЕСТ
1. Технический принцип, заключающийся в обеспечении невозможности утечки
жидкой или газовой среды из одной зоны в другую - принцип …
блокировки
флегматизации
герметизации
слабого звена
2. НЕ физические факторы производственной среды
запыленность и загазованность
электромагнитное, УФ- и инфракрасное излучение
монотонность труда
движущиеся механизмы
3. Опасные (экстремальные) условия труда характеризуются …
уровнем производственных факторов, вызывающих максимальное напряжение
организма уровнем факторов среды, приводящих к функциональным изменениям
состояния организма
уровнем производственных факторов, создающих угрозу для жизни
4. Действие производственного шума на организм человека сводится к …
силикозам
понижению чувствительности роговицы
нарушению концентрации внимания

25

5. Управленческий принцип, опирающийся на распределение материальных благ и
моральных поощрений в зависимости от результатов труда работающего - принцип
… стимулирования
эффективности
последовательности информации
6. Ориентирующий принцип, направленный на поиск хотя бы одного элемента в
системе обстоятельств, искусственное удаление которого позволило бы не
допустить несчастного случая – принцип …
деструкции
системности
снижения опасности
ликвидации опасности
7. Группы вредных и опасных факторов производственной среды
физические, химические, биологические, психологические
физические, природные, технические, электромагнитные
физические, биологические, технические, электромагнитные
технические, химические, психофизиологические, функциональная
8. Принципы обеспечения безопасности труда
подавления химического загрязнения
подавления опасности в источнике ее возникновения
активности и нормализации
9. Организационный принцип, состоящий в одновременном применении нескольких
устройств, способов, приемов, направленных на защиту от одной и той же
опасности - принцип …
резервирования
эргономичности
вакуумирования
прочности
10. Факторы производственной среды и трудового процесса, которые могут быть
причиной острого заболевания, внезапного резкого ухудшения здоровья или
смерти называются … отдельные производственные факторы
вредные производственные факторы
опасными производственными факторами
неблагоприятные факторы производственной среды
11. Биологические факторы производственной среды
лекарственные средства
патогенные микроорганизмы
вредные вещества и промышленные яды
12. Вредные условия труда характеризуются …
наличием вредных производственных факторов, оказывающих
неблагоприятное воздействие на организм
уровнем факторов среды, приводящих к функциональным изменениям состояния
организма
уровнем производственных факторов, создающих угрозу для жизни
уровнем производственных факторов, вызывающих максимальное напряжение
организма
26

13. Герметизация оборудования и аппаратуры, использование дистанционного
управления, тепловой изоляции нагретых поверхностей относятся к методу
обеспечения безопасности…
нормализации ноксосферы
адаптации человека к среде
разделения гомосферы и ноксосферы
повышения защищенности человека
14. Пространство, в котором постоянно существует или периодически возникает
опасность ноксосфера
гомосфера
биосфера
техносфера
15. Организационный принцип, состоящий в том, что для обеспечения безопасности
учитываются антропометрические, психофизические и психологические свойства
человека при создании рабочего места, места отдыха и социально-бытовых нуждпринцип …
Резервирования
эргономичности
вакуумирования
прочности

12 ПРАКТИЧЕСКАЯ РАБОТА № 2: АЛГОРИТМ И ПРАВИЛА ДЕЙСТВИЙ
РАБОТНИКОВ ТОРГОВЛИ ПРИ ЧС

Разработка типовой инструкции

о порядке действий персонала при угрозе или возникновении
чрезвычайных ситуаций
1. Общие положения
1.1.Настоящая инструкция определяет порядок действий работников при угрозе и
возникновении чрезвычайных ситуаций (далее — ЧС) различного характера.
1.2.Все работники, независимо от занимаемой должности, обязаны четко знать и строго
выполнять установленный порядок действий при угрозе и возникновении ЧС, не допускать
действий, которые могут вызвать угрозу жизни и здоровья.
1.3.ЧС могут возникнуть внезапно или после различного по продолжительности периода
угрозы возникновения. Исходя из этого период угрозы возникновения ЧС должен быть в
максимальной мере использован для предотвращения ЧС или уменьшения возможного
ущерба. С этой целью, исходя из режима функционирования территориальной или местной
подсистемы Единой государственной системы предупреждения и ликвидации
чрезвычайных ситуаций (РСЧС), объект переводится в режим повышенной готовности или
в чрезвычайный режим.
2. ДЕЙСТВИЯ РАБОТНИКОВ ПРИ УГРОЗЕ ВОЗНИКНОВЕНИЯ ЧС
2.1. РЕЖИМ ПОВЫШЕННОЙ ГОТОВНОСТИ.
При ухудшении обстановки и получении информации об опасности или угрозе
возникновения ЧС — временно прекратить выполнение повседневных задач и
27

сосредоточить все силы и средства на выполнении работ по предотвращению или
уменьшению последствий возникшей угрозы, для чего необходимо знать, как правильно
действовать в той или иной ситуации.
При угрозе взрыва:
—сообщить о полученной информации оперативному дежурному управления по делам ГО
и ЧС, в ЕДДС администрации района
—безаварийно приостановить все работы, эвакуировать работников из помещений,
проверить наличие всех в установленном месте сбора;
—организовать наблюдение за обстановкой в помещениях и на прилегающей территории;
—встретить прибывшее спецподразделение органов внутренних дел и обеспечить
обследование территории и помещений;
При угрозе возникновения пожара:
—сообщить об угрозе возникновения на пульт пожарной охраны по телефону «01».
Назвать адрес, место возникновения пожара, свою должность, фамилию, имя, отчество;
—задействовать систему оповещения;
—безаварийно приостановить все работы, эвакуировать работников из помещений;
—выключить систему вентиляции, обесточить помещения, закрыть окна и двери;
—приступить к тушению очага возгорания и спасению ценного имущества, документации;
—после эвакуации организовать проверку наличия работников в месте сбора;
—выделить работника для встречи пожарной команды (знающего место пожара,
расположение пожарного гидранта и кранов);
— выставить посты безопасности для исключения возврата в здание работников;
— встретить прибывшее подразделение пожарной охраны и обеспечить доступ к очагу
возгорания;
При угрозе возникновения аварии на энергетических, инженерных и технологических
системах:
– оценить обстановку и ее возможные последствия в случае аварии;
– сообщить в аварийную службу организаций, обслуживающих энергетические,
инженерные и (или) технологические системы;
– организовать наблюдение за опасным участком и при необходимости начать вывод
воспитанников и работников из опасной зоны;
– далее действовать по плану эвакуации при возникновении пожара.
При угрозе химического заражения (подхода облака):
— организовать наблюдение за обстановкой в районе объекта. Оповестить и привести
сотрудников в готовность к возможным действиям в условиях ЧС, сократить до минимума
присутствие людей;
— организовать выдачу сотрудникам средств индивидуальной защиты (далее — СИЗ);
— подготовиться к отключению вентиляции и кондиционеров, создать запас воды или
готовиться к экстренной эвакуации;
— подготовить медикаменты и имущество для оказания первой медицинской помощи
пострадавшим.
28

При угрозе радиоактивного заражения:
— постоянно прослушивать городские программы радиовещания и телевидения для
получения информации Управления по делам ГО и ЧС по вопросам радиоактивного
заражения местности (далее — РЗМ);
—выдать сотрудникам СИЗ, в случае необходимости организовать изготовление ватномарлевых повязок;
—подготовиться к отключению вентиляционных систем и кондиционеров, создать запасы
материалов для герметизации помещений, запас воды в герметичной таре, быть готовым к
эвакуации;
—организовать накопление необходимых количеств препаратов стабильного йода;
—обеспечить постоянное взаимодействие с управлением по ГО и ЧС и комиссией по ЧС
города.
При угрозе возникновения стихийных бедствий (резком изменении температуры
воздуха, сильном ветре, ливневых дождях, снегопадах и т. п.):
—организовать наблюдение за состоянием окружающей среды;
—организовать посменное круглосуточное дежурство администрации;
—оценить противопожарное состояние, провести мероприятия по повышению уровня
готовности пожарного расчета, противопожарной защищенности, усилить контроль за
состоянием коммунально-энергетических сетей;
—Быть готовым к эвакуации.
— организовать взаимодействие с Председателем КЧС и ПБ Волжского района г. Саратова
по тел: и ЕДДС Волжского района г. Саратова по тел.
13-14 ВЫЯВЛЕНИЕ И ОПИСАНИЕ ОПАСНОСТИ НА ДОРОГАХ
Тестирование
1.При пользовании общественным транспортом посадка происходит через
А) переднюю дверь
Б) заднюю дверь
В) те которые открываются
2.Кто является участником дорожного движения?
А) водитель и пассажир
Б) водитель и пешеход
В) водитель, пешеход, пассажир
3.Событие, возникающее в процессе движения по дороге транспортного средства и с его участием,при
котором пострадали люди, груз, трансп.сред-ва
А) это транспортное происшествие
Б) это дорожно-транспортное происшествие
В) это дорожно-транспортное событие
4.Если трамвайные пути расположены на середине дороги, в каком случае можно идти к трамваю?
А) как только увидите трамвай
Б) при полной остановке трамвая

29

В) в любой момент
5.При движении по тротуару, какой стороны нужно придерживаться?
А) правой
Б) левой
В) середины
6.Сколько существует видов пешеходных переходов?
А) 2
Б) 3
В) 4
7.Что нужно сделать в первую очередь при выходе из подъезда?
А) обратить внимание, не приближается ли к вам транспортное средство
Б) закрыть за собой дверь
В) осмотреться не случилось ли по близости ДТП
8. Безопаснее всего переходить проезжую часть:
А) по подземным переходам;
Б) по наземным переходам (зебре).
В) на любом участке дороги.
9.Ожидать маршрутное транспортное средство разрешается:
А) на краю проезжей части;
Б) только на посадочной площадке;
В) на тротуаре, обочине или посадочной площадке.
10. Пересекать проезжую часть пешеходам рекомендуется:
А) перпендикулярно, более коротким путем;
Б) по диагонали;
В) как угодно.
Ответы
1.а, 2.в, 3.б, 4.б, 5.а, 6.б, 7.а, 8.а, 9.в, 10а.

15-16 ПРАКТИЧЕСКАЯ РАБОТА №3 «КАК ОЦЕНИТЬ РИСКИ НА ДОРОГАХ»
«Составление рекомендаций по правилам дорожного движения».
Цели:
обучающая: сформировать теоретическое обоснование о ПДД;
развивающая: развивать умения анализировать сложившуюся обстановку, научить
правильно и своевременно реагировать на чрезвычайную ситуацию;
воспитательная: воспитывать чувство ответственности, проявлять выдержку,
устанавливать последовательность действий при сложившейся ситуации.
Практическая часть
Вариант-1
Задание №1. Перечислите общие правила поведения участников дорожного движения
Ответ:
Участники дорожного движения (водитель, пешеход и пассажир) обязаны:
30

—знать и соблюдать относящиеся к ним требования правил дорожного движения,
сигналов светофора, знаков и разметки, а также выполнять распоряжения
регулировщиков;
—помнить, что в нашей стране установлено правостороннее движение транспортных
средств.
Участникам дорожного движения запрещается:
—повреждать или загрязнять покрытие дорог;
—снимать, загораживать, повреждать, самовольно устанавливать дорожные знаки,
светофоры и другие технические средства организации движения;
—оставлять на дороге предметы, создающие помехи для движения.
Задание №2. Безопасность пассажира, написать памятку
Ответ:
Ожидать автобус, троллейбус и трамвай можно только на посадочных площадках (на
тротуарах, на обочине дороги).
Посадку в транспортное средство начинают только при полной его остановке, соблюдая
очередность и не мешая другим пассажирам.
При посадке в трамвай, если трамвайные пути расположены посередине улицы и
нужно пересечь проезжую часть дороги, необходимо посмотреть в обе стороны
и, убедившись, что путь свободен, направиться к остановившемуся трамваю.
Войдя в салон транспортного средства, необходимо обратить внимание на то, где
расположены запасные и аварийные выходы.
При отсутствии свободных мест для сидения, можно стоять в центре прохода, держась
рукой за поручень или за специальное устройство.
Нельзя стоять у входной двери, а тем более опираться на нее, так как она в любой момент
может открыться.
Передвигаться по салону в общественном транспорте рекомендуется только при его
полной остановке.
Задание №3. Напишите правила движение пешехода по улице в сильный гололед
Ответ:
Перед выходом из дома следует подготовить обувь, чтобы повысить устойчивость при
ходьбе в гололед (натереть подошву наждачной бумагой, приклеить на подошву
изоляционную ленту, чтобы увеличить сцепление обуви с дорогой);
Из дома рекомендуется выходить с запасом времени, чтобы не спешить в пути.
При ходьбе наступать на всю подошву, расслабив ноги в коленях, быть готовым к
падению. Желательно, чтобы руки были свободны от сумок и прочих предметов.
При падении напрячь мускулы рук и ног, при касании земли перекатиться на бок.
Помните! Самое опасное падение —это падение на прямую спину и на расслабленные
прямые руки.
Вариант-2
Задание №1. Напишите правила перехода проезжей части дороги
Ответ:
Переходить проезжую часть дороги нужно по пешеходным переходам. Самый безопасный
переход — подземный или надземный. При их отсутствии переходить проезжую часть
можно на перекрестках по линии тротуаров или обочин.
В местах, где движение регулируется, для перехода проезжей части необходимо
руководствоваться сигналами регулировщика либо пешеходного светофора или
транспортного светофора.
При отсутствии в зоне видимости перехода или перекрестка разрешается переходить
дорогу под прямым углом к краю проезжей части на участках с разделительной полосой
там, где дорога хорошо просматривается в обе стороны.
31

На нерегулируемых пешеходных переходах можно выходить на проезжую часть дороги,
убедившись, что переход будет безопасен. Для этого необходимо внимательно посмотреть
сначала налево, потом направо, чтобы убедиться, что поблизости нет машин.
Нельзя выбегать на дорогу.
Перед переходом дороги надо замедлить шаг и оценить обстановку; даже при переходе
дороги на зеленый сигнал светофора необходимо осмотреться.
Не следует переходить проезжую часть дороги перед медленно идущей машиной, так как
можно не заметить за ней другую машину, идущую с большей скоростью.
Нельзя выходить на проезжую часть из-за стоящего транспортного средства или другого
препятствия, ограничивающего видимость проезжей части дороги, не убедившись в
отсутствии приближающихся транспортных средств.
Пешеходы, не успевшие закончить переход, должны остановиться на линии, разделяющей
транспортные потоки противоположных направлений.
Продолжать переход можно, лишь убедившись в безопасности дальнейшего движения и с
учетом сигнала светофора или регулировщика.
При приближении транспортных средств с включенным синим проблесковым маячком и
звуковым сигналом даже при зеленом сигнале светофора для пешеходов необходимо
воздержаться от перехода проезжей части дороги и уступить этим транспортным
средствам проезжую часть.
Задание №2. Безопасность пешехода на дороге, написать памятку
Ответ:
Пешеходы должны двигаться по тротуарам или пешеходным дорожкам, а при их
отсутствии — по обочинам, велосипедной дорожке или в один ряд по краю проезжей
части дороги.
Вне населенных пунктов при движении по краю проезжей части дороги пешеходы
должны идти навстречу транспортным средствам.
В случае если пешеход ведет велосипед, мотоцикл или мопед, он должен следовать по
ходу движения транспортных средств.
При следовании по улице пешеход должен стараться обходить стороной выезды из
гаражей, с автостоянок и других подобных мест, чтобы не попасть под выезжающий
автомобиль.
Пешеход не должен останавливаться в непосредственной близости от проходящего
автомобиля.
Задание №3. Перечислите меры безопасности при возникновении пожара в автобусе,
троллейбусе, трамвае
Ответ:
Немедленно сообщить о пожаре водителю и пассажирам, потребовать остановить
транспорт и открыть двери.
При блокировании дверей для эвакуации из салона транспортного средства
использовать аварийные люки в крыше и выходы через боковые стекла (при
необходимости можно выбить стекла ногами).
При эвакуации не допускать паники и выполнять указания водителя.
В любом транспортном средстве имеются материалы, которые при горении выделяют
ядовитые газы, поэтому необходимо покинуть салон быстро, но без паники, закрывая рот
и нос платком или рукавом одежды.
Помните! В троллейбусе и трамвае металлические части могут оказаться под
напряжением, поэтому, покидая салон, к ним лучше не прикасаться.
Выбравшись из салона, необходимо отойти подальше от транспортного средства, оказать
посильную помощь пострадавшим
17-18 КАК ЗАЩИТИТЬСЯ ОТ ОПАСНОСТЕЙ НА ДОРОГАХ
32

Тестирование Вариант 1
«КАК ЗАЩИТИТЬСЯ ОТ ОПАСНОСТЕЙ НА ДОРОГАХ»
1) С какого возраста разрешается обучать вождению на мотоцикле?
а. С 14 лет
б. С 16 лет
2) В каких случаях велосипедисту разрешается покидать крайнее правое положение на
проезжей части?
а. Для объезда
б. Для поворота налево или разворота
в. В обоих случаях
3) В каком возрасте разрешается выезжать на велосипеде на дороги общего пользования?
а. Не моложе 10 лет
б. Не моложе 14 лет
4) Можно ли велосипедисту повернуть налево с пересечением трамвайных путей?
а. Нельзя
б. Можно, если вблизи нет трамвая
5) По какой полосе можно проехать на велосипеде?
а. По крайней правой полосе
б. Как можно ближе к правому краю проезжей части
6) Имеет ли право велосипедист развернуться, не слезая с велосипеда, на дороге с двумя
полосами движения в каждом направлении?
а. Не имеет право
б. Имеет право
7) Велосипедист вытянул правую руку в сторону. Что это за сигнал?
а. Поворот направо
б. налево
8) Велосипедист поднял левую руку вверх. Что это за сигнал?
а. Поворот налево
б. Остановка
9) Велосипедист согнул правую руку в локте. Что это за сигнал?
а. Поворот направо
б. налево
10) Разрешается ли водителю мопеда движение по пешеходным дорожкам?
а. Не разрешается
б. Разрешается, если это не мешает пешеходам
11) В каком возрасте разрешается езда на мопеде по дорогам общего пользования?
а. С 16 лет
б. С 18 лет
12) Должен ли велосипедист руководствоваться сигналами светофора?
а. Нет
б. Да
13) Имеет ли право велосипедист ехать по проезжей части, если рядом есть велосипедная
дорожка?
а. Имеет право
б. Не имеет права
14) Нужно ли при движении на мопеде в дневное время включать фару?
а Да
б. Нет
15) Какие грузы запрещается перевозить на велосипеде и мопеде?

33

а. более 10 кг
б. Грузы, мешающие управлению или выступающие более, чем на 0,5м по длине и ширине
16) Разрешается ли перевозка детей на велосипеде?
а. Запрещается
б. Разрешается на специально оборудованном сиденье детей до 7 лет
17) Какое расстояние должно быть между группами велосипедистов при их движении по
дорогам?
а. 30 – 50м
б 80 – 100м
18) Сколько человек может быть в группе велосипедистов?
а. 10 человек
б. 8 – 10 человек
19) Значение слова «велосипед»:
а. Транспортное средство, кроме инвалидных колясок, имеющее 2 колеса и более и
приводимое в действие мускульной силой людей, находящихся на нем.
б. Двухколесное транспортное средство без мотора для взрослых и детей.
ОТВЕТЫ
1. а 2. в 3. б 4. а 5. б 6. а 7. Б 8. б 9. б 10. а 11. а 12. а 13.б 14. А 15. б 16.б 17 б 18.а 19. А
Тестирование Вариант 2

Список вопросов теста
Вопрос 1

Событие, возникшее в процессе движения по дороге транспортного средства и с его участием, при
котором погибли или ранены люди, повреждены транспортные средства, сооружения, грузы либо
причинен иной материальный ущерб.
Варианты ответов





Дорожное движение
Дорожно-транспортное происшествие
Нет верного ответа

Вопрос 2

Кто является участником дорожного движения?
Варианты ответов






Водитель
Пешеход
Пассажир транспортного средства
Все варианты верны

Вопрос 3

Обустроенная или приспособленная и используемая для движения транспортных средств полоса земли
либо поверхность искусственного сооружения - это
Варианты ответов





Дорога
Обочина
Тротуар

Вопрос 4

34

Какие дорожные знаки обозначают "Пешеходный переход"
Варианты ответов






А
Б
В
Г

Вопрос 5
Со скольки лет Правилами дорожного движения разрешается управлять велосипедом по дорогам?
Варианты ответов






13
14
15
16

Вопрос 6

Со скольки лет Правилами дорожного движения разрешается управлять мопедом по дорогам?
Варианты ответов






13
14
15
16

Вопрос 7

Что пассажирам запрещается?
Варианты ответов







при поездке на транспортном средстве, оборудованном ремнями безопасности, быть пристегнутыми ими, а при
поездке на мотоцикле - быть в застегнутом мотошлеме
отвлекать водителя от управления транспортным средством во время его движения
посадку и высадку производить со стороны тротуара или обочины и только после полной остановки транспортного
средства
при поездке на грузовом автомобиле с бортовой платформой стоять, сидеть на бортах или на грузе выше
бортов
открывать двери транспортного средства во время его движения

Вопрос 8

При отсутствии тротуаров, пешеходных дорожек, велопешеходных дорожек или обочин, а также в
случае невозможности двигаться по ним пешеходы могут двигаться
Варианты ответов



По краю проезжей части, навстречу транспортным средствам

35




По своему усмотрению
По велосипедной дорожке

Вопрос 9
Какая категория водительского удостоверения разрешает передвижение на скутере (мопеде)?
Варианты ответов






категория В
категория С
категория D
категория М

Вопрос 10

Вы (на синем автомобиле) намерены повернуть налево. Кому следует уступить дорогу?

Варианты ответов





Всем транспортным средствам.
Только легковому автомобилю и автобусу.
Только автобусу и мотоциклу.

19-20 КАК ОЦЕНИТЬ РИСКИ В СИТУАЦИИ ПОЖАРА В ОБЩЕСТВЕННОМ
МЕСТЕ (ЧС)
Составление инструкции по мерам пожарной безопасности по вариантам:
1 вариант: организационные меры в ТРЦ
2 вариант: технические меры, в быту
21-22 ВЫЯВЛЕНИЕ И ОПИСАНИЕ ОПАСНОСТИ В СИТУАЦИИ ПОЖАРА В
ОБЩЕСТВЕННОМ МЕСТЕ
Решение ситуационной задачи и составление алгоритма действий:
Ситуация: Вы находитесь в общественном месте (кинотеатр, музей, вокзал, рынок), там
возник пожар.
Составьте алгоритм Ваших действий при возникновении пожара в общественном месте.
1.Оценить обстановку и убедиться, что опасность реальна
2.Сообщить о пожаре, нажав на кнопку пожарного из вещателя
3.Продвигаться к ближайшему выходу, помогая детям, пожилым людям и тем, кто не
может двигаться по каким-либо причинам
4.При задымлении или в темноте двигаться, держась за стены или поручни, дышать через
носовой платок (шарф, косынку)
5.Сохранять спокойствие и хладнокровие, не поддаваться панике
6.Оказавшись в безопасном месте, оказать необходимую помощь пострадавшим

36

23-24 ПРАКТИЧЕСКАЯ РАБОТА № 4 КАК БЕЗОПАСНО ВЕСТИ СЕБЯ В
СИТУАЦИИ ПОЖАРА В ОБЩЕСТВЕННОМ МЕСТЕ
Практическая работа № 4
Тема Как безопасно вести себя в ситуации пожара в общественном месте
Цель практической работы: выявления и описание опасностей в ситуации пожара
Оборудование и материалы: презентация борьба с пожарами, тетрадь, ручка.
Задание:
Выписать
в
рабочую
тетрадь
основные
мероприятия по
защите
населения от средств поражения.
Отработка действий по
предупредительным сигналам и сигналам оповещения.
Отработка
порядка
получения
и
пользования средствами
индивидуальной защиты
Решение задачи:
Находясь в современном обществе, живя в большом городе, не так уж трудно оказаться в месте
массового скопления людей. Поэтому необходимо знать основные правила поведения в случае
возникновения пожаров и иных происшествий. При пожарах главным фактором, несущим угрозу
для жизни человека, зачастую становится не сам огонь, а дым. Поэтому первостепенная задача при
пожаре в общественных местах – это как можно быстрее покинуть опасную зону, выйти из
помещения, здания на улицу.
Услышав крики «Пожар!»:
сохраняйте спокойствие и выдержку, и призывайте к этому рядом стоящих людей;
оцените обстановку, убедитесь в наличии реальной опасности (возможно, кто-то этим криком хочет
привлечь внимание людей). - Стоя на месте, внимательно осмотритесь вокруг; увидев телефон или
кнопку вызова пожарной охраны, сообщите о пожаре;
если имеется возможность справиться с огнём, немедленно оповестите об этом окружающих,
потушите пожар;
при заполнении помещения дымом или отсутствии освещения постарайтесь идти к выходу, держась
за стены и поручни, дышите через носовой платок или рукав одежды, ведите детей впереди себя,
держа их за плечи;
в любой обстановке сохраняйте выдержку и хладнокровие, своим поведением успокаивая
окружающих, и не давайте разрастаться панике. Примите на себя руководство по спасению людей!
двигаясь в толпе, пропускайте вперёд детей, женщин и престарелых;
помогите тем, кто не может двигаться, растерялся, разговаривайте спокойно и внятно,
поддерживайте их за руки;
если вы находитесь в многоэтажном здании, не пытайтесь воспользоваться лифтами, спускайтесь
по лестнице вниз; не поддавайтесь желанию выпрыгнуть в окно с большой высоты;
при невозможности выбраться наружу, отступите в незанятые огнём помещения, защищайте органы
дыхания и зрения мокрым платком, полотенцем, одеждой, и там дожидайтесь помощи пожарных;
подавайте знаки прибывающим подразделениям пожарной охраны;
выбравшись из здания, окажите помощь пострадавшим.
В случае происшествия незамедлительно обращайтесь по телефонам
«101» или «112»!
Понятие: опасность пожара в общественном месте – это способность явлений, процессов горения,
горючих материалов и объектов причинять вред людям и материальным ресурсам;
Предметное действие: выявлять и описывать опасности в ситуации пожара в общественном месте;
Правило действия: чтобы выявить и описать опасности пожара нужно определить условия пожара,
при которых элемент системы «человек
– общественное место» становится причиной нанесения вреда человеку.
Алгоритм выявления опасностей в ситуации пожара в общественном месте на примере торгового
центра, кинотеатра, клубах)
выявить пространственную локализацию ЧС (в местах массового скопления людей (торговые
центры, бизнес-центры), в транспорте (учитывать особенности водного, наземного, воздушного,
железнодорожного транспорта);
определить объект защиты (сотрудник, посетитель);
37

выявить источники опасности (опасные факторы пожара) для объекта защиты и составить их
перечень (дым, пары вредных газов, открытое пламя, осколки, части разрушившихся строений,
высокая температура окружающей среды);
выявить условия активизации опасных факторов пожара (разрушение конструкций строения,
расплавление пластика, применение огнетушащих веществ, приток воздуха, вдыхание дыма);
провести классификацию источников опасности по поражающему воздействию на объект защиты
(воздействие на дыхательную, опорно-двигательный аппарат, кожные покровы);
определить варианты поражения организма объекта защиты (ожог, травма, потеря сознания,
удушье, летальное/смерть).

Источники пожарной опасности
Пожар-неконтролируемый процесс горения вне специального
очага, возникший непроизвольно или по злому умыслу, в ходе
которого выделяются
Неиспр тепло и дым, а также который
сопровождается материальным
ущербом и угрожает здоровью
авные
или жизни людей
электро
Гор
Наруш
ючи
ение
е

прави
Подземные
На

транспортн
ых
средствах

Наруш
Лесные, ения
торфяные
при

Внутренние
(закрытые) – скрытые
пути
распространения
пламени

Пожары в
зданиях и
сооружениях
Случай
ный

Домашние

Откр
Техногенные
ыты

пожары в шахтах
и рудниках

Полевые,
степные

Наружные
(открытые)хорошо
просматриваются
пламя и дым

25-26 ВЫЯВЛЕНИЕ И ОПИСАНИЕ ОПАСНОСТИ В СИТУАЦИИ ЗАХВАТА
ЗАЛОЖНИКОВ В ОБЩЕСТВЕННОМ МЕСТЕ (ЧС)
Составить алгоритм выявления опасностей при захвате в заложники
Алгоритм выявления опасностей при захвате в заложники является важной составляющей
тактики правоохранительных органов в подобных ситуациях. Хотя конкретные алгоритмы
могут различаться в зависимости от страны и организации, ниже представлен общий
обзор шагов, которые могут быть включены в такой алгоритм:
1. Оценка ситуации:
 Определение места захвата и его характеристик.
 Определение количества заложников и их состояния.
 Оценка численности и вооруженности злоумышленников.
 Определение возможных укрытий и путей эвакуации.
2. Сбор информации:
 Получение информации о злоумышленниках, их мотивах и требованиях.
 Сбор информации о здании или месте захвата, его плане и особенностях.
38

Анализ предыдущих случаев захвата в заложники для выявления общих паттернов
и тактик.
3. Коммуникация и переговоры:
 Установление контакта с злоумышленниками и начало переговоров.
 Установление коммуникационной связи с заложниками для получения
информации о их состоянии и потребностях.
 Анализ и оценка информации, полученной в результате переговоров.
4. Анализ рисков и опасностей:
 Оценка потенциальных угроз для заложников и правоохранительных органов.
 Идентификация возможных сценариев развития ситуации и их последствий.
 Определение наиболее вероятных мест, где могут находиться злоумышленники и
заложники.
5. Планирование и координация операции:
 Разработка стратегии освобождения заложников с минимальными рисками.
 Определение ролей и задач каждого участника операции.
 Координация действий между различными службами и специалистами.
6. Осуществление операции:
 Проведение специальной операции по освобождению заложников с применением
соответствующих тактик и техник.
 Мониторинг ситуации и реагирование на изменения в реальном времени.
 Минимизация рисков для заложников и правоохранительных органов.
Важно отметить, что алгоритмы и тактики в захвате заложников постоянно развиваются и
совершенствуются. Они зависят от множества факторов, включая местные законы,
доступные ресурсы и опыт правоохранительных органов.


27-28 КАК ОЦЕНИТЬ РИСК РЕАЛИЗАЦИИ СИТУАЦИИ ЗАХВАТА
ЗАЛОЖНИКОВ/СТРЕЛЬБЫ В ОБЩЕСТВЕННОМ МЕСТЕ (ЧС)
Составить памятку по порядку действий при захвате в заложники

39

29 КАК БЕЗОПАСНО ВЕСТИ СЕБЯ В СИТУАЦИИ ЗАХВАТА ЗАЛОЖНИКОВ В
ОБЩЕСТВЕННОМ МЕСТЕ (ЧС)
Тест Правила безопасного поведения при угрозе террористического акта

1. Что в настоящее время является наиболее характерными действиями террористов?
Организация взрывов в местах массового скопления людей и жилых домах
Угон воздушного транспорта
Захват морского транспорта
2. Как стоит передвигаться, если случился взрыв?
Осторожно, не прикасаясь к повреждённым конструкциям и проводам
Максимально быстро в противоположную взрыву сторону
По пластунски в сторону взрыва
3. Что нельзя делать в разрушенном или повреждённом помещении?
Включать свет
Двигаться
Дышать
4. Как защитить органы дыхания в случае задымления?
Смоченной тряпкой
40

Дышать в бумажный пакет
Дышать как можно реже
5. Как выбраться из дома, если выход отрезан завалом или пламенем?
Выйти на балкон и звать на помощь
Спрятаться в ванной
Постараться разобрать завал и прорываться сквозь огонь
6. Как стоит контактировать с преступниками в случае захвата?
Как можно меньше
Агрессивно
Стараться морально подавить их
7. Куда следует действовать при захвате транспортного средства, в котором вы
находитесь?
Как можно скорее передвигаться к выходу
Оставаться на месте
Постараться привлечь внимание преступников
8. Что нужно делать при силовом освобождении заложников?
Бежать к выходу
Подобрать оружие и помогать правоохранительным органам
Попытаться укрыться за предметами и прикрыться от пуль
9. Где опасно находиться во время перестрелки?
У окон
В ванной
За бетонной стеной
10. Какое лучше выбрать укрытие на улице при перестрелке?
Автомобиль
Кирпичный забор
Любой объект
30 ПРАКТИЧЕСКАЯ РАБОТА № 5 АЛГОРИТМ ПОВЕДЕНИЯ ПРИ ЗАХВАТЕ
ТЕРРОРИСТАМИ ТОРГОВОГО ЦЕНТРА
Тема: Алгоритм поведения при захвате террористами ТЦ
Правила безопасного поведения при угрозе террористического акта, захвате в
качестве заложника.
Цель занятия: Ознакомление с глобальной проблемой современности –
терроризмом
Ход работы:
1.
Создать памятку по действиям при угрозе теракта
2.
Правила поведения при захвате в качестве заложника
3. Выполнить первое задание по мерам безопасности при террористических актах
по группам:
Группа № 1 – «Население»
Практическое задание: «Прогремел взрыв. Ваши действия?»

Группа № 2 – «Секретари»
Практическое задание: «Возник пожар. Ваши действия?»

Группа № 3 – «Персонал объекта»
Практическое задание: «Вы оказались под завалом. Ваши действия?»

Группа № 4 – «Руководители объектов»
Практическое задание: «Вас захватили в заложники. Как вы будете себя вести?»
4. Выполнить второе задание по мерам безопасности при террористических актах
по группам:


41



Группа № 1 – «Население»

Практическое задание:
Составить памятки населению по предотвращению террористических актов и при
обнаружении предмета , похожего на взрывоопасный.

Группа № 2 – «Секретари»
Практическое задание:
Составить памятки секретарю при получении угрозы по телефону и контрольный
лист наблюдений при угрозе по телефону.

Группа № 3 – «Персонал объекта»
Практическое задание:
Составить памятки персоналу объекта по предотвращению террористических актов
и при обнаружении предмета, похожего на взрывоопасный.

Группа № 4 – «Руководители объектов»
Практическое задание:
Разработать рекомендации руководителю по предотвращению террористических
актов и при получении угрозе о взрыве.
5. Сдать работу.
ПАМЯТКА СЕКРЕТАРЮ (ДИСПЕТЧЕРУ) ПРИ ПОЛУЧЕНИИ УГРОЗЫ ПО
ТЕЛЕФОНУ
1. Установить прочный контакт с анонимом:
- представиться (назвать своё имя, отчество, должность);
- попытаться успокоить говорившего;
- заверить, что его требования будут немедленно переданы администрации.
2. Выяснить требования анонима и получить информацию о характере угрозы:
- внимательно выслушать и под диктовку записать все требования;
- под любым предлогом предложить повторить свои требования;
- задать уточняющие вопросы о характере угрозы и времени её реализации, стимулируя
анонима рассказать как можно больше;
3. Выяснить мотивы действий анонима:
- задать вопрос о целях, которые преследует аноним, при этом ответы анонима
выслушивать внимательно, проявляя участие;
- предложить анониму другие пути реализации его интересов.
4. В процессе выхода из контакта с анонимом следует повторить основные моменты
беседы с ним, сказать, что его требования будут переданы администрации. Попытаться
под любым благовидным предлогом убедить его повторить звонок.
5. По окончании разговора немедленно заполнить “Лист наблюдений при угрозе по
телефону”(Приложение4).
6. Сообщить о происшествии:
- в правоохранительные органы по телефону “02”;
- администрации объекта.
42

7. Если у вас нет определителя номера или он не сработал, не вешайте телефонную
трубку, а положите её рядом. С другого телефона позвоните на телефонный узел с
просьбой установить номер телефона, откуда был сделан звонок.
8. Не сообщайте об угрозе никому, кроме тех, кому об этом необходимо знать в
соответствии с инструкцией, чтобы не вызвать панику и исключить непрофессиональные
действия.
КОНТРОЛЬНЫЙ ЛИСТ НАБЛЮДЕНИЙ ПРИ УГРОЗЕ ПО ТЕЛЕФОНУ
1. Пол: мужчина, женщина
2. Возраст: подросток, молодой, средний, пожилой
3. Речь: темп ________________________________
наличие акцента _______________________________
наличие дефектов, присутствие попыток изменения
тембра______________________________________
4. Голос: громкость_____________________________
высота ______________________________________
5. Предполагаемое психологическое состояние: возбуждённое, вялое, неадекватное,
спокойное, иное ____
6. Наличие звукового (шумового) фона
_____________________________________________________

ПАМЯТКА НАСЕЛЕНИЮ ПО ПРЕДОТВРАЩЕНИЮ ТЕРРОРИСТИЧЕСКИХ
АКТОВ

Будьте наблюдательны! Только вы способны своевременно обнаружить
предметы и людей, посторонних в вашем подъезде, дворе, улице.

Будьте бдительны! Обращайте внимание на поведение окружающих,
наличие бесхозных и не соответствующих обстановке предметов.

Наведите порядок в собственном доме: установите железную дверь с
домофоном в подъезде, ежедневно проверяйте закрытие подвалов, чердаков и
технических зданий.

Организуйте соседей на дежурство вблизи дома и оказание помощи
правоохранительным органам в охране общественного порядка.

Не делайте вид, что ничего не замечаете при опасном поведении попутчиков
в транспорте! Вы имеете полное право защищать свой временный дом.

Никогда не принимайте на хранение или для передачи другому лицу
предметы, даже самые безопасные.

Обнаружение подозрительного предмета в неподходящем (безлюдном)
месте не должно ослабить вашу осторожность. Злоумышленник мог попросту бросить его,
испугавшись чего-либо.


43

Даже если у вас имеется личный опыт общения со взрывчатыми
веществами, не пытайтесь манипулировать ими. Самодельные взрыватели бывают
сверхчувствительными и изощрённо хитроумны.

Не приближайтесь, а тем более — не прикасайтесь к
подозрительному предмету: это может стоить вам жизни.

Расскажите своим детям о взрывных устройствах.

Научите своих детей мерам безопасности: не разговаривать на улице
и не открывать дверь незнакомым, не подбирать бесхозные игрушки, не
прикасаться к находкам и т.п.


ПАМЯТКА НАСЕЛЕНИЮ ПРИ ОБНАРУЖЕНИИ ПРЕДМЕТА. ПОХОЖЕГО НА
ВЗРЫВООПАСНЫЙ
Заметив подозрительные предметы или чью-либо деятельность, например:
— вещь без хозяина,
— предмет, не соответствующий окружающей обстановке,
— устройство с признаками взрывного механизма,
— бесхозный автотранспорт, припаркованный непосредственно к зданиям,
— разгрузку неизвестными лицами различных грузов в подвальные и
чердачные помещения, арендованные квартиры, канализационные люки и т.п.
1. Не подходите и не прикасайтесь к подозрительному предмету.
2. НЕМЕДЛЕННО сообщите ближайшему должностному лицу (водителю
трамвая, охраннику, дежурному) или
3. ПОЗВОНИТЕ по телефону “02”, а также по контактным телефонам
вашего отделения милиции, территориального управления, ЖЭС, домоуправления
и т.п.
ПАМЯТКА ПЕРСОНАЛУ ОБЪЕКТА ПО ПРЕДОТВРАЩЕНИЮ
ТЕРРОРИСТИЧЕСКИХ АКТОВ
o
Будьте наблюдательны! Только вы можете своевременно обнаружить
предметы и людей, посторонних на вашем рабочем месте.
o
Будьте внимательны! Только вы можете распознать неадекватные
действия посетителя в вашем рабочем помещении или вблизи него.
o
Будьте бдительны! Каждый раз, придя на своё рабочее место,
проверяйте отсутствие посторонних предметов.
o
Потренируйтесь: кому и как вы можете быстро и незаметно передать
тревожную информацию.
o
Соблюдайте производственную дисциплину! Обеспечьте надёжные
запоры постоянно закрытых дверей помещений, шкафов, столов.
Не будьте равнодушны к поведению посетителей! Среди них может
оказаться злоумышленник.
o
Заблаговременно представьте себе возможные действия преступника
вблизи вашего рабочего места и свои ответные действия.
o
Помните, что злоумышленники могут действовать сообща, а также
иметь одну или несколько групп для ведения отвлекающих действий.
o
Получив сведения о готовящемся теракте, сообщите об этом только в
правоохранительные органы по тел. “02” и руководителю объекта. Оставайтесь на
рабочем месте. Будьте хладнокровны. Действуйте по команде.
o

Если вы оказались в заложниках:
44

1) не подвергайте себя излишнему риску;
2) будьте покладисты и спокойны;
3) если преступники находятся в состоянии алкогольного или наркотического
опьянения, постарайтесь ограничить с ними всякие контакты, так как действия их могут
быть непредсказуемы;
4) при первой же возможности постарайтесь сообщить о своем местонахождении
родным или в милицию;
5) постарайтесь установить контакт, вызвать гуманные чувства и завести разговор,
не наводя их на мысль, что вы хотите что-либо узнать;
6) не позволяйте себе падать духом. Используйте любую возможность поговорить с
самим собой о своих надеждах и желаниях;
7) внимательно следите за поведением преступников и их намерениями. При
первой же удобной и безопасной возможности будьте готовы спасаться бегством.
Если вы находитесь в местах большого скопления агрессивно настроенных людей
(митинги, забастовки):
1) держитесь дальше от центра;
2) дальше от группы экстремистов;
3) держитесь уверенно на ногах;
4) держитесь подальше от милиции и от экстремистов любых видов – «красных»,
«коричневых», «черных», «голубых», «зеленых», соблюдайте нейтралитет;
5) если у вас сумка или пакет в руках, будьте бдительны – вам могут подбросить
наркотики, оружие, боеприпасы и прочие «улики»;
6) к толпам людей любых видов лучше вообще не приближаться и к нарядам
милиции тоже;
7) проявляйте максимальную бдительность и внимание на улицах города – это в
целях сохранения вашей свободы и безопасности.
31-32 ИСТОРИЯ СОЗДАНИЯ ВООРУЖЕННЫХ СИЛ РОССИИ.
ТЕСТ ПО ОБЖ: История создания Вооружённых Сил Российской Федерации
1. В какой период в России была сформирована регулярная армия?
1991—1993 гг.
1938—1941 гг.
1812—1814 гг.
1701—1711 гг.
2. Как назывался орган власти в Российской Империи, бывший прообразом Министерства
обороны?
Высший военный совет
Военная комиссия
Военная коллегия
3. Сколько гвардейских корпусов было в сухопутной армии при Екатерине II?
Четыре
Три
Десять
Двадцать
4. В каком году в России была введена воинская повинность для всех мужчин старше 21
года?
1705
1874
1914
1918
45

5. Укажите вид огнестрельного оружия, бывший на вооружении в российской пехоте с
1891 года.
Винтовка Мосина
Пистолет Макарова
Пистолет-пулемет Токарева
6. С 1918 по 1946 года армия России и СССР называлась РККА. Как расшифровывается
это сокращение?
Российская Красная кавалерийская армия
Российская крестьянская Красная армия
Рабоче-крестьянская Красная армия
7. Какая война оказалась последней для Российской империи?
Первая мировая война
Русско-турецкая война 1877-1878 гг.
Русско-японская война
8. Какой из этих полководцев не участвовал в Великой Отечественной войне?
М. В. Фрунзе
Г. К. Жуков
И. Х. Баграмян
Л. А. Говоров
Р. Я. Малиновский
9. Какой из этих видов Вооруженных сил СССР был создан позже других?
Ракетные войска стратегического назначения
Сухопутные войска
Военно-морской флот
10. Укажите официальное наименование армии в России после 1991 года.
Армия Российской Федерации
Вооруженные силы Российской Федерации
Силы обороны Российской Федерации
33-34 ОСНОВНЫЕ ПОНЯТИЯ О ВОИНСКОЙ ОБЯЗАННОСТИ
ТЕСТ по ОБЖ «Основные понятия о воинской обязанности»
1. Под воинской обязанностью понимается:
а) прохождение военной службы в мирное и военное время, самостоятельная подготовка к
службе в Вооруженных Силах;
б) установленный законом почетный долг граждан с оружием в руках защищать свое
Отечество, нести службу в рядах Вооруженных Сил, проходить вневойсковую подготовку
и выполнять другие связанные с обороной страны обязанности;
в) долг граждан нести службу в Вооруженных Силах только в период военного положения
и в военное время.
2. Комиссия по постановке граждан на воинский учет утверждается главой органа местного
самоуправления (местной администрации) в следующем составе:
а) военный комиссар района (города) либо заместитель военного комиссара, специалист по
профессиональному психологическому отбору, секретарь комиссии, врачи-специалисты;
б) представитель командования военного округа, представитель органа местного
самоуправления, врачи-специалисты;
в) военный комиссар района (города), руководитель (заместитель руководителя) органа
внутренних дел, секретарь комиссии, врачи (хирург, терапевт, невропатолог);
3. Заключение по результатам освидетельствования категории «Б» означает:
а) годен к военной службе с незначительными ограничениями;
б) временно не годен к военной службе; в) ограниченно годен к военной службе.
4. Запас Вооруженных Сил Российской Федерации предназначен:
46

а) для развертывания в военное время народного ополчения;
б) для создания резерва дефицитных военных специалистов;
в) для развертывания армии при мобилизации и ее пополнения во время войны.
5. В связи с выполнением обязанностей военной службы гражданам предоставляются
определенные преимущества, которые называются льготами. Это льготы:
а) по налогам и сборам, жилищные льготы, льготы по отдельным вопросам брачносемейного законодательства, льготы в области здравоохранения, в области образования, по
перевозкам, льготы за службу в отдаленных местностях, за выполнение задач при
вооруженных конфликтах;
б) по налогам и сборам, жилищные льготы, льготы в области здравоохранения, в области
образования и культуры, по перевозкам, за службу в отдаленных местностях, за выполнение
задач при вооруженных конфликтах;
в) за службу в отдаленных местностях, льготы за выполнение задач при вооруженных
конфликтах, в области здравоохранения, в области образования и культуры, жилищные
льготы, льготы по налогам, в области материальной и уголовной ответственности, по
перевозкам.
6. Общие правила и обязанности военнослужащих, взаимоотношения между ними,
обязанности основных должностных лиц полка и его подразделений, а также правила
внутреннего распорядка определяет:
а) Устав внутренней службы Вооруженных Сил Российской Федерации;
б) Строевой устав Вооруженных Сил Российской Федерации;
в) Дисциплинарный устав Вооруженных Сил Российской Федерации.
7. Из приведенных ниже ответов определите, кто освобождается от призыва на военную
службу:
а) имеющие ребенка, воспитываемого без матери, имеющие двух или более детей, имеющие
ребенка в возрасте до 3 лет, мать которых, кроме них, имеет двух и более детей в возрасте
до 8 лет или инвалида с детства и воспитывает их без мужа (жены);
б) признанные не годными или ограниченно годными к военной службе по состоянию
здоровья, проходящие или прошедшие военную или альтернативную гражданскую службу
в Российской Федерации, прошедшие военную службу в другом государстве, имеющие
ученую степень кандидата или доктора наук;
в) граждане, достигшие возраста 18 лет и не состоящие на воинском учете, не прошедшие
медицинское освидетельствование в полном объеме и в установленные сроки, граждане,
временно пребывающие за границей.
8. Окончанием военной службы считается день:
а) в который истек срок военной службы;
б) подписания приказа об увольнении с военной службы;
в) передачи личного оружия другому военнослужащему.
9. Какую ответственность несут военнослужащие за проступки, связанные с нарушением
воинской дисциплины, норм морали и воинской чести:
а) административную; б) уголовную; в) дисциплинарную.
10. В качестве знака, обозначающего желание воюющей стороны эвакуировать раненых и
потерпевших кораблекрушение, а также гражданских лиц из зоны боевых действий
используется знак:
а) белый квадрат с красной полосой; б) синий равносторонний треугольник на оранжевом
фоне; в) белый флаг; г) красный крест или красный полумесяц на белом фоне.
11. Военная служба исполняется гражданами:
а) только в Вооруженных Силах Российской Федерации;
б) в Вооруженных Силах Российской Федерации, пограничных войсках Федеральной
пограничной службы Российской Федерация и войсках гражданской обороны;
47

в) в Вооруженных Силах Российской Федерации, других войсках, органах и
формированиях.
12. Уклонившимся от исполнения воинской обязанности считается гражданин:
а) не явившийся по вызову военного комиссариата в указанный срок без уважительной
причины;
б) явившийся по вызову военного комиссариата без необходимых документов;
в) не явившийся по вызову военного комиссариата в указанный срок, даже имея
уважительную причину.
13. Заключение по результатам освидетельствования категории «Г» означает:
а) не годен к военной службе; б) временно не годен к военной службе; в) годен к военной
службе с незначительными ограничениями.
14. Граждане, состоящие в запасе, могут призываться на военные сборы
продолжительностью:
а) до одного месяца, но не чаще одного раза в пять лет;
б) до двух месяцев, но не чаще одного раза в три года;
в) до трех месяцев, но не чаще одного раза в четыре года.
15. Уставы Вооруженных Сил Российской Федерации подразделяются на:
а) уставы родов войск и строевые; б) тактические, стрелковые и общевоинские; в) боевые и
общевоинские.
6. Какой устав определяет предназначение, поря! док организации и несения гарнизонной
и карауль! ной служб, права и обязанности должностных лил гарнизона и военнослужащих,
несущих эти службы! а также регламентирует проведение гарнизонных мероприятий с
участием войск:
а) Устав гарнизонной и караульной службы Boopyженных Сил Российской Федерации;
б) Строевой устав Вооруженных Сил Российской Федерации;
в) Устав внутренней службы Вооруженных Сил Российской Федерации.
7. Из приведенных ниже ответов определите, к кому предоставляется отсрочка от призыва
на военную службу:
а) отбывающим наказание в виде обязательных или исправительных работ, находящимся
под apecтом или осужденным, находящимся в местах лишения свободы, имеющим
неснятую или непогашенную судимость за совершение преступления, в отношении
которых ведется следствие или уголовное дело пере! дано в суд;
б) временно не годным к военной службе по состоянию здоровья, постоянно работающим
в сельской местности врачам — на время этой работы, получающим послевузовское
образование и постоянно работающим на педагогических должностях в сельской местности
— на время этой работы;
в) прошедшим альтернативную службу в РФ, имеющим детей в возрасте от 3 до 18 лет,
проходящим государственную службу в органах местного самоуправления.
18. Каким законодательным актом установлена система воинских званий для всех
составов военнослужащих:
а) Законом Российской Федерации «О безопасности»; б) Федеральным законом Российской
Федерации «О статусе военнослужащих»;
в) Федеральным законом Российской Федерации «О воинской обязанности и военной
службе».
19. На военнослужащего, совершившего дисциплинарное правонарушение, могут
налагаться только те дисциплинарные взыскания, которые определены:
а) в Дисциплинарном уставе Вооруженных Сил Российской Федерации; б) в Уголовном
кодексе Российской Федерации; в) в Гражданском кодексе Российской Федерации.
20. По служебному положению и воинскому званию военносл. могут быть:
48

а) начальниками и подчинёнными; б) командирами и солдатами; в) срочной и контрактной
службы.
ОТВЕТЫ:
1-б, 2-а, 3-а, 4-в, 5-б, 6-а, 7-б, 8-а, 9-в, 10-г;
11-в, 12-а, 13-б, 14-б, 15-в, 16-а, 17-б, 18-в, 19-а, 20-а.
35-36 ОСНОВНЫЕ ПОНЯТИЯ О ПСИХОЛОГИЧЕСКОЙ СОВМЕСТИМОСТИ
ЧЛЕНОВ ВОИНСКОГО КОЛЛЕКТИВА (ЭКИПАЖА, БОЕВОГО РАСЧЕТА).
ТРЕНИНГ БЕСКОНФЛИКТНОГО ОБЩЕНИЯ И САМОРЕГУЛЯЦИИ
Тренинг по отработке стратегий решения конфликтов
Различают следующие конфликты:
Внутренние конфликты — это взаимодействие противоположных сторон внутри данного
объекта. Процесс развития объекта характеризуется не только развертыванием
внутренних конфликтов, но и постоянным взаимодействием его с внешними условиями,
со средой.
Внешние конфликты — это взаимодействие противоположностей, относящихся к разным
объектам, например между обществом и природой, организмом и средой и т. п.
Антагонистические конфликты — это взаимодействие между непримиримо
враждебными социальными группами и силами.
Термин «антагонизм» распространен в биологии и медицине: антагонизм ядов, лекарств,
микробов, антагонизм мышц, зубов и т. п. В своем чистом виде антагонизм проявляется
редко — в ситуации рыночной конкуренции, войны, революции, спортивных состязаниях
и т. п.
Стратегия поведения в конфликтной ситуации:
Приспособление — одна сторона во всём соглашается с другой, но имеет своё мнение,
которое боится высказывать.
Избегание — уход от конфликтной ситуации.
Компромисс — совместное решение, удовлетворяющее обе стороны.
Соперничество — активное противостояние другой стороне.
Сотрудничество — попытка прихода к совместному решению.
Правила бесконфликтного общения:





В своей речи следите за словами , которые могут вызвать обиду или негативную
реакцию сотрудника и спровоцировать конфликтную ситуации в коллективе.
Подходите к любому сослуживцу с позиций гуманистического отношения :
доброжелательно, сочувственно, уважительно.
Старайтесь быть уравновешенными , спокойными и уверенными в общении с
коллегами. Помните, что уверенность и высокомерие не тождественные понятия.
При агрессии, направленных против Вас, смените на некоторое время тему
разговора.

5. … не ущемляйте права человека с которым взаимодействуете. Не задевайте его чувств,
говоря о качествах его личности.
49

6. …всегда своевременно разъясняйте возникшие в ходе обсуждения
недоразумения, задавая уточняющие вопросы.
7. … умейте признавать свои ошибки …
8…. Свободно и своевременно извиняйтесь , если знаете о своей ошибке. Умейте
признавать свою неправоту. В другой раз Вы сможете рассчитывать на такое же
признание со стороны Ваших коллег;
Профилактика появления конфликта
… соблюдение социальной дистанции.
… внесение разнообразия в служебных отношениях.
… сглаживайте острые ситуации.
… старайтесь быть «своим» человеком для всех коллег.
… помните о значимости каждого сослуживца.
Шесть способов расположить людей к себе
1. Проявляйте искренний интерес к другим людям. 2. Улыбайтесь. 3. Помните, что для
человека звук его имени является самым сладким и самым важным. 4. Будьте хорошим
слушателем. Поощряйте других рассказывать Вам о себе. 5. Ведите разговор в круге
интересов Вашего собеседника. 6. Давайте людям почувствовать их значительность и
делайте это искренне.
Десять способов изменить человека, не нанося ему обиды и не возбуждая
негодования
1. Начинайте с похвалы и искреннего признания достоинства человека. 2. Обращая
внимание людей на их ошибки, делайте это в косвенной форме. 3. Прежде чем
критиковать других, скажите о своих собственных ошибках 4. Задавайте вопросы вместо
того, чтобы отдавать приказы. 5. Дайте человеку возможность сохранить свое лицо. 6.
Хвалите каждый, даже скромный успех человека и будьте при этом искренни и щедры в
похвалах. 7. Создавайте человеку доброе имя, чтобы он стал жить в соответствии с ним. 8.
Пользуйтесь поощрениями, сделайте так, чтобы недостатки, которые Вы хотите исправить
в человеке, выглядели легко исправимыми, а дело, которыми Вы его хотите увлечь, легко
выполнимым. 9. Делайте так, чтобы людям было приятно исполнять то, что Вы хотите. 10.
Критика бесполезна так как человек вынужден обороняться, чтобы не терять свое лицо.
Двенадцать способов склонять людей к своей точке зрения
1. Единственный способ добиться наилучшего результата в споре - это уклониться от
спора.
2. Проявляйте уважение к мнению другого. Никогда не говорите человеку, что он не прав.
3. Если Вы не правы, признайте это сразу и чистосердечно.
4. Вначале проявите свое дружеское отношение.
5. Пусть Ваш собеседник с самого начала будет вынужден отвечать Вам "да", "да"
6. Дайте собеседнику возможность выговориться.
7. Пусть Ваш собеседник почувствует, что идея принадлежит ему самому.
8. Честно попытайтесь увидеть вещи с точки зрения другого.
50

9. Проявляйте сочувствие к мыслям и желаниям других.
10. Взывайте к благородным побуждениям.
11. Придавайте своим идеям наглядность, инсценируйте их.
12. Бросайте вызов если хотите склонить мужественных людей с
37-38 КАК СТАТЬ ОФИЦЕРОМ РА. ОСНОВНЫЕ ВИДЫ ВОЕННЫХ
ОБРАЗОВАТЕЛЬНЫХ УЧРЕЖДЕНИЙ ПРОФЕССИОНАЛЬНОГО
ОБРАЗОВАНИЯ
Тестирование
Как стать офицером Российской Армии
Вопрос № 1
Решение о направлении кандидатов в военные образовательные учреждения профессионального
образования для прохождения профессионального отбора принимается призывными
комиссиями военных комиссариатов районов, объявляется кандидатам в десятидневный срок
гуманитарных, социально-экономических, естественно-научных специальных дисциплин в
объёме требований государственных образовательных стандартов
комиссиями военных комиссариатов районов, оформляется протоколом и объявляется
Вопрос № 2
Защита выпускной квалификационной работы в высших военно-учебных заведениях является
обязательной для большинства специальностей
обязательной для большинства гражданских
по индивидуальным планам, которые могут предусматривать сокращение общего срока
обучения до одного года
Вопрос № 3
Окончившие учебное заведение с золотой медалью и дипломом с отличием пользуются
преимущественным правом выбора
места жительство
места службы
место получения образования
Вопрос № 4
За время обучения курсанты приобретают фундаментальные знания в области
гуманитарных, социально-экономических, естественно-научных в объёме требований
государственных образовательных стандартов высшего профессионального образования
гуманитарных, социально-экономических, естественно-научных, общепрофессиональных и
специальных дисциплин в объёме требований государственных образовательных стандартов
высшего профессионального образования
гуманитарных, социально-экономических, естественно-научных специальных дисциплин в
объёме требований государственных образовательных стандартов
51

Вопрос № 5
Все курсанты после успешного завершения ими программ обучения проходят
ОГЭ
итоговую государственную аттестацию
ГТО
Вопрос № 6
В соответствии с положениями Федерального закона "О воинской обязанности и военной службе"
в военные образовательные учреждения профессионального образования имеют право поступать
граждане, не проходившие военную службу, в возрасте от 16 до 22 лет
граждане, не проходившие военную службу, в возрасте от 18 до 22 лет
граждане, не проходившие военную службу, в возрасте от 16 до 25 лет
Вопрос № 7
Офицерский состав - это
административно-правовая категория лиц, имеющих военную и военно-специальную
подготовку
правовая категория лиц, имеющих военную и военно-специальную подготовку
становой хребет любой армии, главный организатор и непосредственный исполнитель задач
Вопрос № 8
Граждане, прошедшие военную службу по контракту, а также проходящие или прошедшие
военную службу по призыву, при зачислении в указанные образовательные учреждения
заключают контракт до
одиночный ответ:
середины обучения
конец обучения
начала обучения
Вопрос № 9
Курсантам, успешно прошедшим итоговую государственную аттестацию
присваивается квалификация по полученной специальности
о индивидуальным планам, которые могут предусматривать сокращение общего срока
обучения до одного года
присваивается квалификация по полученной специальности и выдаётся диплом
Вопрос № 10
Офицерский корпуса-это
правовая категория лиц, имеющих военную подготовку
становой хребет любой армии, главный организатор и непосредственный исполнитель задач
правовая категория лиц, имеющих военную и военно-специальную подготовку
Вопрос № 11
Курсанты второго и последующих курсов могут переводиться на обучение
52

комиссиями военных комиссариатов районов, объявляется кандидатам в десятидневный срок
по индивидуальным планам, которые могут предусматривать сокращение общего срока
обучения до одного года
по индивидуальным планам, которые могут предусматривать сокращение общего срока
обучения до одного года
Вопрос № 12
Основное предназначение академий - готовить высококвалифицированных офицеров, уже
имеющих
образование военных академиях
высшее образование и опыт офицерской службы, на высшие должности
гуманитарных, социально-экономических, естественно-научных специальных дисциплин в
объёме требований государственных образовательных стандартов
Вопрос № 13
Подготовка офицерских кадров для Вооружённых Сил Российской Федерации осуществляется в
военных академиях, в военных университетах и военных институтах
военных академиях
гуманитарных, социально-экономических, естественно-научных специальных дисциплин в
объёме требований государственных образовательных стандартов
Вопрос № 14
Предварительный отбор кандидатов проводится призывными комиссиями военных комиссариатов
районов до 15 мая года поступления на учёбу и включает
определение их соответствия требованиям, предъявляемым к поступающим, и пригодности к
обучению в военных образовательных по результатам медицинского осмотра
определение их соответствия требованиям, предъявляемым к поступающим, и пригодности к
обучению в военных образовательных учреждениях профессионального образования по
результатам медицинского осмотра
определение их соответствия требованиям , и пригодности к обучению в военных
образовательных учреждениях профессионального образования по результатам медицинского
осмотра
Вопрос № 15
Общий объем учебной работы курсантов составляет
не более 54 часов в неделю
не более 60 часов в неделю
не более 64 часов в неделю
Вопрос № 16
В заявлении указываются
фамилия, имя и отчество, год, число и месяц рождения, адрес места жительства,
наименование военного образовательного учреждения профессионального образования и
специальность, по которой кандидат желает обучаться
фамилия, имя и отчество, год, число и месяц рождения, адрес места жительства,
наименование военного образовательного учреждения профессионального образования и
53

специальность, по которой кандидат желает обучаться
год, число и месяц рождения, адрес места жительства, по которой кандидат желает обучаться
Вопрос № 17
Военные институты являются
основным звеном в системе подготовки военных кадров для замещения первичных
офицерских должностей
гуманитарных, социально-экономических, естественно-научных специальных дисциплин в
объёме требований государственных образовательных стандартов
высшее образование и опыт офицерской службы, на высшие должности
Вопрос № 18
Лица из числа гражданской молодёжи, желающие поступить в военные образовательные
учреждения профессионального образования подают...
заявления в военный комиссариат района
заявления в военный пункт
заявления в воинскую казарму

39-40 СТРОЕВАЯ ПОДГОТОВКА
Тестирование
1. Что такое строй?
1. Установленное уставом размещение военнослужащих, подразделений и частей для их
совместных действий в пешем порядке и на машинах.
2. Размещение военнослужащих.
3. Размещение частей.
2. Что такое фланг?
1. Фланг — начало строя.
2. Конец строя.
3. Правая и левая оконечность строя
3. Что такое шеренга?
1. Шеренга — строй, в котором военнослужащие размещены один возле другого на двух
линиях.
2. Шеренга — строй, в котором военнослужащие размещены один возле другого на одной
линии.
3. Шеренга — строй, в котором военнослужащие размещены один возле другого на трех
линиях.
4. Что такое интервал?
1. Интервал — расстояние по фронту между военнослужащими, подразделениями и
частями.
2. Интервал — расстояние в глубину между военнослужащими, подразделениями и
частями.
5. Что такое дистанция?
1. Дистанция — расстояние по фронту между военнослужащими, подразделениями и
частями.
2. Дистанция — расстояние в глубину между военнослужащими, подразделениями и
частями.
6. Что такое ряд?
1. Ряд – двое военнослужащих, стоящих в двухшереножном строю в затылок один
другому.
54

2. Ряд – двое военнослужащих, стоящих в одношереножном строю в затылок один
другому.
7. Что такое колонна?
1. Колонна — строй, в котором военнослужащие расположены по фронту.
2. Колонна — строй, в котором военнослужащие расположены в затылок друг другу.
8. Что такое развернутый строй?
1. Развернутый строй — подразделения расположены на одной линии по фронту в
одношереножной или двухшереножном строю или в линию колонн на интервалах,
установленных уставом или приказом командира.
2. Развернутый строй — подразделения расположены на двух линиях в глубину
одношереножной или двухшереножном строю или в линию колонн на интервалах,
установленных уставом или приказом командира.
3. Развернутый строй — подразделения расположены на трех линиях по фронту в
одношереножной или двухшереножном строю или в линию колонн на интервалах,
установленных уставом или приказом командира.
9. Кто такой направляющий?
1. Направляющий – военнослужащий (подразделение, машина), движущийся вторым в
указанном направлении.
2. Направляющий – военнослужащий (подразделение, машина), движущийся головным в
указанном направлении.
3. Направляющий – военнослужащий (подразделение, машина), движущийся последним в
указанном направлении.
10. Кто такой замыкающий?
1. Замыкающий – военнослужащий (подразделение, машина), движущийся первым в
колонне.
2. Замыкающий – военнослужащий (подразделение, машина), движущийся вторым в
колонне.
3. Замыкающий – военнослужащий (подразделение, машина), движущийся последним в
колонне.
11. По какой команде принимается строевая стойка?
1. «Смирно!» или «Становись!».
2. «Равняйсь!».
3. «Вольно!».
12. Принимается ли строевая стойка без команды?
1. Нет.
2. Да.
13. Можно ли разговаривать по команде «Вольно!»?
1. Нет.
2. Да.
14. По какой команде разрешено поправить оружие, обмундирование и снаряжение?
1. «Вольно!».
2. «Заправиться!».
15. Какой бывает шаг?
1. Строевой.
2. Походный.
3. Строевой и походный.
16. При прохождении торжественным маршем применяется?
1. Строевой шаг.
2. Походный шаг.
17. Движение строевым шагом начинается по команде….
1. «Шагом - марш!».
55

2. «Строевым шагом – марш!»
18. На занятиях по строевой подготовке применяется….
1. Строевой шаг.
2. Походный шаг.
19. Движение шагом осуществляется с темпом…..
1. 90-100 шагов в минуту.
2. 110-120 шагов в минуту.
3. 130-140 шагов в минуту.
20. Размер шага…
1. 40-50 см.
2. 50-60 см.
3. 70-80 см.
41-42 ОГНЕВАЯ ПОДГОТОВКА. ПОРЯДОК НЕПОЛНОЙ СБОРКИ И РАЗБОРКИ
ММГ АК-74
Отработка порядка неполной разборки и сборки ММГ АК-74
Тестирование «Назначение и боевые свойства автомата Калашникова»
1. Для чего предназначен автомат Калашникова?
1. Для уничтожения живой силы противника.
2. Для уничтожения легкобронированных сил противника.
2. Автомат Калашникова является….
1. Коллективным оружием.
2. Индивидуальным оружием.
3. Для поражения противника в рукопашном бою, что присоединяется к автомату?
1. Присоединяется штык.
2. Присоединяется нож.
3. Присоединяется штык-нож.
4. Какой огонь ведется из автомата?
1. Автоматический огонь или одиночный огонь.
2. Автоматический огонь.
3. Одиночный огонь.
5. На чем основано автоматическое действие автомата?
1. На использовании энергии внешних газов.
2. На использовании энергии пороховых газов.
6. Какая бывает разборка автомата?
1. Может быть неполная и полная.
2. Может быть неполная.
3. Может быть полная.
7. Какова прицельная дальность стрельбы?
1. Прицельная дальность стрельбы - 800 м.
2. Прицельная дальность стрельбы - 900 м.
3. Прицельная дальность стрельбы -1000 м.
8. Какой темп стрельбы?
56

1. Темп стрельбы около 500 выстрелов в минуту.
2. Темп стрельбы около 600 выстрелов в минуту.
3. Темп стрельбы около 700 выстрелов в минуту.
9. Какова боевая скорострельность: при стрельбе очередями?
1. До 80 выстрелов в минуту.
2. До 90 выстрелов в минуту.
3. До 100 выстрелов в минуту.
10. Вес автомата без штык-ножа со снаряженным магазином из легкого сплава……
1. АК-74 — 3, 6 кг, АК-74С — 3,8 кг.
2. АК-74 — 3, 9 кг, АК-74С — 4,1 кг.
3. АК-74 — 4, 1 кг, АК-74С — 3,9 кг.
Отработка алгоритма неполной разборки – сборки автомата Калашникова.
Автомат в собранном виде располагается на столе или на чистой
подстилке, на стороне, с которой располагается рукоятка заряжания.
Участник находится на исходном положении в 2-х шагах от стола. По
готовности участника подается команда: «Внимание! К разборке
приступить!» Судья включает секундомер. Участник делает два шага вперед,
в установленной последовательности разбирает автомат, после этого
участник тут же производит сборку автомата.
Отделить магазин. Удерживая автомат левой рукой за шейку приклада
или цевьё, правой рукой обхватить магазин (рис. 38); нажимая большим
пальцем на защёлку, подать нижнюю часть магазина вперёд и отделить его.
После этого проверить, нет ли патрона в патроннике, для чего
перевести переводчик вниз, поставив его в положение «АВ» или «ОД»;
отвести рукоятку затворной рамы назад, осмотреть патронник, отпустить
рукоятку затворной рамы и спустить курок с боевого взвода.
Вынуть пенал принадлежности из гнезда приклада. Утопить пальцем
правой руки крышку гнезда так, чтобы пенал под действием пружины вышел
из гнезда; раскрыть пенал и вынуть из него протирку, ёршик, отвёртку и
выколотку.
Отделить шомпол. Оттянуть конец шомпола от ствола так, чтобы его
головка вышла из-под упора на основании мушки (рис. 39), и вынуть
шомпол.
Отделить дульный тормоз-компенсатор. Утопить отвёрткой или
пальцем фиксатор дульного тормоза-компенсатора. Свернуть дульный
тормоз-компенсатор с резьбового выступа основания мушки, вращая его
против хода часовой стрелки (рис. 40). В случае чрезмерно тугого вращения
дульного тормоза-компенсатора допускается отворачивать его с помощью
выколотки (шомпола), вставленной в окна дульного тормоза-компенсатора.
Отделить крышку ствольной коробки. Левой рукой обхватить шейку
приклада, большим пальцем этой руки нажать на выступ направляющего
стержня возвратного механизма, правой рукой приподнять вверх заднюю
часть крышки ствольной коробки (рис. 41) и отделить крышку.
Отделить возвратный механизм. Удерживая автомат левой рукой за
57

шейку приклада, правой рукой подать вперёд направляющий стержень
возвратного механизма до выхода его пятки из продольного паза ствольной
коробки; приподнять задний конец направляющего стержня (рис. 42) и
извлечь возвратный механизм из канала затворной рамы.
Отделить затворную раму с затвором. Продолжая удерживать
автомат левой рукой, правой отвести затворную раму назад до отказа,
приподнять её вместе с затвором (рис. 43) и отделить от ствольной коробки.
Отделить затвор от затворной рамы. Взять затворную раму в левую
руку затвором кверху (рис. 44); правой рукой отвести затвор назад,
повернуть его так, чтобы ведущий выступ затвора вышел из фигурного
выреза затворной рамы, и вывести затвор вперёд.
Отделить газовую трубку со ствольной накладкой. Удерживая
автомат левой рукой, пальцами правой руки поднять замыкатель газовой
трубки (рис. 45) и снять газовую трубку с патрубка газовой камеры.
Участник должен обязательно коснуться газовой трубкой поверхности
стола, после чего приступить к сборке оружия.
Порядок сборки автомата после неполной разборки. Присоединить
газовую трубку со ствольной накладкой. Удерживая автомат левой рукой,
правой надвинуть газовую трубку передним концом на патрубок газовой
камеры и плотно прижать задний конец ствольной накладки к стволу;
повернуть замыкатель на себя до входа его фиксатора в выем на колодке
прицела.
Присоединить затвор к затворной раме. Взять затворную раму в
левую руку, а затвор в правую и вставить его цилиндрической частью в канал
рамы; повернуть затвор так, чтобы его ведущий выступ вошёл в фигурный
вырез затворной рамы, и продвинуть затвор вперёд.
Присоединить затворную раму с затвором к ствольной коробке. Взять
затворную раму в правую руку так, чтобы затвор удерживался большим
пальцем в переднем положении. Левой рукой обхватить шейку приклада,
правой ввести газовый поршень в полость колодки прицела и продвинуть
затворную раму вперёд настолько, чтобы отгибы ствольной коробки вошли в
пазы затворной рамы, небольшим усилием прижать её к ствольной коробке и
продвинуть вперёд.
Присоединить возвратный механизм. Правой рукой ввести возвратный
механизм в канал затворной рамы; сжимая возвратную пружину, подать
направляющий стержень вперёд и, опустив несколько книзу, ввести его
пятку в продольный паз ствольной коробки.
Присоединить крышку ствольной коробки. Вставить крышку
ствольной коробки передним концом в полукруглый вырез на колодке
прицела; нажать на задний конец крышки ладонью правой руки вперёд и
книзу так, чтобы выступ направляющего стержня возвратного механизма
вошёл в отверстие крышки ствольной коробки.
Спустить курок с боевого взвода и поставить на
предохранитель. Нажать на спусковой крючок и поднять переводчик вверх
до отказа.
Присоединить дульный тормоз-компенсатор. Навернуть дульный
тормоз-компенсатор на резьбовой выступ основания мушки до упора. Если
паз дульного тормоза-компенсатора не совпал с фиксатором, необходимо
отвернуть дульный тормоз-компенсатор (не более одного оборота) до
совмещения паза с фиксатором.
Присоединить шомпол.
58

Вложить пенал в гнездо приклада. Уложить протирку, ёршик, отвёртку
и выколотку в пенал и закрыть его крышкой, вложить пенал дном в гнездо
приклада (рис. 46) и утопить его так, чтобы гнездо закрылось крышкой.
Присоединить магазин к автомату. Удерживая автомат левой рукой за
шейку приклада или цевьё, правой ввести в окно ствольной коробки зацеп
магазина и повернуть магазин на себя так, чтобы защёлка заскочила за
опорный выступ магазина.
Сборка-разборка АК выполняется без перерыва, учитывается общее
время сборки-разборки.__
43-44 ПОМОЩЬ ПРИ СОСТОЯНИЯХ ВЫЗВАННЫХ НАРУШЕНИЕМ СОЗНАНИЯ
Составление типового алгоритма действий человека оказавшегося рядом с пострадавшим
Последовательность поведения человека оказавшегося рядом с пострадавшим
В приложении к приказу Министерства здравоохранения и социального развития
Российской Федерации от 17 мая 2010 г. N 353н "О первой помощи" определён перечень
мероприятий по оказанию первой помощи:
Оценить окружающую обстановку (с определением угрозы для собственной жизни,
угрозы для пострадавших и окружающих, с оценкой количества пострадавших).
2. Вызвать бригаду скорой медицинской помощи, другие специальные службы,
сотрудники которых обязаны оказывать первую помощь по закону или специальному
правилу.
3. Определить признаки жизни (с определением наличия сознания, дыхания, пульса на
сонных артериях).
4. Извлечь пострадавшего из очага поражения и его перемещение безопасное место.
5. Восстановить и поддерживать проходимость верхних дыхательных путей.
6. Провести сердечно-легочную реанимацию.
7. Остановить кровотечение и наложить повязки.
8. Провести опрос больного на наличие признаков сердечного приступа.
9. Провести осмотр пострадавшего (больного) в результате несчастных случаев, травм,
отравлений и других состояний и заболеваний, угрожающих их жизни и здоровью.
10. Наложить герметичную повязку при проникающем ранении грудной клетки.
11. Зафиксировать шейный отдел позвоночника.
12. Провести иммобилизацию (фиксацию) конечностей.
13. Местное охлаждение.
45-46 ПЕРВАЯ ПОМОЩЬ ПРИ НЕОТЛОЖНЫХ СОСТОЯНИЯХ: ЗАКОН И
ПОРЯДОК ОКАЗАНИЯ. АЛГОРИТМ ПОМОЩИ ПОСТРАДАВШИМ ПРИ ДТП И
ЧС
Составление алгоритма действий при ДТП: водитель причастного к ДТП, водитель
очевидец ДТП, составление порядка действий на месте ДТП.
Последовательность оказания первой помощи в разных чрезвычайных ситуациях
3.1. Последовательность оказания первой помощи пострадавшему при дорожнотранспортном происшествии
Дорожно-транспортное происшествие (ДТП) является для человека экстремальным
событием. Пострадавший может испытывать чрезмерное воздействие физических и
психологических стрессовых факторов.
Чрезмерность травмирующих факторов ДТП обуславливают:
Внезапность события.
Неготовность к восприятию картины ДТП, смерти.
59

Масштабность последствий.
Значимость потерь: физических, материальных,
моральных и др.
Физическая и психологическая неготовность к преодолению возникших проблем.
Интенсивность и множественность воздействующих факторов.
Тяжесть травмы усиливается психологическими стрессовыми факторами, особенностями
индивидуально-личностного восприятия ситуации, значимостью потерь.
Физические и психические травмы, полученные в ДТП, могут в дальнейшем снизить
работоспособность, вызвать посттравматические расстройства, психосоматические
заболевания.
Помните о правиле «золотого часа» при оказании первой помощи
пострадавшим (время потрачено на каждый период):
до прибытия на место происшествия не более 12 мин.;
помощь на месте происшествия не более 20 мин.;
транспортировка в лечебное учреждение не более 8 мин.;
приём и обследование в клинике не более 20 мин.
Действия водителя причастного к ДТП:
немедленно остановите транспортное средство;
включите аварийную световую сигнализацию;
выставите знак аварийной остановки (мигающий красный фонарь) позади транспортного
средства (от места аварии в населенных пунктах - 15метров, вне населенных пунктах –
30метров);
не перемещайте предметы, имеющие отношение к происшествию;
вызовите бригады скорой медицинской помощи, центра медицины катастроф, службы
спасения;
примите возможные меры для оказания первой помощи пострадавшим.
Действия водителя очевидца ДТП:
остановите транспортное средство на безопасном
расстоянии от места происшествия и по возможности
необходимо съехать на обочину;
включите аварийную сигнализацию;
выставите знак аварийной остановки;
при наличии, свободного от оказания помощи человека, направьте его с предупреждающим
знаком (мигающий фонарь, кусок красной или белой ткани)в сторону противоположную
движению(предупредить водителей об опасности);
безопасно подойдите к месту ДТП.
Последовательность действий на месте ДТП:
убедитесь в отсутствии опасности для себя, пострадавшего и окружающих;
на поврежденном транспортном средстве выключите двигатель, зажигание, при
необходимости «массу»;
установите стояночный тормоз, блокируйте колеса, выставьте знак аварийной остановки;
при необходимости стабилизируйте поврежденное транспортное средство;
выявите пострадавших с сильным наружным кровотечением и остановите его;
определите количество пострадавших, наличие детей, беременных, зажатие в транспортном
средстве пострадавших;
правильно вызовите соответствующие службы;
осмотрите пострадавших;
оцените состояние пострадавших - сознание, дыхание, пульс;
окажите первую помощь (привлекайте к этому окружающих);
придайте пострадавшим необходимое функциональное положение;
не извлекайте пострадавших из транспортных средств если нет угрозы для их жизни;
60

наблюдайте за пострадавшими до прибытия помощи;
поддерживайте с пострадавшими постоянный словесный и визуальный контакт;
по возможности защитите пострадавших от неблагоприятного воздействия окружающей
среды;
соберите все автомобильные аптечки и огнетушители в одном, доступном месте.
Как найти помощников.
Выясните есть ли:
медицинские работники, спасатели или люди проходившие курсы по оказанию первой
помощи;
очевидцы, проявляющие активность и желание помочь.
Помощник должен быть:
спокоен и уверен в своих действиях;
с хорошими физическими данными;
совершать рациональные действия;
в удобной одежде и обуви;
примерный возраст от 20 до 45 лет;
выбранный вами человек согласен быть вашим помощником.
Задачи для помощников:
оказание первой помощи;
извлечение пострадавших из транспортных средств;
организация безопасности на месте ДТП;
дополнительное ограничение места ДТП (выставить дополнительные знаки аварийной
остановки из табельных или подручных средств);
оставить свободными места для прибытия бригад спасателей и встреча данных бригад;
поиск пострадавших поблизости от места ДТП;
предупреждение об опасности для других водителей;
уход за пострадавшими;
визуальный и словесный контроль за пострадавшими.
Извлечение пострадавшего спасательским приемом со стабилизацией шеи (метод
Раутека).
Для этого руки участника оказания первой помощи проводятся под мышками
пострадавшего, фиксируют его предплечье, при этом одна из рук участника оказания
первой помощи фиксирует за нижнюю челюсть голову пострадавшего.

Порядок общения с пострадавшим:
используйте те приемы, которые вы лучше знаете;
ваше поведение должно быть уверенным и спокойным;
будьте готовы к любым ситуациям;
желательно подойти к пострадавшему спереди;
представьтесь пострадавшему;
общайтесь с пострадавшим по имени и на «Вы»;
объясняйте ваши действия и цель помощи;
манипуляции должны быть бережными и осторожными;
61

превратите ваше сочувствие в фактическую помощь;
постоянно беседуйте с пострадавшим;
дайте конкретные задания людям, которые вам мешают;
не отвечайте на возможную агрессию и оскорбления со стороны пострадавшего, это
относится не к вам, это способ пострадавшего избавиться от собственного страха;
предотвращайте споры между вашими помощниками;
старайтесь делать все как можно лучше!
Выполнить обязательно –укаждого пострадавшего должен фиксироваться шейный
отдел позвоночника подручными средствами.

Опасность для жизни пострадавших находящихся в автомобиле:
задымление, воспламенение автомобиля;
воздействие агрессивных жидкостей, газов;
характер травм и тяжесть состояния не позволяют оказывать помощь внутри автомобиля.
Правила оказания первой помощи:
1. Единоначалие и дисциплина – если вам не удалось в первые минуты происшествия найти
медицинского работника, способного взять на себя оказание помощи пострадавшим в ДТП,
возьмите на себя руководство и оказание первой помощи пострадавшим до прибытия
профессиональной помощи: СМП, ГИБДД, спасателей. Свидетелей ДТП привлекайте к
оказанию помощи в качестве помощников. Просьбы и распоряжения помощникам давайте
четкие и конкретные.
2. Собственная безопасность и безопасность пострадавшего превыше всего – установите
свой автомобиль в безопасном месте и на безопасном расстоянии, выставите знак
«Аварийная остановка», включите аварийную сигнализацию, выключите зажигание
поврежденного автомобиля и блокируйте колеса, используйте огнетушитель при
задымлении транспортного средства, оказывайте помощь пострадавшему только в
медицинских перчатках и т. д.
3. Незамедлительно вызовите скорую медицинскую помощь, используйте «спасательные
вопросы» — Где? Что? Когда? Кто? Как? На вопросы диспетчера СМП отвечайте четко и
спокойно. Излишняя эмоциональность только затруднит понимание диспетчером СМП
ваших ответов.
4. Определите приоритеты оказания помощи – кому из пострадавших и какую помощь
нужно оказать в первую очередь. Помощь начинайте оказывать с жизнеспасающих
62

мероприятий. Первоначально выполните самые необходимые мероприятия. Обратите
внимание на детей, без видимых повреждений, безразлично взирающих на происходящее.
5.Обращайтесь с пострадавшими бережно.
6.Первую помощь оказывайте безотлагательно и правильно.
7.Постоянно поддерживайте с пострадавшим вербальный и невербальный контакт.
8.Не покидайте место ДТП до прибытия профессиональной помощи. При необходимость
экстренной эвакуации пострадавшего по жизненным показаниям, организуйте наблюдение
за оставшимися на мете ДТП пострадавшими.
9. Передавайте пострадавших бригаде СМП, спасателям «из рук в руки».
10. Действуйте смело, но без суеты, в пределах допустимого риска.
47-48 АЛГОРИТМ ПОМОЩИ ПРИ КРОВОТЕЧЕНИЯХ И РАНЕНИЯХ
Тестирование «Первая помощь» (теория)
1. Струей из раны вытекает тёмная по цвету кровь. Такое кровотечение называется:
А) капиллярное
Б) венозное
В) артериальное
2. Рана, нанесенная острым предметом. Входное отверстие небольших размеров, высок
риск инфицирования:
А) резаная
Б) рваная
В) колотая
3. Максимально допустимое время прикладывания льда при ушибах и растяжениях:
А) 40-50 минут через каждые 1-2 часа
Б) 20-30 минут через каждые 2-3 часа
В) 50-60 минут через каждые 2-3 часа
4. Принципы первой помощи ПХСП - это:
А) покой, холод, секвестрация, приподнятое положение
Б) покой, холод, сдавливающая повязка, приподнятое положение
В) покой, холод, сенсибилизация, приподнятое положение
5.Типы открытых ран:
А) резаная рана, авульсия, ампутация, закрытый перелом
Б) резаная рана, ампутация, рваная рана, разрыв связок
В) ссадина, рваная рана, резаная рана, колотая рана, авульсия, ампутация
6. Какие действия нельзя производить при обработке раны:
А) закрывать стерильной повязкой
Б) останавливать кровотечение
В) очищать опасные для жизни раны
7. Какие действия нужно предпринять при колотом ранении:
А) Не удалять из раны инородное тело. Остановить кровотечение.
Б) Удалить из раны инородное тело и остановить кровотечение.
В) Обработать рану, удалить из раны инородное тело. Остановить кровотечение.
8.Перечислите типы переломов:
_____________________________________________
_____________________________________________
_____________________________________________
9. Абсолютными показаниями для обращения в медицинское учреждение являются:
А) Небольшой порез
Б) Сонливость
В) Непрекращающееся кровотечение
63

10. Ваши действия при переломе конечности в походе одного из ребят:
А) аккуратно уложить пострадавшего, приложить холод на болевое место, наложить
тугую повязку
Б) не меняя положения конечности наложить шину, приложить холод на поврежденное
место, травмированную конечность поднять, аккуратно транспортировать на самодельных
носилках.
В) постараться аккуратно вправить кость, наложить шину, приложить холод на
поврежденное место, под колено подложить валик из подручных средств, аккуратно
транспортировать на самодельных носилках.
11.Перечислите ваши действия при переохлаждении:
_____________________________________________________________________________
_____________________________________________________________________________
_____________________________________________________________________________
_____________________________________________________________________________
________________________________________
12.Ваши действия при вывихе сустава:
А) Аккуратно вправьте сустав, приложите холод, наложите тугую повязку
Б) Оцените КЧД (кровообращение, чувствительность, движение), иммобилизуйте сустав с
помощью шины, как при переломе, обратитесь за медицинской помощью.
В) Приложите тепло, чтобы успокоить боль, наложите тугую повязку, обратитесь за
медицинской помощью.
13. Перечислите типы открытых ран:
_____________________________________________________________________________
_____________________________________________________________________________
_____________________________________________________________________________
______________________________
14. Типы внешнего кровотечения:
_____________________________________________________________________________
_____________________________________________________________________________
_____________________________________________________________________________
______________________________
15. При травме лодыжки (стопы) накладывается фиксирующая повязка:
А) Спиралевидная
Б) Бактерицидный пластырь
В) Восьмерка
16. В поврежденном участке тела чувствуется холод, покалывание, жжение или боль,
кожа бледного, воскового или серо-желтого цвета. О каком повреждении идет речь:
А) Переохлаждение
Б) Поверхностное отморожение
В) Солнечный ожег
Ответы:

1
2
3

Б
В
Б

4
5
6
7

Б
В
В
А
64

8
9
10
11




Открытый
Закрытый
В
Б



Прекратить потери тепла. Перенести в помещение.
Укутать.
Бережное отношение
Уложить
Теплое питье
Вызвать СМП





12
13

14

Б










15
16

Ссадина,
Рваная рана,
Резаная рана,
Колотая рана,
Авульсия
Ампутация
Артериальное,
Венозное
Капиллярное
В
Б

49-50 ОКАЗАНИЕ ПОМОЩИ ПОДРУЧНЫМИ СРЕДСТВАМИ В ПРИРОДНЫХ
УСЛОВИЯХ
Тестирование
«Первая медицинская помощь при травмах и повреждениях».
1.
Признаками ушиба являются:
а)
боль, усиливающаяся при движении;
б)
резкое повышение температуры;
в)
припухлость тканей (отек);
г)
кровоподтек, нарушение функций нижних или верхних конечностей.
Определите, какие из приведенных признаков являются признаками ушиба?
2.
Определите последовательность оказания первой медицинской помощи при ушибах:
а)
на место ушиба наложить тугую повязку;
б)
обеспечить покой пострадавшему;
в)
на место ушиба наложить холод;
г)
доставить пострадавшего в медицинское учреждение.
3.
Признаки растяжения:
а)
боль при малейшем движении;
б)
головокружение;
в)
ограничение подвижности;
г)
припухлость (отек), которая быстро увеличивается в размере;
д)
незначительный кровоподтек, превращающийся потом в синяк.
В приведенных примерах допущена ошибка, найдите ее.
4.
Определите последовательность оказания первой медицинской помощи при растяжении:
а)
обеспечить покой поврежденной конечности;
65

б)
придать поврежденной конечности возвышенное положение;
в)
наложить холод на поврежденное место;
г)
наложить тугую повязку на поврежденное место;
д)
доставить пострадавшего в медицинское учреждение.
5.
Признаками разрыва связок являются:
а)
резкая боль;
б)
тошнота и головокружение;
в)
невозможность согнуть или разогнуть руку или ногу;
г)
поврежденное место быстро увеличивается в размере (опухает).
Найдите допущенную ошибку.
6.
Признаками разрыва мышц являются:
а)
резкая внезапная боль в месте разрыва;
б)
появление на коже видимого на глаз западения (вмятинки), ниже которого отмечается
выпячивание;
в)
определяется припухлость, изменяется цвет кожи, она становится синей;
г)
невозможно пошевелить поврежденной рукой или ногой;
д)
кратковременная потеря зрения.
Найдите допущенную ошибку.
7.
Определите последовательность оказания первой помощи при разрывах связок и мышц:
а)
придать поврежденной конечности возвышенное положение;
б)
на поврежденное место наложить холод;
в)
дать пострадавшему обезболивающее средство;
г)
наложить тугую повязку и обеспечить покой пострадавшему;
д)
доставить пострадавшего в медицинское учреждение.
8.
Смещение костей относительно друг друга в области сустава это:
а)
перелом;
б)
сдавление;
в)
вывих.
9.
Признаками вывиха являются:
а)
высокая температура;
б)
нервное возбуждение;
в)
изменение формы сустава;
г)
боль в суставе;
д)
вынужденное (необычное) положение конечности;
е)
изменение длины конечности;
ж)
невозможность движения в суставе.
Найдите допущенные ошибки.
10.
Определите последовательность оказания первой медицинской помощи при вывихах:
а)
при повреждении конечности сделать тугую повязку;
б)
дать пострадавшему обезболивающее;
в)
обеспечить покой поврежденной конечности;
г)
доставить пострадавшего в медицинское учреждение.
11.
В области повреждения появляется очень сильная боль, пострадавший говорит, не
умолкая, делает попытки сдвинуться с места, где он находится, это признаки:
а)
вывиха;
б)
перелома;
в)
разрыва мышц;
г)
сдавления.
12.
Определите последовательность первой медицинской помощи при сдавлении:
а)
обложить поврежденное место холодом;
б)
на поврежденную конечность наложить тугую повязку;
в)
извлечь пострадавшего из завала;
г)
доставить пострадавшего в медицинское учреждение.
13.
Вы с друзьями спускались с горы. Неожиданно один из мальчиков упал и ударился рукой о
твердый предмет. На месте ушиба кожа посинела, появилась шишка (припухлость) и боль в руке.
66

Вам необходимо оказать другу помощь. Выберите из предлагаемых вариантов ваши дальнейшие
действия и определите их очередность:
а)
положить на место ушиба грелку для согревания ушиба;
б)
сделать давящую повязку;
в)
на место повреждения положить холод;
г)
смазать ушиб йодом;
д)
обеспечить поврежденной руке покой.
14.
Полное или частичное нарушение целостности костей в результате удара, сжатия,
сдавления, изгиба или другого воздействия, это:
а)
вывих;
б)
перелом;
в)
ушиб;
г)
сдавление.
Ответы к тестам: 1 (а; в; г); 2 (в; а; б; г); 3 (б); 4 (в; г; а; б; д); 5 (б); 6 (д); 7 (б; г; в; а; д); 8 (в);
9 (а; б); 10 (б; в; г); 11 (г); 12 (в; б; а; г); 13 (в; б; д); 14 (б).

51-52 ПОМОЩЬ ПРИ ВОЗДЕЙСТВИИ ТЕМПЕРАТУР НА ОРГАНИЗМ
ЧЕЛОВЕКА. СПОСОБЫ САМОСПАСЕНИЯ.
Фронтальный опрос:
1. Что такое ожог? Как различают термические ожоги по степени тяжести?
2. В чем заключается первая помощь при термических ожогах различной степени
тяжести?
3. Как оказывается первая помощь при ожогах химическими веществами?
4. Как оказывается первая помощь при электрических ожогах?
5. От чего зависит характер лучевых поражений? Какие выделяют периоды лучевых
ожогов?
6.Как различают лучевые ожоги по степени тяжести?
7.Какую медицинскую помощь оказывают при поверхностных (тяжелых) лучевых
ожогах?
8.К чему приводит длительное воздействие холода
9.Что способствует возникновению отморожений?
10.Каковы признаки отморожения I степени?
11.Каково состояние пострадавшего при отморожении II степени?
12.В чем заключается тяжесть состояния пострадавшего при отморожении III степени?
13.К каким последствиям ведут отморожения IV степени?
14.В чем заключается первая помощь при отморожениях?
15.Почему нельзя растирать отмороженные участки
16.Какие основные способы прекращения воздействия электрического тока на
пострадавшего вы знаете?
17.Какой силы ток опасен для жизни человека?
18.Какие последствия может вызвать прикосновение человека к токонесущим деталям?
19.Какие факторы оказывают влияние на степень поражения человека электрическим
током?
20.Какие меры первой помощи применяются после освобождения пострадавшего от
действия тока
53-54 ПРАКТИЧЕСКАЯ РАБОТА № 6 ОКАЗАНИЕ ПЕРВОЙ ПОМОЩИ
Тест "Оказание первой помощи"
1.Признаки артериального кровотечения
Выберите один или несколько ответов:
67

1. очень темный цвет крови
2. алая кровь из раны вытекает фонтанирующей струей
3. большое кровавое пятно на одежде или лужа крови возле пострадавшего
4. над раной образуется валик из вытекающей крови
5. кровь пассивно стекает из раны
2.Каким образом проводится сердечно-легочная реанимация пострадавшего?
Выберите один ответ:
1. Давление руками на грудину пострадавшего и искусственная вентиляция легких:
вначале 30 надавливаний на грудину, затем 2 вдоха методом «Рот ко рту»
2. Искусственная вентиляция легких и давление руками на грудину пострадавшего:
вначале 1 вдох методом «Рот ко рту», затем 15 надавливаний на грудину
3. Давление руками на грудину пострадавшего и искусственная вентиляция легких:
вначале 5 надавливаний на грудину, затем 1 вдох методом «Рот ко рту»
3.Вторым действием (вторым этапом) при оказании первой помощи является:
Выберите один ответ:
1. Предотвращение возможных осложнений
2. Устранение состояния, угрожающего жизни и здоровью пострадавшего
3. Правильная транспортировка пострадавшего
4.Признаки венозного кровотечения
Выберите один или несколько ответов:
1. кровь пассивно стекает из раны
2. над раной образуется валик из вытекающей крови
3. очень темный цвет крови
4. алая кровь из раны вытекает фонтанирующей струей
5.По каким признакам судят о наличии внутреннего кровотечения?
Выберите один ответ:
1. Цвет кожных покровов, уровень артериального давления, сознание
2. Пульс, высокая температура, судороги.
3. Резкая боль, появление припухлости, потеря сознания
6.Кто может оказывать первую помощь пострадавшему ребенку?
Выберите один ответ:
1. только медицинский работник
2. любой человек, который оказался рядом с пострадавшим ребенком
3. любой человек, который оказался рядом с пострадавшим ребенком, при наличии
специальной подготовки и (или) навыков
7.Разрешено ли давать пострадавшему лекарственные средства при оказании ему
первой помощи?
Выберите один ответ:
1. Разрешено
2. Запрещено
3. Разрешено в случае крайней необходимости
8.Куда накладывается кровоостанавливающий жгут на конечность при
кровотечении?
Выберите один ответ:
1. Непосредственно на рану.
2. Ниже раны на 4-6 см.
3. Выше раны на 4-6 см.
9.При открытом переломе конечностей, сопровождающимся артериальным
кровотечением, оказание первой помощи начинается:
Выберите один ответ:
1. С наложения импровизированной шины
68

2. С наложения жгута выше раны на месте перелома
3. С наложения давящей повязки
10.Как оказывается первая помощь при переломах конечностей, если отсутствуют
подручные средства для их изготовления?
Выберите один ответ:
1. Верхнюю конечность, согнутую в локте, подвешивают на косынке и прибинтовывают к
туловищу. Нижние конечности плотно прижимают друг к другу и прибинтовывают.
2. Верхнюю конечность, вытянутую вдоль тела, прибинтовывают к туловищу. Нижние
конечности прибинтовывают друг к другу, проложив между ними мягкую ткань.
3. Верхнюю конечность, согнутую в локте, подвешивают на косынке и
прибинтовывают к туловищу. Нижние конечности прибинтовывают друг к другу,
обязательно проложив между ними мягкую ткань.
11.Какие из перечисленных мероприятий относятся к оказанию первой помощи?
Выберите один или несколько ответов:
1. восстановление и поддержание проходимости дыхательных путей
2. применение лекарственных препаратов
3. выявление признаков травм, отравлений и других состояний, угрожающих жизни
и здоровью
4. передача пострадавшего бригаде скорой медицинской помощи
5. сердечно-легочная реанимация
6. определение признаков жизни у пострадавшего ребенка
7. придание оптимального положения телу
8. контроль состояния и оказание психологической поддержки
9. временная остановка наружного кровотечения
10. оценка обстановки и создание безопасных условий для оказания первой помощи
11. вызов скорой медицинской помощи
12.В каком порядке проводятся мероприятия первой помощи при ранении?
Выберите один ответ:
1. Остановка кровотечения, наложение повязки
2. Обеззараживание раны, наложение повязки, остановка кровотечения
3. Остановка кровотечения, обеззараживание раны, наложение повязки
13.О каких травмах у пострадавшего может свидетельствовать поза «лягушки»
(ноги согнуты в коленях и разведены, а стопы развернуты подошвами друг к другу)
и какую первую помощь необходимо при этом оказать?
Выберите один ответ:
1. У пострадавшего могут быть переломы костей голени и нижней трети бедра. При
первой помощи наложить шины только на травмированную ногу от голеностопного до
коленного сустава, не вытягивая ногу.
2. У пострадавшего могут быть ушиб брюшной стенки, перелом лодыжки, перелом костей
стопы. При первой помощи вытянуть ноги, наложить шины на обе ноги от голеностопного
сустава до подмышки.
3. У пострадавшего могут быть переломы шейки бедра, костей таза, перелом
позвоночника, повреждение внутренних органов малого таза, внутреннее
кровотечение. Позу ему не менять, ноги не вытягивать, шины не накладывать. При
первой помощи подложить под колени валик из мягкой ткани, к животу по
возможности приложить холод.
14.В какой последовательности следует осматривать ребенка при его
травмировании?
Выберите один ответ:
1. конечности, область таза и живот, грудная клетка, шея, голова
2. голова, шея, грудная клетка, живот и область таза, конечности
69

3. грудная клетка, живот и область таза, голова, шея, конечности
15.В чем заключается первая помощь пострадавшему, находящемуся в сознании,
при повреждении позвоночника?
Выберите один ответ:
1. Пострадавшему, лежащему на спине, подложить под шею валик из одежды и
приподнять ноги
2. Лежащего пострадавшего не перемещать. Следует наложить ему на шею
импровизированную шейную шину, не изменяя положения шеи и тела
3. Уложить пострадавшего на бок
16.Когда должен применяться непрямой массаж сердца?
Выберите один ответ:
1. при кровотечении
2. при применении искусственного дыхания
3. после освобождения пострадавшего от опасного фактора
4. при повышении артериального давления
5. при отсутствии пульса
17.Что делать, если ребенок получил ожог пламенем, кипятком или паром?
Выберите один или несколько ответов:
1. вызвать скорую медицинскую помощь, до ее приезда наблюдать за ребенком и
одновременно с этим охлаждать место ожога холодной проточной водой не менее 20
минут
2. вызвать скорую медицинскую помощь, до ее приезда наблюдать за ребенком и
одновременно с этим приложить холодный предмет к месту ожога, предварительно
обернув его куском ткани
3. вызвать скорую медицинскую помощь, до ее приезда наблюдать за ребенком
18.Как проверить наличие дыхания у ребенка при внезапной потере сознания?
Выберите один ответ:
1. в течение 10 секунд внимательно смотреть на его грудную клетку
2. наклониться к ребенку, приложить ухо к его грудной клетке и в течение 10 секунд
прислушиваться
3. запрокинуть голову ребенка, поднять подбородок, в течение 10 секунд
прислушиваться, пытаться ощутить дыхание ребенка на своей щеке, увидеть
дыхательные движения его грудной клетки
19.Как следует уложить пострадавшего при потере им сознания и наличии пульса на
сонной артерии для оказания первой помощи?
Выберите один ответ:
1. На спину с вытянутыми ногами
2. Чтобы пострадавший не мог погибнуть от удушения в результате западания языка, его
следует положить на живот, чтобы вызвать рвотный рефлекс
3. Чтобы пострадавший не мог погибнуть от удушения в результате западания
языка, его следует положить на бок так, чтобы согнутые колени опирались о землю,
а верхняя рука находилась под щекой
4. На спину с подложенным под голову валиком
20.Признаки обморока
Выберите один или несколько ответов:
1. потере сознания предшествуют резкая слабость, головокружение, звон в ушах и
потемнение в глазах
2. кратковременная потеря сознания (не более 3-4 мин)
3. потеря чувствительности
4. потеря сознания более 6 мин
21.Как проверить признаки сознания у ребенка?
70

Выберите один ответ:
1. аккуратно потормошить за плечи и громко спросить «Что случилось?»
2. поводить перед ребенком каким-нибудь предметом и понаблюдать за движением его
глаз
3. спросить у ребенка, как его зовут
22.В каком объеме проводятся мероприятия при прекращении сердечной
деятельности и дыхания у пострадавшего?
Выберите один ответ:
1. Освобождение дыхательных путей, проведение ИВЛ (искусственной вентиляции
легких) и НМС (непрямого массажа сердца).
2. Освобождение дыхательных путей, проведение ИВЛ (искусственной вентиляции
легких)
3. Проведение НМС (непрямого массажа сердца)
23.При попадании в глаза щелочного раствора:
Выберите один ответ:
1. создать пострадавшему покой
2. необходимо промыть глаза мыльным раствором
3. необходимо промыть глаза проточной водой в большом количестве
24.Как следует расположить руки на грудной клетке пострадавшего при давлении
руками на его грудину (выполнении непрямого массажа сердца)?
Выберите один ответ:
1. Основания ладоней обеих рук должны располагаться на грудной клетке на два пальца
выше мечевидного отростка так, чтобы большой палец одной руки указывал в сторону
левого плеча пострадавшего, а другой – в сторону правого плеча
2. Основания ладоней обеих рук, которые накладываются одна на другую, должны
располагаться на грудной клетке на два пальца выше мечевидного отростка так,
чтобы большой палец одной руки указывал в сторону подбородка пострадавшего, а
другой – в сторону живота
3. Давление руками на грудину выполняют основанием ладони только одной руки,
расположенной на грудной клетке на два пальца выше мечевидного отростка.
Направление большого пальца не имеет значения.
25.Перелом это
Выберите один ответ:
1. трещины, сколы, раздробление костей
2. разрушение мягких тканей костей
3. трещины, сколы, переломы ороговевших частей тела
26.Действия по помощи пострадавшему при попадании инородного тела в
дыхательные пути:
Выберите один ответ:
1. Положить пострадавшего на бок и вызвать интенсивную рвоту.
2. Нагнуть туловище пострадавшего вперед, нанести несколько интенсивных ударов
ладонью между лопаток, при отсутствии эффекта — обхватить пострадавшего сзади,
надавить 4-5 раз на верхнюю часть живота.
3. Нанести пострадавшему, стоящему прямо, несколько интенсивных ударов ладонью
между лопаток.
27.Первая медицинская помощь при вывихе конечности?
Выберите один ответ:
1. Зафиксировать конечность, не вправляя вывих, приложить пузырь (грелку) с горячей
водой, организовать транспортировку в больницу или травмпункт

71

2. Осуществить иммобилизацию конечности, дать доступные обезболивающие
средства, приложить к поврежденному суставу пузырь с холодной водой или льдом,
организовать транспортировку в больницу или травмпункт
3. Дать обезболивающее средство, вправить вывих и зафиксировать конечность
28.Каковы признаки кровотечения из крупной артерии и первая помощь при ее
ранении?
Выберите один ответ:
1. Одежда пропитывается кровью только в месте ранения (цвет крови не имеет значения),
кровь вытекает из раны пассивно. Накладывается кровоостанавливающий жгут ниже
места ранения не менее чем на 3-5 см.
2. Одежда быстро пропитывается кровью, кровь темного цвета вытекает из раны
пассивно. Накладывается давящая повязка на место ранения.
3. Одежда пропитана кровью, кровь алого цвета вытекает из раны пульсирующей
струей. Накладывается кровоостанавливающий жгут выше места ранения не менее
чем на 3-5 см.
29.Основные правила оказания первой помощи при травматическом шоке:
Выберите один ответ:
1. Проведение мероприятий по прекращению действия травмирующих факторов.
Восстановление нарушенного дыхания и сердечной деятельности, временная
остановка кровотечения, борьба с болью, закрытие ран стерильными (чистыми)
повязками, придание пострадавшему наиболее удобного положения, обеспечить
приток свежего воздуха, организовать вызов к месту происшествия скорой
медицинской помощи.
2. Проведение мероприятий по прекращению действия травмирующих факторов. Снять
одежду или ослабить ее давление. Дать понюхать нашатырный спирт. Наложить на лоб
холодный компресс. Обеспечить приток свежего воздуха. Организовать вызов к месту
происшествия скорой медицинской помощи.
3. Уложить пострадавшего на спину. Дать понюхать нашатырный спирт. Наложить
теплые примочки на лоб и затылок.
30.При переломах костей конечностей накладывается шина:
Выберите один ответ:
1. ниже области перелома
2. выше и ниже области перелома, так чтобы шина захватывала не менее двух
ближайших суставов
3. выше области перелома
31.Какие предпринять меры при подозрении на отравление ребенка?
Выберите один или несколько ответов:
1. удалить поступивший яд (например, вызвать рвоту)
2. вызвать скорую медицинскую помощь, до ее приезда наблюдать за ребенком,
оказывая помощь при необходимости (например, при исчезновении признаков
жизни приступить к сердечно-легочной реанимации)
3. прекратить поступление ядовитого вещества в организм ребенка (вынести его из
загазованной зоны, удалить жало насекомого, стереть ядовитое вещество с
поверхности кожи и т. д.)
32.Когда следует начинать сердечно-легочную реанимацию пострадавшего?
Выберите один ответ:
1. При наличии болей в области сердца и затрудненного дыхания
2. При потере пострадавшим сознания, независимо от наличия пульса на сонной артерии,
и признаков дыхания
3. При потере пострадавшим сознания и отсутствии пульса на сонной артерии, а
также признаков дыхания
72

33.Как обеспечить восстановление и поддержание проходимости дыхательных путей
пострадавшего при подготовке к проведению сердечно-легочной реанимации?
Выберите один ответ:
1. Уложить пострадавшего на спину и, не запрокидывая ему голову, сжать щеки, чтобы
раздвинуть губы и раскрыть рот. Очистить ротовую полость от слизи и рвотных масс.
2. Уложить пострадавшего на бок, наклонить его голову к груди. Очистить ротовую
полость от слизи и рвотных масс.
3. Очистить ротовую полость от слизи и рвотных масс. Уложить пострадавшего на
спину, запрокинуть ему голову, поднять подбородок и выдвинуть нижнюю челюсть.
34.Как определить наличие пульса на сонной артерии пострадавшего?
Выберите один ответ:
1. Большой палец руки располагают на шее под подбородком с одной стороны гортани, а
остальные пальцы – с другой стороны
2. Три пальца руки располагают с правой или левой стороны шеи на уровне
щитовидного хряща гортани (кадыка) и осторожно продвигают вглубь шеи между
щитовидным хрящом и ближайшей к хрящу мышцей
3. Три пальца руки располагают с левой стороны шеи под нижней челюстью
35.Какова первая помощь при черепно-мозговой травме, сопровождающейся
ранением волосистой части головы?
Выберите один ответ:
1. Наложить импровизированную шейную шину, на рану наложить стерильный ватный
тампон, пострадавшего уложить на спину, приподняв ноги. По возможности к голове
приложить холод.
2. Наложить импровизированную шейную шину. К ране волосистой части головы
приложить давящую повязку из стерильного бинта, пострадавшего уложить на бок с
согнутыми в коленях ногами, по возможности к голове приложить холод.
3. Шейную шину не накладывать, рану заклеить медицинским пластырем, пострадавшего
уложить на бок только в случае потери им сознания
36.При каких состояниях ребенка педагог может оказать ему первую помощь?
Выберите один или несколько ответов:
1. ожоги
2. травмы различных областей тела
3. инородные тела верхних дыхательных путей
4. отморожения
5. высокая температура
6. отсутствие сознания
7. отравление
8. наружные кровотечения
9. боли в животе
10. остановка дыхания и кровообращения
11. боли в груди
37.Какова первая помощь при наличии признаков термического ожога второй
степени (покраснение и отек кожи, образование на месте ожога пузырей,
наполненных жидкостью, сильная боль)?
Выберите один ответ:
1. Полить ожоговую поверхность холодной водой, накрыть стерильной салфеткой и туго
забинтовать
2. Вскрыть пузыри, очистить ожоговую поверхность от остатков одежды, накрыть
стерильной салфеткой (не бинтовать), по возможности приложить холод, поить
пострадавшего водой
73

3. Пузыри не вскрывать, остатки одежды с обожженной поверхности не удалять,
рану накрыть стерильной салфеткой (не бинтовать), по возможности приложить
холод и поить пострадавшего водой
38.При черепно-мозговой травме:
Выберите один ответ:
1. необходимо положить на голову тепло
2. необходимо положить на голову холод
3. необходимо наложить на голову марлевую повязку
39.Что нужно делать при сильном кровотечении у ребенка в результате травмы до
прибытия бригады скорой медицинской помощи?
Выберите один или несколько ответов:
1. если давящая повязка не помогает и кровотечение не останавливается, прижать
артерию пальцем, наложить кровоостанавливающий жгут
2. промыть рану, обработать ее, затем наложить салфетку, туго забинтовать
3. наложить на рану салфетку, прижать, туго забинтовать
4. обеспечить безопасные условия для оказания первой помощи
40.Внезапно возникающая потеря сознания — это:
Выберите один ответ:
1. Шок
2. Мигрень
3. Обморок
41.При артериальном кровотечении наложенный жгут нельзя держать более:
Выберите один ответ:
1. 1 ч
2. 30 мин
3. 45 мин
42.На какой срок может быть наложен кровоостанавливающий жгут?
Выберите один ответ:
1. Не более получаса в теплое время года и не более одного часа в холодное время года
2. Не более одного часа в теплое время года и не более получаса в холодное время
года
3. Время наложения жгута не ограничено
43.Что делать, если ребенок подавился и не может дышать, говорить и кашлять?
Выберите один или несколько ответов:
1. если удары в спину и толчки в верхнюю часть живота не помогли, уложить
ребенка на пол, проверить признаки дыхания, при их отсутствии – вызвать скорую
медицинскую помощь и приступить к сердечно-легочной реанимации
2. выполнить толчки в верхнюю часть живота (детям до года – в грудь) до 5
попыток, если традиционные удары по спине не помогли
3. ударить по спине между лопатками (до 5 попыток), наклонив ребенка вперед
44.Признаки переохлаждения
Выберите один или несколько ответов:
1. нет пульса у лодыжек
2. посинение или побледнение губ
3. озноб и дрожь
4. нарушение сознания: заторможенность и аппатия, бред и галлюцинации,
неадекватное поведение
5. снижение температуры тела
6. потеря чувствительности
45.Третьим действием (третьим этапом) при оказании первой помощи является:
Выберите один ответ:
74

1. Предотвращение возможных осложнений
2. Прекращение воздействия травмирующего фактора
3. Правильная транспортировка пострадавшего
46.К ушибленному месту необходимо приложить:
Выберите один ответ:
1. Грелку
2. Холод
3. Спиртовой компресс
47.При проведении ИВЛ (искусственной вентиляции легких) методом «рот в рот»
необходимо:
Выберите один ответ:
1. Зажимать нос пострадавшего только в случае, если носовые ходы свободны
2. Нос пострадавшему не зажимать
3. Свободной рукой плотно зажимать нос пострадавшего
48.При ушибах и растяжениях на поврежденное место накладывается:
Выберите один ответ:
1. тепло
2. свободная повязка
3. холод
49.Первым действием (первым этапом) при оказании первой помощи является:
Выберите один ответ:
1. Прекращение воздействия травмирующего фактора
2. Правильная транспортировка пострадавшего
3. Предотвращение возможных осложнений
50.Как остановить кровотечение при ранении вены и некрупных артерий?
Выберите один ответ:
1. Наложить жгут ниже места ранения
2. Наложить давящую повязку на место ранения
3. Наложить жгут выше места ранения
55 ВЫЯВЛЕНИЕ И ОПИСАНИЕ ОПАСНОСТЕЙ РАБОЧЕГО МЕСТА
РАБОТНИКА ТОРГОВЛИ
Тестирование Техническое оснащение торговых организаций и охрана труда

1. Тестовый вопрос 1: Установите соответствие между классификационными признаками
торговой мебели и их видами:
1. По функциональному назначению: а) для торговых залов магазинов (для продажи
товаров)
2. По месту использования:
б) для хранения (стеллажи, горки, шкафы)
3. По способу установки:

в) пристенная
г) универсальная

2. Тестовый вопрос 2: По способу установки торговая мебель бывает:
а) пристенная;
б) островная, витринная;
в) навесная, встроенная
г) все виды.
75

3. Тестовый вопрос 3: Укажите требования, предъявляемые к немеханическому
оборудованию, которые предусматривают оптимальные размеры торговой мебели с учётом
пропорций тела человека:
а) эксплуатационно-технические;
б) эргономические;
в) санитарно-гигиенические;
г) эстетические.
4. Тестовый вопрос 4: Типизация – это:
а) устранение многообразия и отбор наиболее рациональных конструкций мебели
б) приведение к единообразию форм, конструкций и размеров деталей из которых
производится сборка мебели;
в) нормативные требования к размерам, материалам, качеству изготовления мебели;
г) соответствие стандартам.
5. Тестовый вопрос 5: Эксплуатационные требования к торговой мебели предусматривают
следующие характеристики:
а) удобство выбора, надёжность, прочность;
б) удаление загрязнений, поверхность без выступов;
в) унификация деталей, типизация;
г) нормативные требования к размерам, материалам, качеству изготовления
мебели.
6. Тестовый вопрос 6: По конструкции взвешивающего устройства весы подразделяются
на:
а) стационарные, передвижные, настольные;
б) шкальные, гирные, шкально-гирные, циферблатные, оптические;
в) рычажные, электромеханические;
г) автоматические, полуавтоматические, неавтоматические.
7. Тестовый вопрос 7: Измерение массы товара с использованием эффекта
гравитационных сил, действующих на это тело – это … .
8. Тестовый вопрос 8: ….. – это меры массы, применяемые как единицы измерения массы
грузов на весах.
9. Тестовый вопрос 9: … – это измерительный прибор, предназначенный для измерения
массы товаров с использованием эффекта гравитационных сил.
10. Тестовый вопрос 10: По виду указательного устройства весы подразделяются на:
а) стационарные, передвижные, настольные;
б) шкальные, гирные, шкально-гирные, циферблатные, оптические;
в) рычажные, электромеханические;
г) автоматические, полуавтоматические, неавтоматические.
11. Тестовый вопрос 11: Постоянство показаний – это:
а)
метрологическое требование, характеризующее свойство весов при
многократном взвешивании одного и того груза независимо от его место расположения на
грузоприемном устройстве давать одинаковые показатели;
б) метрологическое требование, характеризующее свойство весов показывать массу
с отклонением от действительных значений в пределах допускаемой погрешности;
в) свойство весов при выведении их из состояния равновесия самостоятельно после
нескольких колебаний возвратится в первоначальное положение;
г) свойство весов, характеризующее их способность при взвешивании быстро
приходить в состояние равновесия.
12. Тестовый вопрос 12: По характеру технологических операций фасовочно-упаковочное
оборудование бывает:
а) дозирующее; фасовочное; упаковочное;
76

б) дозирующее; фасовочное; упаковочное; этикетировочное; пакетоформирующее;
комбинированное;
в) этикетировочное; пакетоформирующее; комбинированное;
г) дозирующее; этикетировочное; пакетоформирующее; комбинированное.
13. Тестовый вопрос 13: По виду упаковочного материала фасовочно-упаковочное
оборудование бывает:
а) термоусадочные оболочки, термоусадочные пленки, поливинилхлоридные и
перфорированные на основе полипропилена;
б) полимерные пленки, вискозоармированные оболочки с внутренним и наружным
полиамидным слоем;
в) пищевые самоклеящиеся стреч-пленки из поливинилхлорида; вакуумные пакеты;
подложки из вспененного полистирола; гофрокартон;
г) все вышеперечисленные.
14. Тестовый вопрос 14: Измельчительно-режущее оборудование по режущему
устройству бывает:
а) машины с дисковыми ножами;
б) машины с ленточными пилами;
в) машины со струнами;
г) все вышеперечисленные.
15. Тестовый вопрос 15: В зависимости от функционального назначения подъёмнотранспортное оборудование классифицируют на следующие группы?
а) грузоподъёмное и транспортирующее;
б) грузоподъёмное, транспортирующее и прочее;
в) грузоподъёмное, транспортирующее, погрузочное и штабелирующее;
г) грузоподъёмное, транспортирующее и штабеле обслуживающее.
16. Тестовый вопрос 16:
Какое бывает подъёмно-транспортное оборудование в
зависимости от направления перемещения грузов?
а) оборудование для вертикального (резко наклонного) перемещения;
б) оборудование для вертикального перемещения (резко наклонного) и
горизонтального (слабо наклонного) перемещения;
в) оборудование для вертикального перемещения (резко наклонного) и
горизонтального (слабо наклонного) перемещения в любую точку помещения магазина или
склада;
г) оборудование для вертикального перемещения (резко наклонного),
горизонтального (слабо наклонного) и смешанного перемещения.
17. Тестовый вопрос 17:
В чём конструктивная особенность мостового грейферного
крана?
а) в наличии специального грузозахватного приспособления подъемного механизма
с самозакрывающимися створами;
б) в наличии специального грузозахватного приспособления, позволяющего
поднимать товары на поддонах;
в) в наличии специального приспособления, обеспечивающего удерживание грузов
в подвешенном состоянии;
г) в наличии устройства, позволяющего стеллажировать грузы.
18. Тестовый вопрос 18: Способ получения холода за счет изменения агрегатного
состояния хладагента, кипения его при низких температурах с отводом от охлаждаемого
тела или среды необходимой для этого теплоты парообразования – это:
а) машинное охлаждение;
77

б) охлаждение при помощи снега или льда;
в) охлаждение при помощи сухого льда.
19. Тестовый вопрос 19: Хладагенты – это:
а) вещества, используемые при безмашинном способе отвода тепла;
б) рабочие вещества паровых холодильных машин, с помощью которых
обеспечивается получение низких температур;
в) все вещества, используемые при охлаждении тела.
20. Тестовый вопрос 20: Одним из видов хладагентов, являющимся бесцветным газом с
удушливым сильным характерным запахом и имеющим достаточно высокую объемную
холодопроизводительность, является:
а) хладон-12;
б) хладон-22;
в) аммиак.
21. Тестовый вопрос 21: Какая часть компрессионной холодильной машины является
теплообменным аппаратом, служащим для сжижения паров хладагента путем их
охлаждения?
а) компрессор;
б) конденсатор;
в) ресивер.
22. Тестовый вопрос 22: По температурному режиму хранения торговое холодильное
оборудование подразделяется на:
а) для хранения запаса товаров вне торгового зала, в складских помещениях; для
хранения выставочного и текущего запаса товаров в торговом зале;
б) для охлажденных скоропортящихся продуктов, охлажденных напитков,
кратковременного хранения замороженных продуктов, длительного хранения
замороженных продуктов;
в) для хранения скоропортящихся товаров; для демонстрации образцов товаров в
оконных проемах, витринах, торговых залах магазинов, на выставках; для быстрого
замораживания воды.
23.Тестовый вопрос 23: Назовите виды отчетов в режиме отчетов и гашений:
а) отчет по кодам цен, почасовой отчет;
б) финансовый отчет, отчет по кассирам и секциям;
в) финансовый отчет (суточный), отчет по кассирам, отчет по секциям;
г) все перечисленные отчеты.
24.Тестовый вопрос 24: Исправность ККТ подтверждается:
а) требованием к использованию в соответствии с Классификатором контрольнокассовых машин, средствами визуального контроля «Государственный реестр» и
«Сервисное обслуживание»;
б) техническим обслуживанием в центре технического обслуживания;
в) регистрацией в налоговых органах;
г) всеми перечисленными мероприятиями.
25.Тестовый вопрос 25:… – совокупность сведений о каждом изготовленном экземпляре
модели контрольно-кассовой техники.
26.Тестовый вопрос 26: Виды клавиш цифровой клавиатуры ККТ:
а) цифровые;
б) секционные;
в) функциональные;
г) все виды.
27.Тестовый вопрос 27: Укажите последовательность действий контролера-кассира при
подготовке ККТ к работе:
78

а) напечатать 2 – 3 чека без обозначения суммы с целью проверки четкости
печатания реквизитов на чековой и контрольной ленте;
б) заправить чековую и контрольную ленты в чекопечатающий механизм;
в) пригласить представителя администрации и в его присутствии провести действия
по подготовке ККТ к работе;
г) включить машину в электросеть;
д) приложить в конец дня нулевые чеки к кассовому отчету.
28.Тестовый вопрос 28: Установите последовательность действий при регистрации ККТ в
налоговых органах:
а) проверка достоверности данных указанных в заявлении;
б) подача заявления по месту регистрации ККТ в установленной форме и
предоставление необходимых документов;
в) регистрация ККТ;
г) выдача карточки регистрации ККТ в налоговых органах.
29. Тестовый вопрос 29: …– система сохранения жизни и здоровья работников в процессе
трудовой деятельности, включающая в себя правовые, социально-экономические,
организационно-технические,
санитарно-гигиенические,
лечебно-профилактические,
реабилитационные и иные мероприятия.
30. Тестовый вопрос 30: Установите соответствие причин производственного травматизма
и заболеваний:
Технические
а) нарушение правил эксплуатации транспорта и оборудования,
плохая организация погрузочно-разгрузочных работ, нарушение
режима труда и отдыха, нарушение правил техники
безопасности
Организационные

б)
конструктивные
недостатки
машин,
механизмов,
инструментов,
приспособлений
и
их
неисправность;
несовершенство технологического процесса, неисправность
электропроводки, недостатки в освещении, отоплении, шум

Санитарногигиенические

в) нарушение требований санитарных норм, высокий уровень
шума, вибраций, излучений, нерациональное освещение;
несоблюдение правил личной гигиены

Психофизиологические г) усталость, монотонность, высокая напряженность труда
31. Тестовый вопрос 31: Документ, удостоверяющий соответствие проводимых
работодателем работ по охране труда государственным нормативным требованиям охраны
труда:
а) качественное удостоверение по охране труда;
б) сертификат соответствия качества;
в) сертификат соответствия организации работ по охране труда;
г) карточка соответствия организации работ по охране труда.
32. Тестовый вопрос 32: Производственный инструктаж по характеру и времени
проведения а) подразделяется на:
а) вводный, повторный, внеплановый и текущий;
б) вводный, первичный на рабочем месте, повторный, внеплановый и текущий;
в) вводный, первичный на рабочем месте, текущий;
г) первичный на рабочем месте, повторный, внеплановый.
33. Тестовый вопрос 33:…– совокупность факторов производственной среды и трудового
процесса, оказывающих влияние на работоспособность и здоровье работника.
34. Тестовый вопрос 34: Безопасные условия труда:
79

а) условия труда, при которых воздействие на работающих вредных и (или) опасных
производственных факторов исключено либо уровни их воздействия не превышают
установленных нормативов;
б) условия труда, при которых воздействие на работающих вредных и (или) опасных
производственных факторов исключено;
в) условия труда, при которых воздействие на работающих вредных и (или) опасных
производственных факторов не исключено;
г) условия труда, при которых воздействие на работающих вредных и (или) опасных
производственных факторов исключено либо уровни их воздействия превышают
установленных нормативов.
35. Тестовый вопрос 35:… - технические средства, используемые для предотвращения или
уменьшения воздействия на работников вредных и (или) опасных производственных
факторов, а также для защиты от загрязнения.
36. Тестовый вопрос 36:… – система организационных мероприятий, технических
средств и методов, предотвращающих воздействие на работающих опасных
производственных факторов.
37. Тестовый вопрос 37: Установите соответствие между электробезопасностью и
пожарной безопасностью
Электробезопасность а) состояние защищённости личности, имущества, общества и
государства от пожаров
Пожарная
безопасность

б) система мероприятий, предотвращающих
работающих опасных производственных факторов

воздействие

на

система организационных и технических мероприятий по защите
человека от действия электрического тока, электрической дуги,
статического электричества, электромагнитного поля
37. Тестовый вопрос 37: Системы деактивации могут быть:
а) встраиваемые в прилавок у кассы;
б) размещаемые по усмотрению заказчика,
в) ручные;совмещенные со сканером штриховых кодов;
г) все перечисленные.
38. Тестовый вопрос 38:… - технические средства, используемые для предотвращения или
уменьшения воздействия на работников вредных и (или) опасных производственных
факторов, а также для защиты от загрязнения.
39. Тестовый вопрос 39: Какой федеральный закон определяет основы обеспечения
пожарной безопасности?
а) Федеральный закон от 21.12.1994 N 69-ФЗ "О пожарной безопасности";
б) Федеральный закон от 21.07.1997 N 116-ФЗ "О промышленной безопасности
опасных производственных объектов";
в) Федеральный закон от 28.12.2010 N 390-ФЗ "О безопасности";
г) Федеральный закон от 22.07.2008 N 123-ФЗ "Технический регламент о
требованиях пожарной безопасности".
40. Тестовый вопрос 40: Какой из перечисленных видов пожарной охраны не
предусмотрен федеральным законодательством?
а) Государственная противопожарная служба;
б) межведомственная пожарная охрана;
80

в) муниципальная пожарная охрана;
г) ведомственная пожарная охрана;
д) частная пожарная охрана;
е) Добровольная пожарная охрана
Вариант 2
1. Тестовый вопрос 1: Укажите требование, предусматривающее единообразие всех
типов и размеров выпускаемой мебели:
а) типизация;
б) стандартизация;
в) унификация;
г) все вышеперечисленные.
2. Тестовый вопрос 2: Укажите торговую мебель, предназначенную для хранения товаров.
а) горки;
б) стеллажи;
в) прилавки;
г) витрины.
3. Тестовый вопрос 3: Укажите торговый инвентарь, применяемый для определения
качества товара.
а) кондитерские лопатки;
б) мусат;
в) овоскоп-виноскоп;
г) кондитерские щипцы.
4. Тестовый вопрос 4: Нож для сыра, совок для бакалейных товаров, кондитерские щипцы
относятся к торговому инвентарю:
а) для определения качества товаров;
б) для подготовки к продаже и продажи товаров;
в) вспомогательному инвентарю;
г) для приемки товаров.
5. Тестовый вопрос 5: Скорость взвешивания – это:
а) метрологическое требование, характеризующее свойство весов при многократном
взвешивании одного и того груза независимо от его место расположения на грузоприемном
устройстве давать одинаковые показатели;
б) метрологическое требование, характеризующее свойство весов показывать массу
с отклонением от действительных значений в пределах допускаемой погрешности;
в) свойство весов при выведении их из состояния равновесия самостоятельно после
нескольких колебаний возвратится в первоначальное положение;
г) свойство весов, характеризующее их способность при взвешивании быстро
приходить в состояние равновесия.
6. Тестовый вопрос 6: Укажите предел взвешивания на торговых электронных настольных
весах типа ВЕ – 15 ТЕ. 2:
а) 2 кг;
б) 20 кг;
в) 5 кг;
г) 15 кг.
7. Тестовый вопрос 7: Свойство весов измерять массу товара с отклонением от истиной
массы на величину, не превышающую установленную законом допустимую погрешность –
это …
8. Тестовый вопрос 8: Точность взвешивания – это:
81

а) метрологическое требование, характеризующее свойство весов при многократном
взвешивании одного и того груза независимо от его место расположения на грузоприемном
устройстве давать одинаковые показатели;
б) метрологическое требование, характеризующее свойство весов показывать массу
с отклонением от действительных значений в пределах допускаемой погрешности;
в) свойство весов при выведении их из состояния равновесия самостоятельно после
нескольких колебаний возвратится в первоначальное положение;
г) свойство весов, характеризующее их способность при взвешивании быстро
приходить в состояние равновесия.
9. Тестовый вопрос 9: Установите соответствие между видами требований к весам и их
содержанием:
1. Метрологические
а) наглядность показаний,
надежность, максимальная
скорость взвешивания
2.Эксплуатационные
б) форма конструкции,
цветовое решение
в) точность взвешивания,
чувствительность, устойчивость, постоянство показаний
10. Тестовый вопрос 10:
Установите соответствие между классификационными
признаками весов и их видами:
1. По виду указательного а) весы с термопечатью этикеток
устройства
2. По способу установки
б) электронно-индикаторные
3. По комплектности
в) напольные
4. По источнику
г) от универсальной системы электропитания
электропитания
11. Тестовый вопрос 11: Чем отличается тельфер от электротали?
а) тельфер не на электроприводе;
б) тельфер перемещается под потолком здания по монорельсу;
в) тельфер имеет приспособление, обеспечивающее удерживание грузов в
подвешенном состоянии;
г) тельфер не перемещается в горизонтальном направлении.
12. Тестовый вопрос 12: Для чего служат конвейеры?
а) для горизонтального и слабонаклонного перемещения сыпучих и штучных
товаров;
б) для вертикального перемещения длинномерных грузов;
в) для штабелирования бочек;
г) для стеллажирования грузов.
13. Тестовый вопрос 13: Для чего предназначены грузоподъёмные машины?
а) для механизации вертикального и резко наклонного перемещения грузов;
б) для механизации вертикального, резко наклонного и, в отдельных случаях,
горизонтального перемещения грузов в определённых границах;
в) для механизации вертикального, резко наклонного и горизонтального
перемещения грузов;
г) для механизации вертикального и резко наклонного перемещения грузов, а также
для их стеллажирования.
14. Тестовый вопрос 14: Что представляет собой электроталь?
а) грузоподъёмная машина, перемещаемая под потолком здания по монорельсу;
82

б) транспортирующая машина, несущим и тяговым элементом которой является
замкнутая лента из прорезиненной ткани, натянутой на приводной и натяжной барабаны;
в) компактная подвесная подвижная или неподвижная грузоподъёмная машина в
виде вращающего барабана со стальным тяговым канатом и с приводом от
электродвигателя;
г) машина, предназначенная для стеллажирования грузов.
15. Тестовый вопрос 15: Краны-манипуляторы – это…
а) стационарные машины, предназначенные для погрузочно-разгрузочных работ;
б) машины, передвигающиеся по рельсам и выполняющие погрузочно-разгрузочные
работы;
в) машины, смонтированные на транспортных средствах, предназначенные для
погрузки и разгрузки этих транспортных средств;
г) машины, смонтированные на транспортных средствах, предназначенные для
выполнения погрузочных работ на открытых площадках.
16. Тестовый вопрос 16: Погрузчики – это…
а) стационарные машины, предназначенные для погрузочно-разгрузочных работ;
б) передвижные машины, необходимые только для подъёма грузов;
в) передвижные машины, выполняющие складские работы с помощью специальных
грузозахватных приспособлений;
г) машина, для погрузки навалочных грузов, грузонесущим элементом которой
являются ролики.
17. Тестовый вопрос 17: Централизованное хладоснабжение:
а) представляет собой систему хладоснабжения на базе автономных компрессорноконденсаторных агрегатов, располагаемых в машинном отделении, изолированном от
торговых помещений, при этом каждый агрегат может обеспечивать холодом несколько
потребителей;
б) представляет собой многокомпрессорный блок с единым микропроцессорным
управлением, как правило, на базе полугерметичных поршневых или спиральных
компрессоров;
в) представляет собой систему хладоснабжения на базе автономных компрессорноконденсаторных агрегатов
18. Тестовый вопрос 18: Какой способ получения искусственного холода основан на
свойстве твердой углекислоты при поглощении тепла переходить из твердого состояния в
газообразное, минуя жидкое состояние?
а) льдосоляное охлаждение;
б) охлаждение при помощи сухого льда;
в) охлаждение водным льдом.
19. Тестовый вопрос 19: Сублимация – это:
а) переход твердого тела в жидкое состояние при определенной температуре;
б) переход тел из твердого состояния в парообразное, минуя жидкую фазу;
в) называется процесс превращения жидкости в пар.
20. Тестовый вопрос 20: … – техническое средство, которое по определенной программе
принимает платежное средство и автоматически выдает товар покупателю.
21. Тестовый вопрос 21: Системы электронного слежения за товарами (ЕАS) по принципу
действия делятся на группы:
а) радиочастотные, электромагнитные, акустические, пропускные (проходные),
видеонаблюдения;
б) радиочастотные, электромагнитные;
в) акустические;
83

г) пропускные (проходные), видеонаблюдения.
22. Тестовый вопрос 22: В радиочастотных и электромагнитных системах защиты товаров
от несанкционированного выноса используется эффект резонанса колебательного
а) контура;
б) движения;
в) звучания;
г) голоса.
23. Тестовый вопрос 23: Из приведенного перечня укажите те организации, которые в силу
специфики своей деятельности могут осуществлять денежные расчеты с населением без
применения контрольно-кассовой техники:
а) торговля на розничных рынках, ярмарках;
б) реализация товаров в гастрономе;
в) реализация товаров в супермаркете;
г) все вышеперечисленные организации.
24. Тестовый вопрос 24: … – это электронные вычислительные машины, иные
компьютерные устройства и их комплексы, обеспечивающие запись и хранение
фискальных данных в фискальных накопителях, формирующие фискальные документы,
обеспечивающие передачу фискальных документов в налоговые органы через оператора
фискальных данных и печать фискальных документов на бумажных носителях в
соответствии с правилами, установленными законодательством Российской Федерации о
применении контрольно-кассовой техники.
25.Тестовый вопрос 25: Первичный учетный документ, сформированный в электронной
форме и (или) отпечатанный с применением контрольно-кассовой техники в момент
расчета между пользователем и покупателем (клиентом), содержащий сведения о расчете,
подтверждающий факт его осуществления и соответствующий требованиям
законодательства Российской Федерации о применении контрольно-кассовой техники:
а) кассовый чек;
б) товарный чек;
в) счет-фактура;
г) фискальный документ.
26.Тестовый вопрос 26: В чем смысл кода цен и отчета по продажам через коды цен:
а) в данном отчете отображается общая информация о всех продажах за день;
б) в данном режиме печатается отчет для каждого кассира;
в) в данном отчете отражается информация о продажах по каждой секции отдельно;
г) данный режим позволяет получить информацию о количестве реализованного
товара по каждой товарной группе (подгруппе, наименованию).
27.Тестовый вопрос 27: Обязательные реквизиты кассового чека:
а) наименование документа; порядковый номер за смену; дата, время и место (адрес)
осуществления расчета;
б) наименование организации-пользователя или фамилия, имя, отчество
индивидуального предпринимателя - пользователя; идентификационный номер
налогоплательщика пользователя; применяемая при расчете система налогообложения;
в) наименование товаров, работ, услуг, платежа, выплаты, их количество,
цена;форма расчета и сумма оплаты; регистрационный номер контрольно-кассовой
техники; заводской номер экземпляра модели фискального накопителя; фискальный
признак документа;
г) все вышеперечисленные.
28.Тестовый вопрос 28: Фискальный регистратор – это ККТ:
а) способная работать только в составе компьютерно-кассовой системы, получая
данные через каналы связи;
84

б) расширение функциональных возможностей, которой может достигаться только
за счет подключения дополнительных устройств;
в) имеющая возможность работать в компьютерно-кассовой системе, но не имеющая
возможности управлять работой системы;
г) имеющая возможность работать в компьютерно-кассовой системе и управлять ею.
29. Тестовый вопрос 29: Укажите последовательность оказания первой медицинской
помощи при ушибах, растяжениях и разрывах связок и мышц:
а) дать пострадавшему обезболивающее средство;
б) обеспечить поврежденной конечности покой и придать ей возвышенное
положение;
в) наложить холод на поврежденное место и тугую повязку;
г) доставить пострадавшего в медицинское учреждение.
30. Тестовый вопрос 30: В зависимости от характера воздействия производственные
несчастные случаи делятся на:
а) механические (ушибы, переломы, вывихи);
б) термические (ожоги);
в) химические (отравление);
г) электрические (поражение током).
31. Тестовый вопрос 31: Установите соответствие между методами анализа несчастных
случаев:
Топографический а) состоит в изучении причин несчастных случаев по месту их
происшествия
Статистический

б) основан на изучении причин травматизма по документам за
определенный период времени

Групповой

в) включает в себя детальное исследование всего комплекса
производственных условий, при которых произошел несчастный
случай

Монографический г) основан на изучении повторяющихся несчастных случаев вне
зависимости от тяжести повреждения
32. Тестовый вопрос 32: …– это случай, происшедший с работающим вследствие
воздействия опасного производственного фактора в процессе непосредственного
исполнения должностных обязанностей.
33. Тестовый вопрос 33: …– это случай, происшедший с работающим вследствие
воздействия опасного производственного фактора в процессе непосредственного
исполнения должностных обязанностей.
34. Тестовый вопрос 34: Установите соответствие между условиями труда и их
содержанием:
Оптимальные а) характеризуются уровнями производственных факторов, воздействие
условия труда которых в течение рабочей смены создает угрозу для жизни, высокий
риск развития острых профессиональных заболеваний, в том числе и
тяжелых форм
Допустимые
условия труда

б) условия, при которых сохраняется здоровье работников и создаются
предпосылки для поддержания высокого уровня работоспособности

85

Вредные
условия труда

в) характеризуются уровнями факторов среды и трудового процесса,
которые не превышают установленных гигиенических нормативов для
рабочих мест, а возможные изменения функционального состояния
организма восстанавливаются во время регламентированных перерывов
или к началу следующей смены и не должны оказывать неблагоприятного
действия в ближайшем и отдаленном периоде на состояние здоровья
работающих и их потомство

Опасные
г) характеризуются наличием вредных производственных факторов,
(экстремальные) превышающих
гигиенические
нормативы
и
оказывающих
условия труда неблагоприятное действие на организм работающего и его потомство.
35. Тестовый вопрос 35: Установите соответствие между факторами, влияющими на
условия труда в предприятиях:
Физические факторы а) патогенные микроорганизмы (бактерии, вирусы, риккетсии,
спирохеты, грибы), а также макроорганизмы (растения и животные)
Психофизиологически б) загрязненность воздуха вредными веществами, воздействие
е факторы
агрессивных веществ (кислот, щелочей) неприятных запахов
Биологические
факторы

в) движущиеся машины и механизмы, шум, вибрация, повышенная
или пониженная температура, подвижность, влажность, отсутствие
или недостаток естественного света, пульсация светового потока,
повышенная контрастность, прямая или отраженная блесткость

Химические факторы

г) физические перегрузки (статические и динамические) и нервнопсихические (умственное перенапряжение, монотонность труда,
эмоциональные перегрузки)

36. Тестовый вопрос 36: Срок хранения Акта по форме Н-1 о несчастном случае:
а) 10 лет;
б) 45 лет;
в) 5 лет
г) 20 лет.
37. Тестовый вопрос 37: Средства коллективной защиты от статического электричества по
принципу действия делятся на следующие виды:
а) заземляющие устройства, нейтрализаторы;
б) увлажняющие устройства, антиэлектростатические вещества;
в) экранирующие устройства;
г) все вышеперечисленные.
38. Тестовый вопрос 38: Ответственность должностных лиц за нарушение
законодательства пожаробезопасности возможна в виде:
а) дисциплинарного взыскания;
б) административного наказания;
в) уголовной ответственности;
г) все вышеперечисленные.
39. Тестовый вопрос 39: Самоклеящиеся метки подразделяются на:
а) невидимые метки, которые представляют собой прозрачную самоклеящуюся
основу с тонкой антенной посередине,
б) ценники-этикетки (различных цветовых исполнений) для двухстрочных этикетпистолетов с возможностью печати артикула, даты и цены;
86

в) термо- и термотрансферные метки, которые предназначены для печати на них
дополнительной информации о товаре (артикул, штриховой код, цена и т. п.) на термо- и
термотрансферных принтерах соответственно;
г) все вышеперечисленные.
40. Тестовый вопрос 40: … - технические средства, используемые для предотвращения или
уменьшения воздействия на работников вредных и (или) опасных производственных
факторов, а также для защиты от загрязнения.
Критерии оценки:
- оценка 5 (отлично) выставляется студенту за 86 – 100% выполненных тестовых заданий;
- оценка 4 (хорошо) выставляется студенту за 70 – 85 % правильно выполненных тестовых
заданий;
- оценка 3 (удовлетворительно) выставляется студенту за 50 – 69 % правильно выполненных
тестовых заданий;
- оценка 2 (неудовлетворительно) выставляется студенту, если выполнено менее 50 %
тестовых заданий.
56 ПРАКТИЧЕСКАЯ РАБОТА №7 ПОРЯДОК ПОВЕДЕНИЯ РАБОТНИКОВ
ТОРГОВЛИ В УСЛОВИЯ ВОЗНИКНОВЕНИЯ И РАЗВИТИЯ
НЕЖЕЛАТЕЛЬНЫХ СОБЫТИЙ. ПОРЯДОК ПРОВЕДЕНИЯ
ИДЕНТИФИКАЦИИ ОПАСНОСТЕЙ НА РАБОЧЕМ МЕСТЕ
Составление алгоритма действий по направлениям
Инструкция по эвакуации людей для руководителя и работников торгового центра в
случае возникновения угрозы и совершении террористических актов
1. Общие положения
Настоящая инструкция разработана в целях обеспечения антитеррористической
безопасности торгового центра (далее – ТЦ) и устанавливает порядок действий
руководителя и работников ТЦ при возникновении угрозы совершения
террористического акта и при его совершении.
Инструкция подготовлена на основе федерального законодательства, рекомендаций
Национального антитеррористического комитета.
В настоящей инструкции применяются следующие основные понятия, относящиеся к
сфере противодействия терроризму:
Террористический акт – совершение взрыва, поджога или иных действий, устрашающих
население и создающих опасность гибели человека, причинения значительного
имущественного ущерба, либо наступления иных тяжких последствий, в целях
воздействия на принятие решения органами власти или международными
организациями, а также угроза совершения указанных действий в тех же целях.
Контртеррористическая операция – комплекс специальных, оперативно- боевых,
войсковых и иных мероприятий с применением боевой техники, оружия и
специальных средств по пресечению террористического акта, обезвреживанию
террористов, обеспечению безопасности физических лиц, организаций и
учреждений, а также по минимизации последствий террористического акта.
При возникновении угрозы совершения террористического акта или его совершении
общее руководство мероприятиями осуществляет руководитель ТЦ или лицо его
замещающее, который обеспечивает максимальную безопасность работников,
87

посетителей ТЦ и самого объекта от террористического акта, создает условия,
способствующие расследованию преступления правоохранительными органами.
2. Действия руководителя и работников ТЦ
при возникновении угрозы совершения террористического акта
2.1. Руководитель ТЦ.
Руководитель ТЦ с
получением сообщения об
угрозе
совершения террористического акта ОБЯЗАН:
- При получении сообщения из официальных источников (территориальных
подразделений УФСБ, МВД, МЧС, ФС ВНГ и др.): обратной связью проверить
достоверность полученного сообщения;
2.1.1. При получении сообщения от анонимного источника по телефону:
- зафиксировать точное время начала разговора и его продолжительность;
- при наличии автоматического определителя номера (АОНа) сразу записать
определившийся номер на бумаге;
- при отсутствии АОНа или в случае, если он не сработал, не прерывать телефонного
разговора, не класть телефонную трубку на аппарат, а положить её рядом, с
другого телефона позвонить на телефонный узел связи или дежурную часть МВД с
просьбой установить номер телефона, откуда был сделан анонимный телефонный
звонок;
- при наличии звукозаписывающей аппаратуры следует сразу же извлечь кассету
(минидиск) с записью разговора и принять меры для её сохранности, установив на
её место другую кассету;
- подробно записать полученное сообщение на бумаге;
- по ходу разговора отметить:
• пол (мужской или женский) звонившего и особенности его (её) речи: голос (громкий,
тихий, грубый, веселый, невнятный и т.д.),
темп речи (быстрый, медленный),
произношение (отчетливое, искаженное, с заиканием, шепелявое, с акцентом или
диалектом),
манера речи (развязанная, с издевкой, с нецензурными выражениями), состояние
(спокойное, возбужденное);
• звуковой фон (шум автомашин или железнодорожного транспорта, музыка, звук телерадиоаппаратуры, голоса и др.);
• тип звонка (городской или междугородний);
- по возможности в ходе разговора получить ответы на следующие вопросы:
• когда, кому и по какому телефону звонит этот человек?
• какие требования он (она) выдвигает?
• выступает ли в роли посредника или представляет группу лиц?
• на каких условиях он (она) или они согласны отказаться от задуманного?
• как и когда с ним (ней) можно связаться?
• кому сообщить об этом звонке?
- добиться от звонящего максимально возможного промежутка времени для принятия
решений или совершения каких-либо действий;
- если возможно, еще в процессе разговора с помощью других работников сообщить в
правоохранительные органы, а если такой возможности нет, то после разговора;
2.1.2. При поступлении угрозы в письменной форме (по почте и в различного рода
анонимных материалах (записках, надписях, информации на диске и т.д.):
88

- после получения такого документа обращаться с ним максимально осторожно,
стараться не оставлять на нем отпечатков своих пальцев;
- не мять документ, не делать на нем пометок. По возможности убрать его в чистый
плотно закрываемый полиэтиленовый пакет и поместить в отдельную жесткую
папку. Если документ поступил в конверте – его вскрытие производить только с
левой или правой стороны, аккуратно отрезая кромки ножницами;
- сохранять все: документ с текстом, любые вложения, конверт и упаковку, т. к. они
могут содержать информацию о преступниках;
- зафиксировать круг лиц, имевших доступ к документу;
- не расширять круга лиц, имеющих доступ к документу;
- анонимные заявления направлять в территориальные подразделения УФСБ с
сопроводительным письмом, в котором необходимо указать признаки анонимных
материалов (вид, качество, каким способом и на чем исполнено), а также
обстоятельства, связанные с их распространением, обнаружением или получением;
анонимные материалы не должны сшиваться, склеиваться, на них не разрешается
делать надписи, подчеркивать или обводить отдельные места в тексте, писать
резолюции или указания. На анонимных материалах не должно оставаться
давленых следов при исполнении резолюций и других надписей на
сопроводительных письмах;
- оценить реальность угрозы для работников, посетителей ТЦ и объекта в целом;
уточнить у дежурного охранника (начальника службы безопасности) сложившуюся
на момент получения сообщения обстановку и возможное нахождение
подозрительных лиц (предметов) на территории ТЦ или вблизи него;
- отдать распоряжение о доведении информации в соответствии с разделом 10
настоящей инструкции и усилении охраны ТЦ;
поставить задачу работникам на ограничение доступа посетителей на территорию
ТЦ, обязать их немедленно докладывать при обнаружении подозрительных лиц
(предметов) руководителю лично;
- организовать экстренную эвакуацию посетителей и работников с угрожаемого участка
территории ТЦ. При невозможности определения конкретного участка проведения
террористического акта – с территории всего ТЦ. При оповещении посетителей об
эвакуации, с целью недопущения паники, следует употреблять формы сообщения,
не раскрывающие истинного характера угрозы и причины эвакуации;
- обеспечить безаварийное прекращение опасных технологических процессов;
- организовать перестановку припаркованных автомобилей на расстояние не ближе 100
м от объекта;
- обеспечить пути подъезда для специальных автомобилей УФСБ, МВД, МЧС, ФС ВНГ,
скорой медицинской помощи;
- отдать распоряжения о подготовке помещений для работы оперативного штаба по
проведению контртеррористической операции, оповестить и собрать специалистов,
способных быть проводниками или консультантами для прибывающих сил
правоохранительных органов, подготовить документацию антитеррористической
защищенности ТЦ, паспорт безопасности и т.д.;
- до прибытия сил, планируемых для участия в аварийно-спасательных и других
неотложных работах приступить к проведению первоочередных мероприятий,
направленных на обеспечение безопасности посетителей и работников ТЦ;
с прибытием оперативной группы правоохранительных органов доложить
обстановку, передать управление её руководителю и далее действовать по его
89

указанию, принимая все меры по обеспечению проводимых оперативной группой
мероприятий;
- обеспечить спасение и эвакуацию пострадавших посетителей и работников ТЦ;
- организовать встречу спецподразделения УФСБ, МВД, МЧС, ФС ВНГ, скорой
медицинской помощи, обеспечить им условия для проведения мероприятий по
предотвращению, локализации или ликвидации последствий террористического
акта.
2.2. Дежурный охранник.
С получением сообщения об угрозе совершения террористического акта дежурный
охранник ОБЯЗАН:
2.2.1. При получении сообщения из официальных источников (территориальных
подразделений УФСБ, МВД, МЧС, ФС ВНГ и др.):
- обратной связью проверить достоверность полученного сообщения;
- записать в журнале полученных и отданных сигналов дату и время получения
сообщения, от кого принято;
- по окончании разговора незамедлительно доложить о нём руководителю ТЦ (лицу, его
замещающему) в рабочее время суток по тел.
, в нерабочее время суток,
в выходные и праздничные дни по тел.
;
- в дальнейшем действовать по указаниям руководителя ТЦ;
2.2.2. При получении сообщения от анонимного источника по телефону:
- зафиксировать точное время начала разговора и его продолжительность;
- при наличии автоматического определителя номера (АОНа) сразу записать
определившийся номер. При наличии звукозаписывающей аппаратуры следует
сразу же извлечь кассету (минидиск) с записью разговора и принять меры для её
сохранности, установив на её место другую кассету;
- при отсутствии определителя номера или в случае, если он не сработал, не прерывать
телефонного разговора, не класть телефонную трубку на аппарат, а положить её
рядом, с другого телефона позвонить на телефонный узел связи или дежурную
часть МВД с просьбой установить номер телефона, откуда был сделан анонимный
телефонный звонок;
- подробно записать полученное сообщение на бумаге;
- по ходу разговора отметить:
• пол (мужской или женский) звонившего и особенности его (её) речи: голос (громкий,
тихий, грубый, веселый, невнятный и т.д.),
темп речи (быстрый, медленный),
произношение (отчетливое, искаженное, с заиканием, шепелявое, с акцентом или
диалектом),
манера речи (развязанная, с издевкой, с нецензурными выражениями), состояние
(спокойное, возбужденное);
• звуковой фон (шум автомашин или железнодорожного транспорта, музыка, звук телерадиоаппаратуры, голоса и др.);
• тип звонка (городской или междугородний);
- по возможности в ходе разговора получить ответы на следующие вопросы:
• когда, кому и по какому телефону звонит этот человек?
• какие требования он (она) выдвигает?
• выступает ли в роли посредника или представляет группу лиц?
• на каких условиях он (она) или они согласны отказаться от задуманного?
• как и когда с ним (ней) можно связаться?
90

• кому сообщить об этом звонке?
• в ходе разговора предложить звонившему соединить его с руководством ТЦ;
- по окончании разговора незамедлительно доложить о нём руководителю ТЦ (лицу, его
замещающему) в рабочее время суток по тел.
, в нерабочее время суток,
в выходные и праздничные дни по тел.
;
- в дальнейшем действовать по указаниям руководителя ТЦ.
2.3. Работники ТЦ.
С получением сообщения об угрозе совершения террористического акта
работники ТЦ ОБЯЗАНЫ:
- тщательно осматривать все возможные для совершения актов терроризма места на
предмет возможного обнаружения взрывных устройств или подозрительных
предметов, мин, снарядов, гранат, самодельных взрывных устройств - в сумках,
дипломатах, свертках и т.д. Не предпринимать самостоятельных мер по их
обезвреживанию, не трогать и не переставлять их;
- при обнаружении на территории ТЦ подозрительных посетителей совместно с
сотрудниками охраны принимать меры к их задержанию. Особое внимание
обращать на наличие у них каких-либо предметов, свертков и т.д.;
при появлении вблизи ТЦ вооруженных лиц незамедлительно ставить в известность
своих руководителей и сотрудников охраны;
- действовать по распоряжению руководителя с учетом сложившейся обстановки.
3. Действия руководителя и работников ТЦ при попытке вооруженного проникновения и
проникновении вооруженных лиц на территорию ТЦ
3.1. Руководитель ТЦ.
Руководитель ТЦ с получением информации (сигнала) о попытке вооруженного
проникновения и проникновении вооруженных лиц ОБЯЗАН:
- оценить реальность угрозы для посетителей, работников ТЦ и всего объекта в целом;
лично или через уполномоченное лицо незамедлительно информировать дежурные
службы в соответствии с разделом 10 настоящей инструкции;
принять меры к пресечению возможной паники, приступить к эвакуации посетителей
и работников ТЦ с угрожаемых направлений;
- обеспечить безаварийное прекращение опасных технологических процессов;
- отдать распоряжения о подготовке помещений для работы оперативного штаба по
проведению контртеррористической операции, оповестить и собрать специалистов,
способных быть проводниками или консультантами для прибывающих сил
правоохранительных органов, подготовить документацию, необходимую при
проведении контртеррористической операции;
с прибытием оперативной группы правоохранительных органов доложить
обстановку, передать управление её руководителю и далее действовать по его
указаниям, принимая все меры по обеспечению проводимых оперативной группой
мероприятий;
- организовать встречу спецподразделений УФСБ, МВД, МЧС, ФС ВНГ, скорой
медицинской помощи, обеспечить им условия для проведения мероприятий по
предотвращению, локализации или ликвидации последствий террористического
акта.
3.2. Дежурный охранник.

91

С получением информации (сигнала) о попытке вооруженного проникновения или
проникновении вооруженных лиц на территорию ТЦ дежурный охранник
ОБЯЗАН:
доложить о происшедшем руководителю ТЦ (лицу, его замещающему) в рабочее
время суток по тел.
, в нерабочее время суток, в выходные и праздничные
дни по тел.
;
- по указанию руководителя ТЦ или самостоятельно информировать дежурные службы
территориальных подразделений в соответствии с разделом 10 настоящей
инструкции;
- в дальнейшем действовать по распоряжениям руководителя;
- в экстренных случаях организовать эвакуацию работников ТЦ и посетителей.
3.3. Работники ТЦ.
При получении информации (сигнала) о попытке вооруженного проникновения или
проникновении вооруженных лиц на территорию ТЦ ОБЯЗАНЫ:
сообщить о случившемся руководителю ТЦ (лицу, его замещающему) о случившемся,
по его указанию или самостоятельно сообщить в «Службу «01» МЧС России» по
тел. «01» или по мобильному телефону «112» с указанием наименования объекта и
его адреса;
- в дальнейшем действовать по распоряжениям руководителя.
4. При обнаружении на территории ТЦ или в непосредственной близости от него
предмета, похожего на взрывное устройство
4.1. Руководитель ТЦ.
Руководитель ТЦ с получением информации об обнаружении на территории ТЦ или в
непосредственной близости от него предмета, похожего на взрывное устройство
ОБЯЗАН:
- оценить обстановку и полученную информацию;
- отдать распоряжение о доведении информации в соответствии с разделом 10
настоящей инструкции;
- до прибытия оперативно-следственной группы организовать на безопасном
расстоянии оцепление места нахождения подозрительного предмета.
Рекомендуемые расстояния удаления и оцепления при обнаружении взрывного устройства
или предмета похожего на взрывное устройство:
граната РГД-5 - не менее 50 м; граната Ф-1 - не менее 200 м;
тротиловая шашка массой 200 гр – 45 м; тротиловая шашка массой 400 гр – 55 м; пивная
банка 0,33 литра – 60 м;
чемодан (кейс) – 230 м; дорожный чемодан – 350 м;
автомобиль типа «Жигули» – 460 м; автомобиль типа «Волга» – 580 м; микроавтобус –
920 м;
грузовая машина (фургон) – 1240 м.
оградить и перекрыть доступ посетителей и работников ТЦ к месту обнаружения
подозрительного предмета;
- отдать распоряжение о запрещении пользоваться радио- и мобильной связью вблизи
обнаруженного предмета;
- отдать распоряжение о подготовке к эвакуации, выключении электроприборов и
электрооборудования, о нераспространении сведений о сложившейся ситуации,
соблюдении организованности, не допущении паники и самостоятельных действий
персонала и посетителей;
92

обеспечить возможность беспрепятственного подъезда к месту обнаружения предмета,
похожего на взрывное устройство, автомашин УФСБ, МВД, МЧС, скорой
медицинской помощи и аварийных служб;
- обеспечить присутствие лиц, обнаруживших находку, до прибытия оперативно- следственной группы и фиксирования их установочных данных;
- отдать распоряжение о подготовке помещений для работы оперативного штаба по
проведению контртеррористической операции, оповестить и собрать работников,
способных быть проводниками или консультантами для прибывающих сил
правоохранительных органов, подготовить документацию, необходимую при
проведении контртеррористической операции;
с прибытием оперативной группы правоохранительных органов доложить
обстановку, передать управление её руководителю и далее действовать по его
указаниям, принимая все меры по обеспечению проводимых оперативной группой
мероприятий, предоставить руководителю группы поэтажный план объекта и
указать место нахождения подозрительного предмета;
- организовать встречу спецподразделений УФСБ, МВД, МЧС, ФС ВНГ, скорой
медицинской помощи и создать им условия для проведения мероприятий по
предотвращению, локализации или ликвидации последствий террористического
акта;
- приступить в случае необходимости к эвакуации посетителей и работников с учетом
обхода места обнаружения подозрительного предмета.
4.2. Дежурный охранник.
С получением информации об обнаружении на территории ТЦ или в непосредственной
близости от него предмета, похожего на взрывное устройство дежурный охранник
ОБЯЗАН:
немедленно доложить о происшедшем руководителю ТЦ (лицу, его замещающему) в
рабочее время суток по тел. ; в нерабочее время суток, в выходные и праздничные
дни по тел.
;
- по указанию руководителя ТЦ или самостоятельно информировать дежурные службы
территориальных подразделений в соответствии с разделом 10 настоящей
инструкции;
- провести экстренную эвакуацию посетителей и работников ТЦ с угрожаемых участков,
силами охраны ТЦ оцепить опасную зону;
- обеспечить присутствие лиц, обнаруживших находку, до прибытия оперативноследственной группы правоохранительных органов и фиксирования их
установочных данных;
- при прибытии групп правоохранительных органов и МЧС действовать по их
указаниям.
4.3. Работники ТЦ.
С получением информации об обнаружении на территории ТЦ или в непосредственной
близости от него предмета, похожего на взрывное устройство работники ТЦ
ОБЯЗАНЫ:
при обнаружении подозрительного предмета (получении информации о заложенном
взрывном устройстве) немедленно сообщить руководителю ТЦ и сотрудникам
охраны;
- не нарушать (не трогать, не перемещать, не открывать, не развязывать и т.д.)
целостность обнаруженных предметов, не предпринимать самостоятельных мер по
их обезвреживанию;
-

93

осмотреть помещение и постараться запомнить приметы посетителей, их поведение,
место нахождения;
- принять меры к закрытию и опечатыванию денежных хранилищ, касс и других
помещений, где находятся материальные ценности;
- выключить электроприборы, проверить наличие и состояние средств оказания первой
медицинской помощи;
- оставаясь на рабочем месте, выполнять указания руководителя ТЦ;
- подготовиться к эвакуации, прослушав сообщение по сети оповещения об эвакуации
(или по распоряжению руководителя), организовать вывод посетителей с ТЦ,
соблюдая меры предосторожности.
5. При получении сообщения
об угрозе минирования (минировании) ТЦ
5.1. Руководитель ТЦ.
Руководитель ТЦ при получении сообщения (информации) об
угрозе минирования ТЦ лично ОБЯЗАН:
- отдать распоряжение о доведении информации в соответствии с разделом 10
настоящей инструкции;
- отдать распоряжения на усиление охраны ТЦ;
- организовать эвакуацию посетителей и работников со всего ТЦ;
- до прибытия оперативной группы правоохранительных органов организовать на
безопасном расстоянии оцепление объекта, оградить и перекрыть доступ к нему
граждан;
- отдать распоряжение о запрещении пользоваться радио и мобильной связью;
- отдать распоряжения о подготовке помещений для работы оперативного штаба по
проведению контртеррористической операции, оповестить и собрать специалистов,
способных быть проводниками или консультантами для прибывающих сил
правоохранительных органов, подготовить документацию, необходимую при
проведении контртеррористической операции;
с прибытием оперативной группы правоохранительных органов доложить
обстановку, передать управление её руководителю и далее действовать по его
указаниям, принимая все меры по обеспечению проводимых оперативной группой
мероприятий;
- организовать встречу спецподразделений УФСБ, МВД, МЧС, ФС ВНГ, скорой
медицинской помощи и обеспечить им условия для проведения мероприятий по
предотвращению, локализации или ликвидации последствий террористического
акта.
5.2. Дежурный охранник.
Дежурный охранник при получении сообщения (информации) об
угрозе минирования ТЦ ОБЯЗАН:
немедленно доложить о происшедшем руководителю ТЦ (лицу, его замещающему) в
рабочее время суток по тел.
, в нерабочее время суток, в выходные и
праздничные дни по тел.
;
- по указанию руководителя ТЦ или самостоятельно информировать дежурные службы
территориальных подразделений в соответствии с разделом 10 настоящей
инструкции;
- в дальнейшем действовать по распоряжениям руководителя ТЦ;
5.3. организовать эвакуацию посетителей и работников со всего ТЦ.
Работники ТЦ.
-

94

При получении сообщения (информации) об угрозе минирования ТЦ ОБЯЗАНЫ:
- сообщить руководителю ТЦ (лицу, его замещающему) об угрозе минирования, по их
указанию или самостоятельно сообщить в «Службу «01» МЧС России» по тел.
«01» или по мобильному телефону «112» с указанием наименования объекта и его адреса;
- в дальнейшем действовать по распоряжениям руководителя.
6. При совершении на территории ТЦ взрыва
6.1. Руководитель ТЦ.
Руководитель ТЦ при совершении на территории ТЦ взрыва ОБЯЗАН:
- оценить обстановку;
- обеспечить своевременное оповещение посетителей и работников ТЦ;
- принять все возможные меры, направленные на сохранение жизни и здоровья людей,
организовать эвакуацию посетителей и работников ТЦ;
- отдать распоряжение о доведении информации в соответствии с разделом 10
настоящей инструкции;
с прибытием оперативной группы правоохранительных органов доложить
обстановку, передать управление её руководителю и далее действовать по его
указаниям, принимая все меры по обеспечению проводимых оперативной группой
мероприятий;
- организовать оказание медицинской помощи пострадавшим и эвакуацию их в лечебные
учреждения, вывод посетителей и работников ТЦ в безопасные места;
- организовать встречу спецподразделений УФСБ, МВД, МЧС, ФС ВНГ, скорой
медицинской помощи и обеспечить им условия для проведения мероприятий по
локализации или ликвидации последствий террористического акта.
6.2. Дежурный охранник.
С получением сигнала (информации) о совершении взрыва дежурный охранник ОБЯЗАН:
немедленно доложить о происшедшем руководителю ТЦ (лицу, его замещающему) в
рабочее время суток по тел.
, в нерабочее время суток, в выходные и
праздничные дни по тел.
,
по указанию руководителя ТЦ или самостоятельно информировать дежурные службы
территориальных подразделений в соответствии с разделом 10 настоящей
инструкции;
- принять все меры по оповещению и эвакуации работников и посетителей ТЦ;
- отдать распоряжения по принятию мер к спасению раненых и пораженных, оказанию
первой медицинской помощи.
6.3. Работники ТЦ.
При совершении на территории ТЦ взрыва работники ТЦ ОБЯЗАНЫ:
- сообщить руководителю ТЦ (лицу, его замещающему) о совершении взрыва, по его
указанию или самостоятельно сообщить в «Службу «01» МЧС России» по тел.
«01» или по мобильному телефону «112» с указанием наименования объекта и его
адреса;
- принять меры к выводу посетителей с ТЦ согласно плану эвакуации;
- принять необходимые меры предосторожности во время возможной давки, возникшей
вследствие паники.
7. При захвате заложников на территории ТЦ
7.1. Руководитель ТЦ.
Руководитель ТЦ с получением информации о захвате заложников на территории ТЦ
ОБЯЗАН:
95

отдать распоряжение о доведении информации в соответствии с разделом 10
настоящей инструкции;
- принять меры к пресечению возможной паники, в случае необходимости подготовить
эвакуацию посетителей, работников и т.д.;
не допускать действий, которые могут спровоцировать нападающих к применению
оружия и человеческим жертвам;
- при необходимости выполнять требования преступников, если это не связано с
причинением ущерба жизни и здоровью людей. Не противоречить преступникам,
не рисковать жизнью окружающих и своей собственной; инициативно не вступать
в переговоры с террористами;
- принять все возможные меры, направленные на сохранение жизни и здоровья людей,
организовать эвакуацию работников и посетителей;
- отдать распоряжение о подготовке помещения для работы оперативного штаба по
проведению контртеррористической операции, оповестить и собрать специалистов,
способных быть проводниками или консультантами для прибывающих сил
правоохранительных органов, подготовить документацию, необходимую при
проведении контртеррористической операции (паспорт безопасности и т.д.);
- принять меры к беспрепятственному проходу (проезду) на объект сотрудников
правоохранительных органов, автомашин УФСБ, МВД, МЧС, ФС ВНГ, скорой
медицинской помощи;
с прибытием оперативной группы правоохранительных органов доложить
обстановку, передать управление её руководителю и далее действовать по его
указаниям, принимая меры по обеспечению проводимых оперативной группой
мероприятий.
7.2. Дежурный охранник.
Дежурный охранник с получением информации о захвате заложников на территории ТЦ
ОБЯЗАН:
немедленно доложить о происшедшем руководителю ТЦ (лицу, его замещающему) в
рабочее время суток по тел. _; в нерабочее время суток, в выходные и праздничные
дни по тел.
;
- по указанию руководителя ТЦ или самостоятельно информировать дежурные службы
территориальных подразделений в соответствии с разделом 10 настоящей
инструкции;
не допускать действий, которые могут спровоцировать нападающих к применению
оружия и человеческим жертвам;
- при необходимости выполнять требования преступников, если это не связано с
причинением ущерба жизни и здоровью людей. Не противоречить преступникам,
не рисковать жизнью окружающих и своей собственной; инициативно не вступать
в переговоры с террористами;
- принять меры к беспрепятственному проходу (проезду) на объект сотрудников
правоохранительных органов, автомашин УФСБ, МВД, МЧС, ФС ВНГ, скорой
медицинской помощи.
7.3. Работники ТЦ.
В случае захвата в заложники на территории ТЦ работники ОБЯЗАНЫ:
не допускать действий, которые могут спровоцировать нападающих к применению
оружия и привести к человеческим жертвам;
- по своей инициативе не вступать в переговоры с террористами;
-

96

- постараться запомнить приметы преступников, отличительные черты их лица, одежду,
имена, клички, возможные шрамы и татуировки, особенности речи и манеры
поведения, тематику разговоров, вооружение, средства передвижения и т.д.;
- стараться спокойно переносить лишения, оскорбления и унижения, не смотреть в глаза
преступникам, не вести себя вызывающе;
при необходимости выполнять требования нападающих, действовать с
максимальной задержкой, но без риска для жизни окружающих и своей
собственной;
- на совершение любых действий спрашивать разрешение у преступников;
- при наличии возможности (отсутствии угрозы себе и окружающим) сообщить
сотрудникам правоохранительных органов информацию о складывающейся
ситуации и преступниках.
7.4. При проведении спецслужбами операции по освобождению от преступников
руководитель, работники и посетители ТЦ обязаны неукоснительно соблюдать
следующие требования:
- лечь на пол лицом вниз, голову закрыть руками и не двигаться;
- не бежать навстречу сотрудникам спецслужб или от них, так как они могут принять вас
за преступников;
- если есть возможность, держаться подальше от проемов дверей и окон;
- при ранении постараться не двигаться с целью уменьшения потери крови.
8. При совершении террористического акта с применением биологических веществ
8.1. Руководитель ТЦ.
Руководитель ТЦ при получении информации о совершении террористического акта на
территории ТЦ с применением биологического вещества (аэрозоля) ОБЯЗАН:
- оценить обстановку и полученную информацию;
- отдать распоряжение перекрыть все выходы на улицу из здания (помещения), где
совершён террористический акт с применением биологического вещества,
установить на всех выходах посты охраны, прекратить сообщения между этажами,
движение персонала, посетителей в другие помещения;
- отключить вентиляцию, кондиционеры, закрыть форточки, окна, двери;
- отдать распоряжение о доведении информации в соответствии с разделом 10
настоящей инструкции;
- вывесить на входных дверях ТЦ объявление о временном его закрытии;
прекратить доступ посетителей и работников в здание (помещение), где совершен
террористический акт с применением биологического вещества, до прибытия
специалистов служб;
- до прибытия специалистов служб обеспечить присутствие всех лиц, в том числе
работников,
находящихся
в
зоне поражения, предварительно
записав
их ФИО, домашние адреса, телефоны, места работы, должности;
обеспечить выполнение всех рекомендаций и требований прибывших сотрудников
служб.
8.2. Дежурный охранник.
Дежурный охранник при получении информации о террористическом акте на территории
ТЦ с применением биологического вещества (аэрозоля) ОБЯЗАН:
немедленно доложить о происшедшем руководителю ТЦ (лицу, его замещающему) в
рабочее время суток по тел. _; в нерабочее время суток, в выходные и праздничные
дни по тел.
;
97

по указанию руководителя ТЦ или самостоятельно информировать дежурные службы
территориальных подразделений в соответствии с разделом 10 настоящей
инструкции;
- перекрыть все выходы из здания (помещения), установить на всех выходах посты
охраны, прекратить сообщения между этажами, движение персонала, посетителей
в другие помещения;
- вывесить на входных дверях ТЦ объявление о временном его закрытии;
- отключить вентиляцию, кондиционеры, закрыть форточки, окна, двери;
- всех контактировавших с биологическим веществом изолировать в помещении, где
совершен акт биотерроризма, предварительно записав их ФИО, домашние адреса,
телефоны, места работы и должности, обеспечить их присутствие до прибытия
специалистов служб;
обеспечить выполнение всех рекомендаций и требований прибывших сотрудников
служб.
8.3. Работники ТЦ.
Работники ТЦ при получении информации о совершении террористического акта на
территории ТЦ (в помещении) с применением биологического вещества
ОБЯЗАНЫ:
- немедленно, не выходя из помещения, доложить о происшествии или через работников
руководителю ТЦ (лицу его замещающему) по телефону
_, а при его
отсутствии в «Службу «01» МЧС России» по тел. «01» или по мобильному
телефону «112» и дежурному охраннику ТЦ;
- прекратить доступ других лиц на территорию ТЦ (в помещение) до прибытия
специалистов служб;
- отключить вентиляцию, кондиционеры, закрыть форточки, окна, двери;
- оставаться в помещении, где совершен биотеррористический акт, переписать всех
контактировавших с биологическим веществом, указав их ФИО, домашние адреса,
телефоны, места работы и должности;
обеспечить выполнение всех рекомендаций и требований прибывших сотрудников
служб.
9. При совершении террористического акта
с применением химически опасных и радиоактивных веществ
9.1. Руководитель ТЦ.
Руководитель ТЦ при получении информации о совершении террористического акта на
территории ТЦ (в помещении) с применением химически опасных и
радиоактивных веществ ОБЯЗАН:
- оценить обстановку и полученную информацию;
- отдать распоряжение:
•
оповестить посетителей, работников ТЦ, сообщить маршрут выхода в безопасное
место;
•
отключить вентиляцию, кондиционеры, закрыть форточки, окна, двери, отключить
электронагревательные и бытовые приборы;
• подготовить воду, 2 % раствор питьевой соды в случае выброса химических веществ,
йодистый препарат (раствор йода) – в случае радиоактивного загрязнения;
• подготовить простейшие средства защиты дыхания (ватно-марлевые повязки, платки,
шарфы, изделия из тканей, предварительно смоченные содовым раствором или
водой);
• выдать противогазы;
-

98

отдать распоряжение о доведении информации в соответствии с разделом 10
настоящей инструкции;
обеспечить выполнение всех рекомендаций и требований прибывших сотрудников
служб.
9.2. Дежурный охранник.
Дежурный охранник при получении информации о террористическом акте на территории
ТЦ (в помещении) с применением химически опасных и радиоактивных веществ
ОБЯЗАН:
немедленно доложить о происшедшем руководителю ТЦ (лицу, его замещающему) в
рабочее время суток по тел. _; в нерабочее время суток, в выходные и праздничные
дни по тел.
;
- по указанию руководителя ТЦ или самостоятельно информировать дежурные службы
территориальных подразделений в соответствии с разделом 10 настоящей
инструкции;
оповестить посетителей, работников ТЦ, сообщить маршрут выхода в безопасное
место;
отключить вентиляцию, кондиционеры, закрыть форточки, окна, двери, отключить
электронагревательные и бытовые приборы;
- подготовить воду, 2 % раствор питьевой соды - в случае выброса химических веществ,
йодистый препарат (раствор йода) – в случае радиоактивного загрязнения;
- подготовить простейшие средства защиты дыхания (ватно-марлевые повязки, платки,
шарфы, изделия из тканей, предварительно смоченные водой);
- выдать противогазы;
- оставаться на месте до прибытия специалистов служб;
обеспечить выполнение всех рекомендаций и требований прибывших сотрудников
служб.
9.3. Работники ТЦ.
Работники ТЦ при получении информации о совершении террористического акта на
территории ТЦ (в помещении) с применением химически опасных и
радиоактивных веществ ОБЯЗАНЫ:
- немедленно надеть противогаз;
в случае отсутствия противогаза необходимо немедленно выйти из зоны заражения,
при этом для защиты органов дыхания использовать подручные средства
(ватно-марлевые повязки, платки, шарфы, изделия из тканей, предварительно смоченные
содовым раствором или водой);
отключить вентиляцию, кондиционеры, закрыть форточки, окна, двери, отключить
электронагревательные и бытовые приборы;
- предупредить посетителей, быстро, но без паники выйти с территории ТЦ в указанном
в информации направлении или в сторону, перпендикулярную направлению ветра,
на хорошо проветриваемый участок местности, где необходимо находиться до
получения дальнейших распоряжений;
- при невозможности выхода из зоны заражения нужно немедленно укрыться в
помещении и загерметизировать его. (Следует помнить, что опасные химические
вещества тяжелее воздуха (хлор, фосген и др.) будут проникать в нижние этажи
зданий и подвальные помещения, в низины и овраги, а опасные химические
вещества легче воздуха (аммиак) будут заполнять более высокие места);
-

99

- после выхода из зоны заражения, необходимо снять верхнюю одежду и оставить её на
улице, принять душ с мылом (пройти санитарную обработку), тщательно промыть
глаза и прополоскать рот;
лица, получившие незначительные поражения должны исключить любые физические
нагрузки, принять обильное теплое питье и обратиться к прибывшим работникам
медицинского учреждения для определения степени поражения и проведения
профилактических и лечебных мероприятий.
10. Порядок информирования об угрозе совершения или о совершении
террористического акта
10.1.
При обнаружении угрозы совершения террористического акта на территории
ТЦ, получении информации (в том числе анонимной) об угрозе совершения
террористического акта на территории ТЦ или совершении террористического акта
на территории ТЦ руководитель ТЦ (лицо, его замещающее) либо уполномоченное
им лицо незамедлительно информирует об этом любыми доступными средствами
связи:
- дежурного Управления ФСБ России по Саратовской области, тел.
;
- дежурного Управления МВД России по Саратовской области, тел.
;
- дежурного Управления Росгвардии по Саратовской области, тел. _
;
оперативного дежурного ГУ МЧС России по Саратовской области, тел.
;
- правообладателя торгового объекта (территории)
;
- администрацию муниципального образования, тел
.
10.2.
При передаче в соответствии с пунктом 10.1 настоящей инструкции информации
с помощью средств связи лицо, передающее информацию, сообщает:
- свои фамилию, имя, отчество (при наличии) и должность;
- наименование ТЦ и его точный адрес;
дату и время обнаружения угрозы совершения террористического акта, получения
информации об угрозе совершения террористического акта или совершения
террористического акта;
- количество находящихся на территории ТЦ людей;
- другие значимые сведения по запросу территориальных органов ФСБ, МВД,
Росгвардии, МЧС.
10.3.
Лицо, передавшее информацию об угрозе совершения или о совершении
террористического акта, фиксирует фамилию, имя, отчество (при наличии), должность
лица, принявшего информацию, а также дату и время ее передачи. При направлении такой
информации с использованием средств факсимильной связи лицо, передающее
информацию, удостоверяет сообщение своей подписью.

100

57 ПОРЯДОК ПРОВЕДЕНИЯ ИДЕНТИФИКАЦИИ ОПАСНОСТЕЙ НА РАБОЧЕМ
МЕСТЕ
Заполнение карты уровня профессионального риска

58 ОЦЕНКА РИСКОВ НА РАБОЧЕМ МЕСТЕ
Заполнение карточки риска на рабочем месте кладовщик, кассир, товаровед,
грузчик.
Сообщение о небезопасной ситуации
№______

от ____________

г. Саратов
Автор сообщения
Структурное подразделение
Место возникновения опасности
При выполнении каких трудовых функций
возникает опасность
Какие безотлагательные меры, на Ваш
взгляд, можно предпринять:
101

Ответственный
Какие корректирующие меры, на Ваш
взгляд, можно предпринять (если
необходимо)

59 ПРАКТИЧЕСКАЯ РАБОТА №8
ОСНОВНЫЕ ПРИЧИНЫ ТРАВМАТИЗМА И ПРОФЕССИОНАЛЬНЫХ
ЗАБОЛЕВАНИЙ.
Установление и описание причин производственного травматизма и заболеваний для
конкретного рабочего места согласно установленных групп:
1. Технические причины: конструктивные недостатки машин, механизмов, инструментов,
приспособлений или их неисправность; отсутствие, несовершенство, неисправность
оградительных, блокировочных, вентиляционных устройств, зануления или заземления
электроустановок; подтекание ядовитых жидкостей, газов через неплотности соединений
трубопроводов, шлангов и др..
2. Организационные причины: несвоевременное или некачественное проведение
инструктажей и обучения по охране труда работающих; отсутствие инструкций по охране
труда; недостаточный контроль за выполнением требований охраны труда работающими;
неудовлетворительное содержание рабочего места; недостатки в организации
коллективных работ, в обеспечении рабочих спецодеждой и другими СИЗ; использование
техники, инструментов не по назначению; нарушение режима труда и отдыха,
технологического процесса.
3. Санитарно-гигиенические причины: неблагоприятные природно-климатические условия
или микроклимат в помещениях; повышенное содержание в воздухе вредных веществ;
высокий уровень шума, вибраций, излучений; нерациональное освещение;
антисанитарное состояние рабочих мест и бытовых помещений; несоблюдение правил
личной гигиены и др..
4. Психофизиологические причины: монотонность, высокая напряжённость труда;
несоответствие анатомо-физиологических и психологических особенностей организма
условиям труда; усталость; неудовлетворительная психологическая обстановка в
коллективе и др..
5. Субъективные причины: личная недисциплинированность работника; невыполнение
инструкций по охране труда; нахождение в состоянии алкогольного или наркотического
опьянения, в болезненном состоянии и др..
6. Экономические причины: стремление работающих обеспечить высокую выработку и
заработную плату при пренебрежительном отношении к вопросам охраны труда;
недостаточное выделение средств на мероприятия по улучшению условий труда и др

102

60 МЕТОДЫ УМЕНЬШЕНИЯ ОПАСНОСТЕЙ НА РАБОЧЕМ МЕСТЕ, ВЫБОР
СРЕДСТВ ИНДИВИДУАЛЬНОЙ И КОЛЛЕКТИВНОЙ ЗАЩИТЫ.
Тестирование
1.Средство защиты кожи – это:
комбинезон;
противогаз;
ватно-марлевая повязка; респиратор.
2. По принципу защиты, средства индивидуальной защиты могут быть:
промышленные;
простейшие;
фильтрующие;
подручные.
3. Наиболее надежное средство защиты органов дыхания:
респиратор;
противогаз;
ватно-марлевая повязка;
тканевая маска.
4. Воздухонепроницаемые средства защиты называются:
изолирующими;
табельными;
статическими;
фильтрующими
5. По способу изготовления, средства индивидуальной защиты могут быть:
промышленные;
противопылевые;
табельные;
подручные.
6. Время защиты от АХОВ брезентовых изделий в зимнее время:
10 минут;
30 минут;
1 час;
3 часа.
7. Средство защиты рук от опасных факторов:
скафандры;
мази;
каски;
перчатки.
8. Преимуществом фильтрующих средств защиты является:
ограниченный срок годности;
затрудненность дыхания;
возможность длительного применения;
малый вес.
9. К коллективным средствам защиты от ионизирующих излучений не относят:
герметизирующие;
знаки безопасности;
теплоизолирующие устройства;
оградительные.
10. Какой тип коллективного средства защиты предотвращает попадание человека в
опасную зону? оградительный;
теплоизолирующий;
103

герметизирующий;
сигнализирующий.
11. Какие устройства используются для автоматического отключения оборудования в
случае каких-либо отклонений при попадании человека в опасную зону?
информационные;
предохранительные;
ограждающие;
герметизирующие.
12. Система промышленной сигнализации не может быть:
цветовой;
световой;
механической;
знаковой.
13. Если убежище вмещает до 2000 человек, то оно относится к категории:
малого;
большого;
крупного;
среднего.
14. Режим вентиляции убежища, при котором наружный воздух очищается от пыли:
фильтрующая;
очистная;
поглотительная;
специальная.
15. Аварийный запас воды в убежищах на случае выхода из строя водопровода:
3л/сут;
2,5л/сут;
2л/сут;
1,5л/сут.

61-62 АЛГОРИТМ БЕЗОПАСНОГО ПОВЕДЕНИЯ ПРИ ВОЗНИКНОВЕНИИ ЧС
ПРИРОДНОГО ХАРАКТЕРА
ТЕСТИРОВАНИЕ ВАРИАНТ №1.
Выберите три верных ответа из шести предложенных.
1.На здоровье человека оказывает влияние факторы риска естественной природной
среды. Укажите их.
1.Солнечная активность.
2.Изменение микроклимата.
3.Возрастание напряжённости электромагнитного поля Земли.
4.Радиация.
5.Изменение метеорологических условий.
6. Изменение ландшафта окружающей местности.
2. В задании перечислены возможные места для укрытия в здании при
землетрясении. Укажите их.
104

1. Места внутри шкафов, комодов, гардеробов.
2. Места под прочно закреплёнными столами.
3. Места под подоконниками.
4. Углы, образованные внутренними перегородками.
5. Проёмы в капитальных внутренних стенах.
6. Дверной проём на лестничную площадку.
3. Назовите причины несчастных случаев при землетрясениях.
1. Неконтролируемые действия людей в результате паники.
2. Отсутствие защитных сооружений.
3. Отсутствие средств пожаротушения.
4. Отсутствие средств индивидуальной защиты органов дыхания.
5. Разрушение и падение конструкций зданий.
6. Падение тяжёлых предметов в квартире.
4.Признаками приближающегося землетрясения могут быть разные явления. Укажите
их.
1. Резкое изменение погодных условий.
2. Голубоватое свечение внутренней поверхности домов.
3. Самовоспламенение и самовозгорание горючих веществ и материалов.
4. Запах газа в районах, где раньше этого не отмечалось.
5. Вспышки в виде рассеянного света зарниц.
6. Выпадение обильных осадков в виде дождя или снега.
5. Вы находитесь дома. Неожиданно вы почувствовали подземные толчки,
задребезжали стёкла, посуда. Времени, чтобы выбежать из дома, нет. Что вы будете
делать?
1.Отключу электричество, газ, воду.
2. Займу безопасное место в проёме двери.
3. Позвоню в аварийную службу.
4. Упакую документы.
5. Отойду от окон и предметов мебели, которые могут упасть.
6. Укроюсь в ванной комнате.
ВАРИАНТ № 2.
Выберите три верных ответа из шести предложенных.
1. Вы проживаете в селеопасном районе. Находясь дома, услышали по радио
сообщение об угрозе схода селя. У вас в запасе 30 минут. Каковы будут ваши
действия?
1. Плотно закроете вентиляционные и другие отверстия.
2. Закроете все двери, окна.
3. Предупредите соседей о сходе селя.
4. Будете выходить на склон горы, находящийся на селебезопасном направлении.
5. Будете выходить на склон горы через ущелье или небольшую долину.
6.Соберёте необходимое, то, что положено иметь с собой при эвакуации.
2. В случае схода оползней, селей, обвалов, лавин необходимо как можно скорее
оказаться в безопасном месте. Укажите их.
105

1. Склоны гор, где оползневые процессы не очень интенсивны.
2. Ущелья и выемки между горами.
3. Возвышенности, расположенные с противоположной стороны селеопасного
направления.
4. Большие деревья с толстыми стволами.
5. Склоны гор и возвышенностей, не вовлечённые в оползневой процесс.
6. Большие камни, за которыми можно укрыться.
3. Ниже предложены варианты причин образования селей. Выберите верные ответы.
1.Подвижки земной коры или землетрясения.
2. Наводнения, вызванные авариями на гидросооружениях.
3. Лесные и торфяные пожары.
4. Естественный процесс разрушения гор.
5. Прямое воздействие солнечных лучей на ледники.
6. Хозяйственная деятельность человека.
4. К каким последствиям приводят оползни, сели, обвалы и снежные лавины?
1. Изменение климата и погодных условий.
2. Перекрытие русел рек и изменение ландшафта.
3. Гибель людей и животных. 4. Разрушение зданий и сооружений.
5. Лесные пожары.
6. Извержение вулканов.
5.Что нужно сделать при заблаговременном оповещении об угрозе бурь, ураганов,
смерчей?
1.Убрать с подоконников, балконов и лоджей вещи, которые могут быть подхвачены
воздушным потоком.
2. Одеться потеплее, приготовить мобильный телефон.
3. Побыстрее вынести мусор из квартиры.
4. Сесть на машину и уехать в безопасный район.
5. Отключить газ, воду и электричество.
6. Приготовить запасы продуктов питания и питьевой воды.
ВАРИАНТ № 3.
Выберите три верных ответа из шести предложенных.
1.Находясь на улице, вы увидели, что приближается буря. Времени укрыться в доме
или других постройках у вас фактически нет. Неподалёку от себя вы увидели
канаву. Что вы будете делать?
1.Внимательно, но быстро определите расстояние до границы бури.
2.спрячетесь в стоящее рядом строительном вагончике.
3. Забежите и спрячетесь за угол первого попавшегося на пути здания.
4. Прыгнете в канаву, ляжете на дно и прижмётесь к земле.
5. Будете ждать, когда утихнут порывы ветра.
6. Перебежите улицу и спрячетесь там под деревом.
2. В зависимости от масштаба, повторяемости и наносимого ущерба наводнения
имеют классификацию. Какие бывают наводнения согласно этой классификации?
1. Низкие 2. Высокие. 3. Выдающиеся. 4. Малые. 5. Полные. 6. Опасные.
106

3. При наводнении происходит быстрый подъём и затопление прилегающей
местности. Назовите вторичные последствия наводнений.
1. Спрямление русел извилистых рек.
2. Перенос водой вылившихся из повреждённых хранилищ вредных веществ и
загрязнение ими обширных территорий.
3. Разрушение зданий и сооружений.
4. Взрывы промышленных объектов в результате действия волны прорыва.
5. Осложнение санитарно-эпидемиологической обстановки.
6. Заболачивание местности.
4. Какие меры безопасности следует предпринимать в сильную жару?
1.Носить светлую воздухопроницаемую одежду и головной убор.
2. Передвигаться не спеша; избегать прямого воздействия солнечных лучей, при
перегревании – охладить тело, перейти в тень, на ветер, принять душ, медленно
выпить
1-1,5 литра воды.
3. Есть почаще мороженное, держать постоянно включенным кондиционер.
4. Пить только охлаждённые воду и соки.
5. При значительном ухудшении состояния – обратиться к врачу; при тепловом ударе
и других значительных ухудшениях состояния кого-либо из окружающих – оказать
пострадавшему первую медицинскую помощь и вызвать врача или доставить
пострадавшего в медучреждение.
6. Больше времени находиться в воде ближайшего водоёма.
5. Вы в группе из 10 туристов остановились в лесу на привале около ручья. Все
очень устали, и руководитель принял решение разбить лагерь.
Быстро поставили
палатки, развели огонь, сварили обед, поели. После обеда ребята залезли в палатки и
уснули. Внезапно спящих разбудили крики: «Пожар! Огонь!». Все выскочили из
палаток. Вокруг горели трава, кусты. Огонь подбирался к палаткам. Дым был
повсюду, на расстоянии 15-20 метров почти ничего не было видно. Стало трудно
дышать. Ваши дальнейшие действия?
1.Будете выходить из зоны пожара в наветренную сторону.
2. Быстро соберёте палатки, рюкзаки и убежите из зоны пожара по направлению
ветра, сообщите органам местного самоуправления о пожаре.
3. Накроете голову и верхнюю часть тела мокрой одеждой, будете дышать через
мокрый платок или смоченную одежду.
4. Организуете тушение огня водой из ручья.
5. Укроетесь в палатк5ах, которые всё время будете поливать водой из ручья.
6. В случае сильного задымления пригнётесь и станете дышать воздухом,
прилегающем к земле.
ОТВЕТЫ.

2.2.

1

135

256

156

245

125

2

346

135

125

234

156

3

145

123

345

125

236
107

63-64 АЛГОРИТМ БЕЗОПАСНОГО ПОВЕДЕНИЯ ПРИ ВОЗНИКНОВЕНИИ ЧС
ТЕХНОГЕННОГО ХАРАКТЕРА

РЕКОМЕНДАЦИИ НАСЕЛЕНИЮ ПО ОБЕСПЕЧЕНИЮ
ЛИЧНОЙ БЕЗОПАСНОСТИ В УСЛОВИЯХ ЧРЕЗВЫЧАЙНЫХ СИТУАЦИЙ
ПРИРОДНОГО И ТЕХНОГЕННОГО ХАРАКТЕРА
Вариант № 1
1. Что такое землетрясение?
1) область возникновения подземного удара
2) подземные толчки и колебания поверхности Земли
3) проекция центра очага землетрясения на земную поверхность
4) это природное явление, связанное с процессами в окружающей атмосфере
2. По шкале итальянского ученого Меркалли в баллах оценивается(-ются):
1) сила землетрясения, его интенсивность
2) величина смещения земной коры в очаге землетрясения
3) величина разрывов в верхней части мантии
4) степень разрушений в эпицентре землетрясения
3. Чрезвычайные ситуации техногенного характера (наиболее характерные) по месту их
возникновения можно разделить на:
1) химические, биологические, военные
2) радиационные, химические, гидродинамические, транспортные
3) социальные, экологические, психологические
4) региональные, федеральные, радиационные
4. Экстренной мерой по защите населения от поражающих факторов чрезвычайной
ситуации является:
1) своевременное оповещение населения об опасностях, возникающих в условиях
чрезвычайных ситуаций
2) обеспечение населения средствами индивидуальной защиты
3) качественное обучение правилам поведения при возникновении чрезвычайных
ситуаций
4) эвакуация населения из опасных районов
5. Как вы будете действовать при получении сигнала оповещения о радиационной аварии,
если вы находитесь в своем доме (квартире)?
1) освободите от продуктов питания холодильник, вынесете скоропортящиеся продукты и
мусор, выключите газ, электричество, погасите огонь в печи и проследуете на сборный
эвакуационный пункт
2) включите радио и выслушаете сообщение, выключите электричество, наденете средства
индивидуальной защиты, вывесите на двери табличку «В квартире жильцов нет» и
проследуете на сборный эвакуационный пункт
3) немедленно закроете окна, двери, вентиляционные отверстия, включите радиоприемник
или телевизор и будете готовы к приему информации о дальнейших действиях
4) выключите газ, электричество, возьмете необходимые продукты питания, вещи и
документы, наденете средства индивидуальной защиты и проследуете на сборный
эвакуационный пуВариант №2
1. Вы находитесь дома одни. Вдруг задрожали стекла и люстры, с полок начали падать
посуда и книги. Вы срочно:
108

1) позвоните родителям на работу, чтобы предупредить о происшествии и договориться о
месте встречи
2) займете место в дверном проеме капитальной стены
3) закроете окна и двери, быстро спуститесь в подвальное помещение
4) подойдете к окну и узнаете у прохожих, что случилось
2. Что надо делать с получением сигнала о приближении урагана, бури, смерча при
нахождении в доме (квартире)?
1) закрыть окна, включить радиоприемник для получения информации от управления ГО
и ЧС
2) плотно закрыть двери и окна, отключить электроэнергию, занять безопасное место у
стен внутренних помещений
3) закрыть окна и двери с наветренной стороны здания, а с подветренной открыть, убрать
с балконов и подоконников вещи, которые могут быть подхвачены воздушным потоком
4) закрыть краны газовой сети, включить радиоприемник с автономным питанием для
получения информации от управления ГО и ЧС
3. Как следует выходить из зоны лесного пожара?
1) навстречу ветру, используя для этого просеки, дороги
2) перпендикулярно направлению ветра, используя для этого открытые пространства
3) если загорелась одежда, то нужно бегом покинуть опасную зону по направлению ветра
4) если вы в составе группы, то нужно разделиться и по одному выходить навстречу ветру
4. При химической аварии необходимо выполнить ряд действий. Среди приведенных
ниже вариантов ответов найдите ошибочный:
1) включить радиоприемник или телевизор для получения достоверной информации об
аварии и о рекомендуемых действиях
2) при поступлении информации об эвакуации надеть резиновые сапоги, плащ, взять
документы и необходимые вещи, продукты на трое суток и выходить из зоны возможного
заражения перпендикулярно направлению ветра
3) для защиты органов дыхания используйте ватно-марлевую повязку или подручные
изделия из ткани, смоченные в воде или 2-5% -ном растворе пищевой соды (для защиты
от хлора), 2% -ном растворе лимонной или уксусной кислоты (для защиты от аммиака)
4) для защиты органов дыхания используйте противогаз, а при его отсутствии ватномарлевую повязку или подручные изделия из ткани, смоченные в воде или 2% -ном
растворе лимонной или уксусной кислоты (для защиты от хлора), 2-5% -ном растворе
пищевой соды (для защиты от аммиака)
5. Сирены и прерывистые гудки предприятий и транспортных средств означают сигнал
оповещения:
1) «Тревога!»
2) «Внимание! Опасность!»
3) «Внимание всем!»
4) «Химическая (радиационная) опасность!»
ЕДИНАЯ ГОСУДАРСТВЕННАЯ СИСТЕМА ПРЕДУПРЕЖДЕНИЯ
И ЛИКВИДАЦИИ ЧРЕЗВЫЧАЙНЫХ СИТУАЦИЙ (РСЧС)
Вариант № 1
1. С какой целью создана РСЧС? Выберите правильный ответ:
109

1) прогнозирование чрезвычайных ситуаций на территории Российской Федерации и
организация проведения аварийно-спасательных и других неотложных работ
2) обеспечение первоочередного жизнеобеспечения населения, пострадавшего в
чрезвычайных ситуациях на территории Российской Федерации
3) объединение усилий органов центральной власти, органов исполнительной власти,
субъектов Российской Федерации, городов и районов, а также организаций, учреждений и
предприятий, их сил и средств в области предупреждения и ликвидации чрезвычайных
ситуаций
4) совершенствование подготовки руководящего состава и специалистов РСЧС по
действиям в чрезвычайных ситуациях
2. Для чего создаются территориальные подсистемы РСЧС? Выберите правильный ответ:
1) для ликвидации чрезвычайных ситуаций в городах и районах
2) для предупреждения чрезвычайных ситуаций в жилых и нежилых зданиях
3) для локализации чрезвычайных ситуаций на промышленных объектах
4) для предупреждения и ликвидации чрезвычайных ситуаций в субъектах Российской
Федерации в пределах их территорий
3. На каких уровнях действует Единая государственная система предупреждения и
ликвидации чрезвычайных ситуаций?
1) объектовый, производственный, местный
2) федеральный, межрегиональный, региональный, муниципальный, объектовый
3) поселковый, районный, региональный
4) территориальный, республиканский
4. Комиссия по предупреждению и ликвидации чрезвычайных ситуаций и обеспечению
пожарной безопасности органа местного самоуправления является координирующим
органом РСЧС на:
1) региональном уровне
2) федеральном уровне
3) муниципальном уровне
4) территориальном уровне
5. Органом повседневного управления РСЧС на муниципальном уровне является (-ются):
1) информационный центр органа исполнительной власти субъекта Российской
Федерации
2) центры управления в кризисных ситуациях
3) дежурно-диспетчерские службы организаций (объектов)
4) единые дежурно-диспетчерские службы муниципальных образований
Вариант № 2
1. В каких режимах могут функционировать органы управления и силы РСЧС?
1) в режиме постоянной готовности и повседневной деятельности
2) в режиме готовности к действиям при возникновении чрезвычайных ситуаций
3) в режимах готовности к оповещению населения и проведения аварийно-спасательных
работ
4) в режимах повседневной деятельности, повышенной готовности и режиме
чрезвычайной ситуации
110

2. Что не является основными мероприятиями, проводимыми органами управления и
силами единой системы в режиме повседневной деятельности? Найдите в приведенных
ответах ошибку:
1) изучение состояния окружающей среды и прогнозирование чрезвычайных ситуаций
2) проведение при необходимости эвакуационных мероприятий
3) подготовка населения к действиям в чрезвычайных ситуациях
4) пропаганда знаний в области защиты населения и территорий от чрезвычайных
ситуаций и обеспечения пожарной безопасности
3. С какой целью федеральными органами исполнительной власти созданы
функциональные подсистемы РСЧС?
1) организация работы в области защиты населения и территорий от чрезвычайных
ситуаций в сфере деятельности этих органов
2) разработка предложений по реализации государственной политики в области
предупреждения и ликвидации чрезвычайных ситуаций и обеспечения пожарной безопасности
3) разработка и реализация целевых и научно-технических программ и мер по
предупреждению чрезвычайных ситуаций и обеспечению пожарной безопасности
4) введение при необходимости круглосуточного дежурства руководителей и
должностных лиц органов управления и сил единой системы на стационарных пунктах
управления
4. Укажите закон, определяющий права и обязанности граждан России в области защиты
от чрезвычайных ситуаций:
1) Федеральный закон Российской Федерации «О безопасности»
2) Федеральный закон Российской Федерации «Об обороне»
3) Федеральный закон Российской Федерации «О защите населения и территорий от
чрезвычайных ситуаций природного и техногенного характера»
4) Федеральный закон Российской Федерации «О гражданской обороне»
5. Определите, какой нормативно-правовой акт закрепляет правовые основы обеспечения:
безопасности личности, общества и государства.
1) Федеральный закон Российской Федерации «Об обороне»
2) Федеральный закон Российской Федерации «О гражданской обороне»
3) Федеральный закон Российской Федерации «О безопасности»
4) Концепция национальной безопасности Российской Федерации
ТАБЛИЦЫ ПРАВИЛЬНЫХ ОТВЕТОВ
Выставляется оценка «отлично» за все правильные ответы на вопросы задания, за четыре
правильных ответа – «хорошо» и т.д. За выполнение задания с выбором ответа
выставляется 1 балл при условии, если обведен только один номер верного ответа. Если
обведены два и более ответов, в том числе правильный, то ответ не засчитывается.
Номер задания

Вариант

Рекомендации
населению по
обеспечению
личной
безопасности в

1
2

Вопросы
1
2
2

2
1
3

3
2
1

4
4
4

5
3
3

111

условиях
чрезвычайных
ситуаций
природного и
техногенного
характера
Единая
1
государственная 2
система
предупреждения
и ликвидации
чрезвычайных
ситуаций
(РСЧС)

3
4

4
2

2
1

3
3

4
3

65-66 АЛГОРИТМ БЕЗОПАСНОГО ПОВЕДЕНИЯ ПРИ ВЕДЕНИИ ВОЕННЫХ
ДЕЙСТВИЙ И В СЛЕДСТВИИ ЭТИХ ДЕЙСТВИЙ
Тестирование
1.Проживание населения в жилых домах с ограниченным пребыванием на открытой
местности относится к … радиационной защиты.
этапам
режимам
уровням
способам
2.Противогазовые респираторы защищают от…
воздействия паро-, газообразных вредных веществ
воздействия ОВ, АХОВ, находящихся в газообразном состоянии
вредных веществ, присутствующих в воздухе в виде паров, газов и аэрозолей
неограниченного содержания вредных веществ
3.Виды защиты, НЕ применяемые для защиты населения и экономики от ЧС
радиационная
зоологическая
инженерная
фитопатологическая
4. Комплекс организационных и технических мероприятий, направленных на обеспечение
безопасности людей, на предотвращение пожара, ограничение его распространения и
создание условий его тушения называется …
пожарной защитой
пожарной безопасностью
пожарным процессом
пожарным изменением
5.Сигнал, подаваемый при обнаружении начала выпадения радиоактивных веществ на
территории населенного пункта
«Внимание всем»
«Всем в противорадиационные укрытия»
«Радиационная опасность»
«Внимание, на территории вводиться радиационная обстановка»
6.Порядок действий населения и применения средств и способов защиты в зонах
заражения (загрязнения) с целью максимального снижения возможных доз поражения
112

радиационная и химическая разведка
радиационный и химический контроль
режим защиты
обеззараживание участков
7. Коэффициент защиты, который должно иметь противорадиационное укрытие для
персонала некатегорированных объектов в зонах возможного опасного радиоактивного
заражения, но за границей возможных сильных разрушений -не менее …
200
100
50
10
8.Не является исходными данными для планирования эвакуации и рассредоточения
города (района) …
общая численность населения, проживающего в городе, районе
особенности производственной деятельности
санитарное состояние населенных пунктов
наличие защитных сооружений, их вместимость и защитные свойства
9.Медико-биологическая защита достигается в результате осуществления комплекса
мероприятий, которые включают в себя …
применения различных средств и способов защиты
прогнозирование медико-санитарных последствий
режимно-ограничительные мероприятия
10.К защитным сооружениям ГО относятся
убежища 5 классов
ПРУ 3 классов
погреба, подвалы и приспособленные жилые помещения
специально оборудованные подземные переходы, метро, горные
все варианты
11.Медицинские средства защиты
водно-масляная эмульсия
индивидуальный противохимический пакет
средства защиты кожи и органов дыхания
12.Комплекс мероприятий по организованному вывозу (выводу) из городов, отнесенных к
группе по ГО и размещение в загородной зоне для проживания и отдыха рабочих и
служащих объектов экономики, производственная деятельность которых в военное время
будет продолжаться в этих городах, называется …
рассредоточением
эвакуацией
способом защиты
обеспечением производства и жизнедеятельности объектов экономики
13.Решение о проведении эвакуации в мирное время принимает (ют) …
руководители органов местного самоуправления и Президент РФ
руководитель объекта экономики или жилищно-эксплуатационного управления
председатель правительства РФ и руководители органов исполнительной власти
субъектов РФ
руководители органов исполнительной власти субъектов РФ и органов местного
самоуправления
14.Территории, из которых необходимо проводить эвакуацию и рассредоточение
зоны, где возможно опасное химическое заражение при концентрациях аммиака 10 мг *
мин/л
зоны катастрофического затопления с добеганием волны до 4 часов
113

зоны, где возможно опасное химическое заражение при концентрациях сероуглерода 20
мг * мин/л
15.Решение о проведении эвакуации в военное время принимает (ют) …
руководители органов исполнительной власти субъектов РФ
Президент РФ, председатель правительства РФ
председатель правительства РФ и руководители органов исполнительной власти
субъектов РФ
Президент РФ и руководители органов исполнительной власти субъектов РФ
ДИФФЕРЕНЦИРОВАННЫЙ ЗАЧЕТ
ДИФФЕРЕНЦИРОВАННЫЙ ЗАЧЕТ ПО ОБЖ 1 ВАРИАНТ
Вопрос № 1
Кто является Верховным Главнокомандующим Вооруженными Силами Российской
Федерации?
Министр обороны
Президент РФ
Государственная Дума
Министр внутренних дел
Совет безопасности
Вопрос № 2
В каком году были созданы Вооружённые Силы РФ?
1922 г.
1938 г.
1999 г.
1992 г.
Вопрос № 3
Назовите основные виды Вооруженных Сил Российской Федерации?
Военно-Морской Флот (ВМФ)
Войска противовоздушной обороны (ПВО)
Ракетные войска стратегического назначения (РВСН)
Воздушно-Космические Силы (ВКС)
Сухопутные войска (СВ)
Воздушно десантные войска (ВДВ)
Вопрос № 4
Под воинской обязанностью понимается:
прохождение военной службы в мирное и военное время, самостоятельная подготовка
к службе в Вооруженных Силах
установленный законом почетный долг граждан с оружием в руках защищать свое
Отечество, нести службу в рядах Вооруженных Сил, проходить вневойсковую подготовку
и выполнять другие связанные с обороной страны обязанности
114

долг граждан нести службу в Вооруженных Силах только в период военного
положения и в военное время
Вопрос № 5
Военная служба исполняется гражданами:
В ВС РФ, пограничных войсках и войсках ГО
В ВС РФ, других войсках, органах и формированиях
Только в ВС РФ
Вопрос № 6
Граждане РФ проходят военную службу:
По призыву и по контракту
Только по призыву, по достижении определенного возраста
Только в добровольном порядке
Вопрос № 7
Персональный воинский учет ведется:
Районными (городскими) военными комиссариатами
Управлениями (отделами) кадров военных округов
Специально уполномоченным сотрудником органа управления образованием района
(города)
Вопрос № 8
В соответствии с Федеральным законом "О воинской обязанности и военной службе"
первоначальная постановка на учет осуществляется:
В период с 1 января по 31 марта в год достижения гражданами возраста 17 лет
В период с 1 января по 31 марта в год достижения гражданами возраста 18 лет
В период с 1 января по 31 марта в год достижения гражданами возраста 16 лет
Вопрос № 9
Какой правовой акт устанавливает права и свободы военнослужащих:
Федеральный закон "О воинской обязанности и военной службе"
Указ Президента Российской Федерации "О создании ВС РФ"
Федеральный закон "О статусе военнослужащих"
Вопрос № 10
Призыв граждан на военную службу проводится:
Один раз в год с 1 апреля по 30 июля
На основании приказа министра обороны РФ с 1 апреля по 31 декабря
Два раза в год с 1 апреля по 15 июля и с 1 октября по 31 декабря
Вопрос № 11
В каком возрасте призываются мужчины на военную службу в Российскую армию?
от 16 до 18 лет
от 18 до 27 лет
от 28 до 32 лет
от 33 до 35 лет
115

Вопрос № 12
Первый контракт о прохождении военной службы вправе заключать граждане в возрасте:
от 21 до 45 лет
от 16 до 35 лет
от 18 до 40 лет
Вопрос № 13
К общевоинским уставам Вооруженных Сил Российской Федерации относятся:
Устав внутренней службы Вооруженных Сил РФ, Устав гарнизонной и караульной
служб Вооруженных Сил РФ, Дисциплинарный устав Вооруженных Сил РФ, Строевой
устав Вооруженных Сил РФ
Устав внутренней и гарнизонной служб Вооруженных Сил РФ, Устав караульной
службы Вооруженных Сил РФ Устав корабельной службы ВМФ РФ, Строевой устав
Вооруженных Сил РФ, Дисциплинарный устав Вооруженных Сил РФ
Устав внутренней службы ВС РФ, Устав гарнизонной службы ВС РФ,
Дисциплинарный устав ВС РФ, Строевой устав ВС РФ
Вопрос № 14
Какие ограничения вводятся по отношению к военнослужащим в соответствии с
законодательством Российской Федерации?
какие-либо ограничения отсутствуют
запрещение бастовать, пикетировать
запрещение на участие в политических акциях и занятиях коммерческой
деятельностью
Вопрос № 15
Федеральным законом "О воинской обязанности и военной службе" установлены:
обязательная подготовка к военной службе
необходимая подготовка к военной службе
принудительная подготовка к военной службе
специальная подготовка к военной службе
добровольная подготовка к военной службе
полная подготовка к военной службе
Вопрос № 16
Право гражданина Российской Федерации на замену военной службы альтернативной
гражданской службой определено:
в Федеральном законе "О статусе военнослужащих"
в Федеральном законе "О воинской обязанности и военной службе"
в Конституции Российской Федерации
Вопрос № 17
Граждане, состоящие в запасе, могут призываться на военные сборы
продолжительностью:
до 2 месяцев, но не чаще одного раза в 2 года
до 3 месяцев, но не чаще одного раза в 3 года
116

до 2 месяцев, но не чаще одного раза в 3 года
до 3 месяцев, но не чаще одного раза в 2 года
Вопрос № 18
Какой категории профессиональной пригодности гражданина, призываемого на военную
службу, соответствует формулировка "рекомендуется условно"?
первой
второй
третьей
четвертой
Вопрос № 19
В какой период осуществляется первоначальная постановка на воинский учёт граждан
женского пола после получения ими военно-учетной специальности?
С 15 октября по 31 декабря
С 1 января по 31 марта
В течение всего календарного года
С 1 апреля по 30 июня
Не осуществляется вообще
Вопрос № 20
На сколько периодов разделяют обязательную подготовку гражданина к военной службе?
1
2
3
4
Вопрос № 21
Какой орган государственной власти осуществляет руководство ВС РФ:
Государственная дума
Генеральный штаб
Парламент
Президент
Министерство обороны
Вопрос № 22
Назовите Самостоятельные рода войск Вооруженных Сил Российской Федерации?
Военно-морской флот
Сухопутные войска
Воздушно десантные войска
Космические войска
Войска противовоздушной обороны
Ракетные войска стратегического назначения
Вопрос № 23
117

Назовите составные части организационной структуры Вооруженных Сил РФ?
Виды Вооруженных сил
Рода войск
Виды войск
Рода Вооруженных сил
Вопрос № 24
Какую ответственность несут военнослужащие за проступки, связанные с нарушением
воинской дисциплины, норм морали и воинской чести:
Дисциплинарную
Уголовную
Административную
Вопрос № 25
Уклонившимся от исполнения воинской обязанности считается гражданин:
Явившийся по вызову военного комиссариата без необходимых документов
Не явившийся по вызову военного комиссариата в указанный срок без уважительной
причины
Не явившийся по вызову военного комиссариата в указанный срок, даже имея
уважительную причину
Вопрос № 26
Гражданам, признанным временно не годными к военной службе, предоставляется
отсрочка от призыва для обследования и лечения на срок:
6 или 12 месяцев
12 или 18 месяцев
3 или 6 месяцев
Вопрос № 27
Началом военной службы для граждан, не пребывающих в запасе и признанных на
военную службу, считается:
День прибытия в воинское подразделение
День принятия воинской присяги
День убытия из военного комиссариата к месту службы или день внесения в списки
воинской части
Вопрос № 28
На какой срок заключается контракт для поступающих впервые на службу на должности
солдат, сержантов и им равных?
на один год
на два года
на три года
на пять лет
Вопрос № 29
В каком году вступил в силу Федеральный закон "Об альтернативной гражданской
службе"?
118

2001 года
2002 года
2003 года
2004 года
Вопрос № 30
В какие сроки осуществляется призыв граждан России на действительную военную
службу?
с 1 октября по 31 декабря
с 1 января по 31 марта
с 1 апреля по 15 июля
в любые сроки
ДИФФЕРЕНЦИРОВАННЫЙ ЗАЧЕТ ПО ОБЖ 2 ВАРИАНТ
Вопрос № 1
Каким требованиям должны отвечать граждане, принимаемые по контракту на военную
службу?
должны соответствовать основной группе здоровья
должны соответствовать уровню образования 8-ми классов
должны соответствовать медицинским, психологическим, физическим требованиям,
службу по конкретным специальностям в соответствующих видах (родах) войск
должны соответствовать уровню профессиональной и общеобразовательной
подготовки
Вопрос № 2
Когда гражданин приносит военную присягу в современных условиях?
в соответствии с Конституцией
в соответствии с Федеральным законом "Об альтернативной гражданской службе"
после прохождения начальной военной подготовки, но не позднее двух месяцев со
дня прибытия в воинскую часть
Вопрос № 3
Одним из направлений проводимой в настоящее время военной реформы является:
перевод комплектования Вооружённых Сил и других войск на контрактную основу
создание превосходства ВС России над ВС блока НАТО
уточнение задач Вооружённым Силам на участие во внутренних вооружённых
конфликтах
Вопрос № 4
Какой род войск называют "Ядерный щит России"?
Ракетные войска стратегического назначения
Военно-морской флот
119

Воздушно-десантные войска
Воздушно-космические силы
Вопрос № 5
Какую ответственность несут военнослужащие за совершенные правонарушения?
не несут никакой ответственности
за проступки, связанные с нарушением воинской дисциплины, норм морали и чести,
они несут дисциплинарную ответственность, которая установлена в соответствии с
Дисциплинарным уставом
военнослужащие могут быть привлечены к материальной ответственности за
причинённый ущерб государству при исполнении обязанностей военной службы
за совершение преступления военнослужащие могут привлекаться к уголовной
ответственности
Вопрос № 6
Служебное подчинение младших старшим, нижестоящих органов вышестоящим, а также
исполнение правил служебной, в том числе воинской, дисциплины во взаимоотношениях
между различными по служебному положению и званию лицами - это:
дисциплинированность
субординация
исполнительность
уважительное отношение
Вопрос № 7
По результатам профессионального психологического отбора определяется оценка о
профессиональной пригодности гражданина к исполнению обязанностей в сфере военной
деятельности. Она может быть такой:
рекомендуется в первую очередь
рекомендуется во вторую очередь
рекомендуется условно
не рекомендуется
рекомендуется в основном
рекомендуется с ограничениями
рекомендуется
Вопрос № 8
Гражданин при первоначальной постановке на воинский учет подлежит медицинскому
освидетельствованию врачами-специалистами:
терапевтом, хирургом, невропатологом, психиатром, окулистом, отоларингологом,
стоматологом, а в случае необходимости - врачами других специальностей
терапевтом, физиотерапевтом, травматологом, психиатром, окулистом,
эндокринологом и др.
хирургом, терапевтом, кардиологом, физиотерапевтом, пульмонологом,
стоматологом, окулистом и др.
Вопрос № 9
Военно-Морской Флот - это:
120

род войск, обеспечивающий выполнение боевых задач по разгрому военно-морских
сил противника
вид войск, обеспечивающий решение боевых задач с применением специальной
морской техники и морского вооружения
вид ВС, предназначенный для нанесения ударов по промышленно-экономическим
центрам, важным военным объектам противника и разгрома его военно-морских сил
Вопрос № 10
Какой орган государственной власти осуществляет оперативное руководство ВС РФ:
Государственная дума
Генеральный штаб
Парламент
Президент
Министерство обороны
Вопрос № 11
Наиболее активная форма реализации гражданами воинской обязанности называется:
Воинский учет
Призыв на военную службу
Призыв на военные сборы
Прохождение военной службы по контракту
Прохождение военной службы по призыву
Вопрос № 12
Тыл Вооружённых Сил - это:
силы и средства, осуществляющие тыловое и техническое обеспечение армии и флота
вид войск, осуществляющий тыловое обеспечение армии и флота в мирное и военное
время
род войск, осуществляющий техническое обеспечение армии и флота в мирное и
военное время
Вопрос № 13
Окончанием военной службы считается день:
В который истек срок военной службы
Подписания приказа об увольнении с военной службы
Передачи личного оружия другому военнослужащему
Вопрос № 14
В каких случаях предоставляется право на замену военной службы по призыву
альтернативной гражданской службой?
в случае, если несение военной службы тяжело для гражданина
в случае, если несение военной службы противоречит убеждениям гражданина
в случае, если несение военной службы противоречит вероисповеданию гражданина
в случае, если гражданин относится к коренному малочисленному народу, ведёт
121

традиционный образ жизни, осуществляет традиционное хозяйствование и занимается
традиционными промыслами
Вопрос № 15
Какие наказания ожидают гражданина, уклоняющегося от призыва на военную или
альтернативную службу в соответствии со статьёй 328 Уголовного Кодекса Российской
Федерации?
в виде лишения свободы на срок до 15 суток
в виде лишения свободы на срок до двух лет
штраф в размере до 200000 рублей
в виде лишения свободы на срок до трёх лет
Вопрос № 16
Что определяет устав внутренней службы Вооружённых Сил Российской Федерации?
порядок дежурства
порядок прохождения медицинского освидетельствования военнослужащих
определяет общие права и обязанности военнослужащих и взаимоотношения между
ними
определяет обязанности основных должностных лиц, правила внутреннего порядка и
другие вопросы повседневной жизни и быта, подразделений и частей
Вопрос № 17
По результатам медицинского освидетельствования врачами-специалистами могут быть
вынесены следующие заключения о годности к военной службе:
годен к военной службе
пригоден к военной службе
годен к военной службе с незначительными ограничениями
условно годен к военной службе
ограниченно годен к военной службе
неограниченно годен к военной службе
временно не годен к военной службе
не годен к военной службе
Вопрос № 18
Запас Вооруженных Сил Российской федерации предназначен:
для подготовки населения к ведению военных действий в случае мобилизации
для развертывания армии при мобилизации и ее пополнения во время войны
для создания резерва различных военных специалистов, необходимых ВС РФ при
мобилизации
Вопрос № 19
Профессиональный психологический отбор граждан, призываемых на военную службу,
осуществляется с целью:
определения индивидуального физического развития призывников, так как с первых
дней военной службы они испытывают значительные нагрузки
обеспечения соответствия индивидуально-психологических качеств граждан,
122

призываемых на военную службу, современным требованиям в Вооруженных Силах
Российской Федерации
определения качества освоения образовательных программ по начальной военной
подготовке и основам военной службы
Вопрос № 20
Принцип военного руководства, при котором командиры и начальники наделены всей
полнотой распорядительной власти в отношении подчиненных и несут ответственность за
все стороны жизни и деятельности военнослужащих, находящихся под их командой, - это:
принцип демократического централизма
принцип единоначалия
принцип взаимодействия
Вопрос № 21
Современная воинская деятельность может быть условно разделена на три основных вида:
в мирное время, в военное время, деятельность после военных действий
учебная, строевая, боевая
боевая, учебно-боевая, повседневная
Вопрос № 22
Какую ответственность несут военнослужащие за правонарушения и преступления против
военной службы:
Дисциплинарную
Уголовную
Административную
Вопрос № 23
Преданность своему Отечеству, любовь к Родине, стремление служить ее интересам и
защищать ее от врагов - это:
героизм
патриотизм
воинский долг
мужество
Вопрос № 24
Общевоинские уставы Вооруженных Сил Российской Федерации регламентируют:
действия военнослужащих при ведении военных операций
основы ведения боевых действий воинских подразделений и частей
жизнь, быт и деятельность военнослужащих
Вопрос № 25
Руководящим принципом в системе боевой подготовки войск ВС РФ является положение:
"Учить войска тому, что необходимо на войне"
"Что бесполезно на войне, то вредно вводить в мирное обучение"
"Просвещение ума есть важнейшая часть в воспитании каждого военного и
невоенного человека"
Вопрос № 26
123

Кто из граждан России освобождается от призыва на военную службу?
признанные не годными или ограниченно годными к военной службе по состоянию
здоровья
по личному желанию гражданина
прошедшие военную службу в вооружённых силах другого государства
имеющие неснятую судимость за совершение тяжкого преступления
Вопрос № 27
На какой максимальный срок заключается контракт с поступающими на службу на
должности прапорщиков (мичманов) и офицеров?
на два года
на три года
на десять лет
на пять лет
Вопрос № 28
Когда были созданы Ракетные войска стратегического назначения (РВСН)?
1949 г.
1959 г.
1987 г.
1999 г.
Вопрос № 29
Какой категории профессиональной пригодности гражданина, призываемого на военную
службу, соответствует формулировка "рекомендуется"?
первой
второй
третьей
четвертой
Вопрос № 03
Как называют человека, состоящего на воинском учете, до момента отправки его со
сборного пункта к месту прохождения военной службы?
Допризывником
Военнообязанным
Призывником
Военнослужащим.

124


Наверх
На сайте используются файлы cookie. Продолжая использование сайта, вы соглашаетесь на обработку своих персональных данных. Подробности об обработке ваших данных — в политике конфиденциальности.

Функционал «Мастер заполнения» недоступен с мобильных устройств.
Пожалуйста, воспользуйтесь персональным компьютером для редактирования информации в «Мастере заполнения».